SlideShare ist ein Scribd-Unternehmen logo
1 von 94
Downloaden Sie, um offline zu lesen
Universidad La Salle.
Facultad Mexicana de Medicina.
Curso de Extensión Universitaria para la Preparación del Examen Nacional para
Aspirantes a Residencias Médicas.
Examen de Ginecología y Obstetrícia.
1.- Se trata de paciente femenino de 29 años de edad con tumor anexial de 6 cm líquido, dolor
abdominal, fiebre, leucorrea, con historia de cervicovaginitis de repetición y dispareunia
crónica, última menstruación hace una semana. El diagnóstico más probable de ésta paciente
es:
a) Cistoadenoma
b) Enfermedad pélvica inflamatoria
c) Embarazo ectópico
d) Quiste de ovario
La EIP puede cursar con los siguientes síntomas:
 Dolor abdominal bajo (incluyendo dolor anexial, dispareunia). Es el síntoma más
frecuente (95%)
 Aumento del flujo vaginal, flujo de características anormales (74%)
 Sangrado anormal (intermestrual, poscoital) (45%)
 Síntomas urinarios (35%)
 Vómitos (14%)
 Es posible la ausencia de síntomas
Y en ella podemos encontrar estos signos:
 Dolor a la movilización del cuello, dolor anexial en la exploración vaginal bimanual
(99%)
 En el examen con espéculo observamos cervicitis y descarga endocervical purulenta
(74%)
 Fiebre (> 38º C) (menos del 47%).
 Masa pélvica: sugiere abceso tuboovárico (ATO)
 Peritonitis
CRITERIOS CLINICOS PARA EL DIAGNOSTICO DE SALPINGITIS
a. Dolor abdominal con o sin rebote.
b. Sensibilidad a la movilización del cérvix.
c. Sensibilidad anexial.
Los tres criterios anteriores son necesarios para establecer el diagnóstico, con uno o más de
los siguientes:
a. Extendido de Gram de endocérvix positivo, para diplococos gram negativos
intracelulares
b. Temperatura mayor de 38°C
c. Leucocitosis (mayor de 10.000 por c.c.)
d. Material purulento (positivo para leucocitos) en la cavidad peritoneal obtenido
por culdocentesis o laparoscopia.
Establecido el diagnóstico clínico de EPI, se debe hacer la definición del estado clínico y
anatómico de la patología pélvica:
a) No complicada (limitada a trompas u ovarios)
1) Sin peritonitis pélvica
2) Con peritonitis pélvica
b) Complicada (masa inflamatoria o absceso que compromete trompa (s) u ovario (s)
1) Sin peritonitis pélvica
2) Con peritonitis pélvica
o Beigi RH, Wiesenfeld HC. Pelvic inflammatory disease: new diagnostic criteria and
treatment. Obstet Gynecol Clin Norh Am. 2003; 30 (4): 777 – 93
o Center for Disease Control. Guidelines for treatment of sexually transmited diseases.
MMWR Recomm Rep. 2002 May 10;51(RR-6):1-78
o Center for Disease Control. Guidelines for prevention and management (MMWR. 40: 1 -
25 1991) Pelvic inflammatory disease: guidelines for prevention and management.
MMWR Recomm Rep. 1991 Apr 26;40(RR-5):1-25.
o Hager WD, Eschenbach DA, Spence MR, Sweet RL. Criteria for diagnosis and grading
of salpingitis. Obstet Gynecol. 1983 Jan;61(1):113-4.
o Prodigy Guidance. Pelvic inflammatory disease. [Internet]. UK : NHS, Department of
Health; 2003. [Acceso 18 de Junio de 2005]. Disponible en:
o Ross J. Pelvic inflammatory disease. Clin Evid. 2004 Dec;(12):2259-65.
o Royal College of Obstetricians and Gynecologists. Pelvic Inflammatory Disease.
Guideline nº 32. [Internet]. RCOG; Mayo 2003. [Acceso 18 de Junio de 2005].
2.-Femenino de 25 años con antecedentes de G/2, P/1, C/1 acude al servicio de consulta
externa, refiere que presenta una secreción transvaginal bastante líquida, de baja viscosidad,
maloliente de color amarillo y gris, espumoso.
El tratamiento de elección para esta entidad es:
a) Metronidazol 500 mg. VO c/12 por 7 a 10 días
b) Ampicilina 1g VO c/ 6 hrs.
c) Clotrimazol + Metronidazol 500 mg VO c/12 hrs. por 7 a días
d) Clindamicina 300 mg VO cada 12 hrs. x 5 días.
Referencias bibliográficas:
1. Secretaría de Salud. Norma Oficial Mexicana NOM -039-SSA2-2002, Para la prevención y
control de las infecciones de transmisión sexual. D.O.F. 19 de Septiembre 2003.
2. Kettler H, White K, Hawkes S. Mapping the landscape for sexually transmitted infections:
key findings and recommendations. Geneva, TDR (TDR/STI/ IDE/04.1).
3. CDC. Trends in Reportable Sexually Transmitted Diseases in the United States. CDC,
National Report. 2004
4. Distribución de los casos nuevos de enfermedades por mes Estados Unidos Mexicanos
2004. Sistema Único de Información para la Vigilancia Epidemiológica/Dirección General
de Epidemiología/SSA
5. Aral S O. Sexual risk behavior and infection: epidemiological considerations. Sex. Transm.
Inf. 2004;80:8-12
3.- Femenino de 25 años, con embarazo de término, sin antecedentes de control prenatal. G 3.
C-1. Se ingresa al servicio de obstetricia por presentar actividad uterina regular y dolorosa. Ef.:
Deambulante, tranquila, adecuada coloración de tegumentos, abdomen con fondo uterino a 32
cm. con producto único vivo en situación transversa dorso inferior FCF 144, al tacto vaginal
cérvix dilatado a 3 cm. y membranas íntegras. Se realiza cesárea con retención de placenta e
invasión a vejiga.
Estudio de gabinete de elección previo al evento obstétrico es:
a) prueba sin estrés
b) perfil biofísico
c) simple de abdomen
d) ultrasonido doppler
PLACENTA:
 Se divide en:
 Placenta acreta.
 Placenta increta.
 Placenta percreta.
 ACRETA: Las vellosidades se adhieren al miometrio.
 INCRETA: Penetran más de la mitad del espesor del miometrio.
 PERCRETA: Atraviesa todo el espesor del miometrio, llegando a la
serosa, incluso atravesándola y adhiriéndose a órganos vecinos.
Factores:
 Endometriósis previa.
 Tumores submucosos. (Miomas)
 Cicatríz uterina previa. (Cesárea, miomectomía)
 Implantación baja. (Placenta previa)
 Malformaciones placentarias. (Placenta extracorial)
 Legrado enérgico previo.
 Extracción manual previa de una placenta.
Diagnóstico Pre-parto
 Ultrasonografía Döppler.
 Resonancia Magnética.
Diagnóstico transparto-:
 Placenta retenida por más de 20 minutos.
 Imposibilidad para encontrar un plano de separación placentaria cuando se intenta su
extracción manual.
 Hemorragia incontrolable después de la pseudoextracción.
 El diagnóstico histopatológico corrobora el diagnóstico clínico.
 Escenario menos deseable.
Tratamiento:
 Histerectomía Obstétrica.
 Constituye una cirugía no planeada y secundaria al hallazgo del acretismo
placentario con sangrado incohercible.
 Cesárea-Histerectomía. (Con diagnóstico previo)
 Cirugía planificada ante un correcto diagnóstico prenatal.
 Recomendación ACOG:
 Maduración pulmonar intrauterina.
 Inyectar al cordón umbilical 50 mg de metrotexate.
 Ligar el cordón en el nacimiento placentario y dejar la placenta in-situ.
 Embolización inmediata de arterias uterinas bilaterales, así como de ramas de
la división anterior de la arteria iliaca interna con alcohol polivinílico.
 Continuar con 5 dosis I.M. de 50 mg de metrotexate y cuantificar niveles de
βhCG.
 Programar Histerectomía Total Radical Abdominal y/o Cistectomía parcial y/o
resección pared anterior recto.
Lee et al. Conservative Management of Placenta Percreta. Obstet Gynecol, 112(2):421-424
4.- Mujer que acude al servicio de ginecología por referir ciclos opso-menorreicos desde el
inicio de su menarquia, ha incrementado 15 Kg. de lo que pesaba habitualmente, se aprecia
una gran cantidad de acné, pero además refiere depilarse el área del bigote cada semana, y
cree que esto le sensibiliza la piel para que aumente el acné.
El diagnóstico más probable en esta paciente es:
a) Sx. De Asherman
b) Sx. Stein Leventhall
c) Sx. Amenorrea Galactorrea
d) Sx. Karman
Síndrome de Ovario Poliquístico (SOP) es uno de los más comunes trastornos endocrinos que
afectan a las mujeres alrededor del 5% al 10% de las mujeres en edad reproductiva (12-45
años) y se piensa que es una de las principales causas de la infertilidad femenina. Las
características principales son la obesidad, anovulación (dando lugar a la menstruación
irregular) o amenorrea, acné, y las cantidades excesivas o los efectos de androgénicos
(masculinizantes) hormonas. Los síntomas y la severidad del síndrome varían mucho entre las
mujeres. Si bien las causas son desconocidas, resistencia a la insulina, la diabetes y la
obesidad están fuertemente correlacionadas con el SOP.
Bulun SE, Adashi EY. The physiology and pathology of the female reporductive axis. In:
Kronenberg HM, Melmed S, Polonsky KS, Larsen PR, eds. Williams Textbook of
Endocrinology. 11th ed. Philadelphia, Pa: Saunders Elsevier; 2008:chap 16.
5.- Se trata de mujer de 43 años, G- 4, P-3, A-1, con diagnóstico de anemia ferropénica, de 9.5
g/dl, refiere ciclos menstruales de 31,32 x 8,9 días de duración, acompañados de coágulos, los
cuales aparecieron después del nacimiento de su segundo hijo hace 13 años. E.F.: Buen
estado general, TA 130/80, genitales con evidencia de sangrado activo, al tacto vaginal se
detecta útero de consistencia firme voluminosa, irregular, aproximadamente de 12 cm. anexos
libres.
El diagnóstico más probable es:
a) Adenomiosis uterina.
b) Cáncer cervicouterino.
c) Miomatosis uterina.
d) Hiperpalsia adenomatosa de endometrio.
MIOMATOSIS UTERINA
Definición:
Tumor benigno que se origina en el miometrio, por lo que su componente histológico
predominante es el tejido muscular y, en menor medida, el conectivo y fibroso. El único
tratamiento efectivo es el quirúrgico; sin embargo, sólo requieren ser tratados aquellos que
producen síntomas.
Evaluación y Diagnóstico:
Historia:
1. El síntoma más frecuente suele ser la hemorragia uterina.
2. Los síntomas principales están relacionados con el crecimiento del tumor.
3. La paciente puede notar una masa en hipogastrio o abdomen inferior.
4. La masa se puede asociar a dolor pélvico, o manifestaciones por compresión de
órganos o estructuras vecinas.
5. Puede haber alteración de la fertilidad.
Examen Físico:
1. Se debe realizar con la vejiga y el recto vacuo.
2. El hallazgo primordial es el aumento de volumen y consistencia del útero, el cual puede
ser simétrico (nódulos submucosos) o irregular (nódulos intramurales o subserosos).
Exámenes Auxiliares:
1. Papanicolau cérvico–vaginal: Indicado siempre; permite descartar neoplasia epitelial
cervical o cáncer infiltrante de cérvix.
2. Ultrasonido transabdominal y transvaginal: Indicado siempre; permite evaluar la
localización, tamaño y número aproximado de miomas.
3. Hemoglobina, hematocrito: Indicado cuando hay historia de sangrado; orienta en la
severidad del sangrado y anemia.
4. Hemograma y VSG: Indicado cuando hay historia de fiebre; si es anormal sugiere
infección o necrosis del mioma (puede ser apropiado descartar infección de otro órgano
o sistema).
5. Grupo sanguíneo y factor Rh: Si hay anemia severa o en el preoperatorio.
6. Perfil de coagulación (tiempo de protrombina y de tromboplastina parcial, recuento de
plaquetas): Si hay historia de sangrado exagerado.
7. Gonadotrofina coriónica (sub–unidad sérica): Permite descartar posible embarazo en
casos de ciclos irregulares, retraso menstrual o útero de consistencia blanda.
Diagnóstico Diferencial:
1. Embarazo.
2. Tumor de ovario.
3. Enfermedad inflamatoria pélvica, complejo inflamatorio anexial pélvico.
4. Endometriosis.
5. Adenomiosis.
6. Tumor extragenital: colon, retroperitoneo.
Referencias Bibliográficas:
1. Hillard PA. Benign Diseases of the Female Reproductive Tract: Symptoms and Signs.
En: Berek JS, Adashi EY, Hillard PA, eds. Novak's Gynecology. Baltimore: Williams and
Wilkins, 1996:331-97.
1. Hutchins FL, Greenber MD. Miomas Uterinos: Diagnóstico e Indicaciones de
Tratamiento. Clinicas de Ginecología y Obstetricia. Temas Actuales. 1995;5:609-14.
2. Davis KM, Sclass WD. Tratamiento Médico para Miomatosis Uterina. Clinicas de
Ginecología y Obstetricia. Temas Actuales. 1995;5:671-81.
1. Selwyn P, Oskowitz MB. Leiomyomata Uteri. En: Friedman EA, ed. Gynecological
Decision Making. St. Louis: Mosby, 1983:148-9.
1. Diaz Huamán V. Tumores Benignos del Aparato Reproductor Femenino. En: Ludmir A,
Cervantes R, Castellano C, eds. Ginecología y Obstetricia, Prevención - Diagnóstico -
Tratamiento. Lima: Concytec, 1996:907-25.
6.- Femenino de 34 años que cursa con 38.5 semanas de gestación que ingresa al servicio
con trabajo de parto. En el transcurso de trabajo de parto durante la dilatación presenta dolor
intenso y brusco. A la exploración usted observa metrorragia escasa y aumento del tono
uterino a la palpación abdominal que resulta muy doloroso. El diagnóstico más probable es:
a) Placenta previa.
b) Rotura de vasos previos.
c) Crioamnionitis hemorrágica.
d) Desprendimiento de placenta.
Fisiopatología de la hemorragia
La hemorragia es el signo fundamental que domina el cuadro clínico de la placenta previa. Para
explicar su mecanismo existen distintas teorías:
Mecanismo de Jacquemier: Se produce crecimiento armónico de la placenta y del útero hasta
la semana 26, 28. Después el segmento inferior crece más deprisa y favorece el
despegamiento lo que origina la hemorragia en el embarazo.
Mecanismo de Schroeder: Las contracciones uterinas en el parto traccionan del segmento
inferior hacia arriba y empujan al feto hacia abajo despegando la placenta.
Mecanismo de Pinard: Explica las hemorragias gestacionales y del parto. El estiramiento de las
membranas de la zona de menor radio (orificio interno cervical) como consecuencia de las
contracciones tira de la placenta y la desprenden.
Mecanismo de Bartholomew: Explica la hemorragia en los casos de placenta previa central. La
zona placentaria que reviste el orificio interno es un área isquémica ya que no recibe vasos
deciduales. A este nivel disminuye la presión sanguínea, por lo que la sangre tiende a dirigirse
hacia esta zona y escapa por la cara materna.
En el alumbramiento también puede haber una hemorragia importante producida por un doble
mecanismo:
-desprendimiento parcial antes de la expulsión en los casos de placenta oclusiva.
-atonía uterina en la zona de inserción después de expulsada la placenta y vascularización
anómala.
Manifestaciones clínicas de la placenta previa en el embarazo
- Síntomas: Principalmente la hemorragia. Toda hemorragia vaginal acontecida en el tercer
trimestre debe hacer pensar en una placenta previa. Las hemorragias suelen ser espontáneas,
no acompañadas de dolor, de sangre roja y se presentan de forma intermitente, con intervalos
variables entre las mismas. Progresivamente se van haciendo más frecuentes y más graves. La
primera hemorragia suele aparecer en forma inesperada generalmente nocturna, cesando en
menos de media hora. Las hemorragias ulteriores son más graves y más precoces.
- Signos: La consecuencia fundamental es la anemia materna que depende de la cuantía de la
hemorragia (la sangre es de origen materna ya que procede de espacios intervellosos).
- Exploración:
Exploración general para valorar la existencia de signos de anemia.
Exploración obstétrica: valorar el tamaño del útero (adecuado para la edad gestacional), es
blando e indoloro. A menudo la estética fetal está alterada (transverso, oblicuo, nalgas). No
debe efectuarse tacto vaginal cuando haya existido hemorragia en embarazo avanzado por el
riesgo de infección y de despegamiento y aumentar así la hemorragia.
Auscultación fetal normal.
Exploración ecográfica: es una técnica fundamental en el diagnóstico de la placenta previa.
Permite determinar la localización placentaria y la variedad de la placenta previa. En general, la
placenta puede identificarse a partir de la 9ª semana. No obstante, a lo largo de la gestación
por crecimiento uterino se produce un cambio en sus relaciones con el útero ("emigración
placentaria", imagen de desplazamiento). Así el diagnóstico de certeza de límites placentarios
solo puede establecerse hacia la semana 34. Siempre será necesario hacer una adecuada
identificación del orificio cervical interno (más fácil con sonda transvaginal). Añadiendo al
estudio Doppler color se observa la vascularización y las zonas que sangran.
7.- Se trata de femenino de 24 años de edad gesta 1, tuvo un parto vaginal espontáneo con un
producto con peso de 4,350 g. después de 5 minutos de tracción suave del cordón umbilical se
expulsó la placenta, que parece estar intacta. Se inició el masaje del fondo uterino y se pidió a
la enfermera que administrara 20 unidades de oxitocina en 100 ml de solución Ringer lactato.
Después de una inspección cuidadosa del canal del parto se observa una laceración de
segundo grado y una laceración de 2 cm en la pared vaginal izquierda que se intentó reparar.
En la exploración física se encuentra un fondo uterino blando y atónico. Los signos vitales son:
temperatura 37.1°C, TA 164/92, FC 130x’, FR 18 X’. El tratamiento más adecuado en éste
caso es:
(a) Oxitocina 10 unidades directas en goteo intravenoso
(b) Metilergonovina 0.2 mg IM
(c) Prostaglandina F 0.25 mg IM
(d) Legrado
Morgan M, Siddighi S. Ginecología y obstetricia, National Medical Series. 5° edición. Mc Graw
Hill. Pp. 28. La atonía uterina es la causa más común de hemorragia puerperal. El masaje
energético y la oxitocina diluida no han sido útiles para interrumpir la hemorragia y por tanto el
siguiente paso es agregar un fármaco uterotónico. La metilergonovina está contraindicada
porque la paciente se encuentra hipertensa a pesar de la hemorragia intensa, el siguiente
fármaco es la prostaglandina. La administración de oxitocina no diluida, 10 UI por vía IV podría
causar hipotensión grave. La exploración manual podría ser apropiada si se sospecha
laceración como causa de hemorragia. El legrado es apropiado para la hemorragia puerperal
tardía, cuando se sospecha retención de los productos de la concepción.
8.- Se trata de paciente femenino de 20 años de edad, que acude a consulta refiriendo
presentar 6 meses sin regla. Se manifiesta preocupada por su sobrepeso, ha estado a dieta y
ha perdido 6 kg en 8 meses. Actualmente pesa 46 Kg. con talla de 1,65 mts. Signos vitales
dentro de sus parámetros normales. Se realiza prueba de embarazo con resultado negativa. La
causa más probable de su amenorrea es:
a) Disgenesia gonadal.
b) Hipogonadismo hipogonadotropo.
c) Síndrome de ovario poliquístico.
d) Adenoma hipofisario.
Sigue siendo práctica y eficaz la diferenciación de los hipogonadismos en hipergonadotrópicos
(con fallo de función en la gónada) e hipogonadotrópicos (con fallo en hipotalámo y/o hipófisis).
La deficiencia puede ser total o parcial y afectar a una o ambas funciones gonadales
(producción de esteroides y germinal). Las causas son muy variadas. En la tabla 1 se han
recopilado las principales de origen hipogonadotrópico, aunque algunas de ellas raramente se
observan durante la infancia (p. ej., hiperprolactinemia) y en la tabla 2 las de origen
hipergonadotrópico. Dentro del apartado de “idiopático” poco a poco se van dilucidando las
causas y, así, con la ayuda de la biología molecular se encuentran mutaciones del gen del
receptor de GnRH, de LH o alteraciones del gen DAX 1. Por otra parte debe tenerse en cuenta
que en diferentes síndromes puede aparecer un hipogonadismo hipogonadotrópico (tabla 3).
MANIFESTACIONES CLÍNICAS Y ANALÍTICAS La ausencia de datos clínicos de
hipogonadismo durante la infancia es la regla. Tan sólo la presencia de micropene o
criptorquidia en algún varón puede hacer sospechar de la existencia de un hipogonadismo,
pero ni es patognomónica (se da también en el déficit de GH y en otras situaciones no
hormonalmente deficitarias), ni su ausencia excluye la deficiencia gonadal. Por lo tanto, y si se
descarta la sospecha del hipogonadismo por otras características asociadas (p. ej., diversos
síndromes, alteraciones evidentes: tumoraciones o radioterapia hipotálamo-hipofisaria), sólo el
retraso de aparición de la pubertad será la primera indicación de una deficiencia de estas
características. Sin embargo, aquí se produce el principal problema para alcanzar un
diagnóstico temprano al resultar muy difícil distinguirlo de una variante normal del desarrollo: el
retraso constitucional del crecimiento y adolescencia. Respecto al hipogonadismo
hipogonadotrópico los estudios hormonales son escasamente discriminatorios en la época
previa a la pubertad por lo que el momento en que se realiza el diagnóstico, y por lo tanto el
inicio del tratammiento, suele ser tardío. La ausencia de caracteres sexuales secundarios, junto
con un tamaño infantil de las gónadas (por palpación testicular o ecografía ovárica), es
suficiente para valorar una situación prepuberal. En todo caso, la confirmación, según el sexo,
de bajos niveles de testosterona circulante, o de estrógenos (directamente o por citología
vaginal/vesical), permiten una sencilla, fácil y rápida comprobación. No debe olvidarse que en
relación a la causa del hipogonadismo (p. ej., una tumoración hipofisacualquier edad, es
objetivar una anosmia (síndrome de Kallmann), más aún si es parcial. En los casos de
hipogonadismo hipergonadotrópico puede encontrarse en ciertos periodos a lo largo de la
infancia niveles elevados de gonadotrofinas basales y que tras estímulo con GnRH muestran
una respuesta más elevada de lo normal. Sin embargo, lo más habitual es encontrarse con
niveles propios de la infancia al realizar las determinaciones basales. Recientemente se ha
demostrado por medio de una técnica muy sensible que no era cierta la pretendida ausencia de
estradiol en las niñas, por lo que si se consigue generalizar su empleo será un arma valiosa de
diagnóstico durante la infancia.
TABLA 1. Causas de hipogonadismo hipogonadotrópico
Panhipopituitarismo
Idiopático
Tumoración hipofisaria o hipotalámica
Misceláneo
Enfermedad granulomatosa
Traumatismo
Vasculitis
Infarto
Hemocromatosis
Deficiencia aislada de gonadotrofinas
Síndrome de Kallman y variantes
Hipogonadismo hipotalámico idiopático
Deficiencia aislada de LH
Deficiencia aislada de FSH
Hipoplasia adrenal congénita ligada al cromosoma X
Defectos de desarrollo de la línea media
Síndromes con diversas malformaciones (tabla 2)
Prader-Willi
Laurence-Moon-Bardet-Biedl
Alteraciones sistémicas
Deficiencias nutritivas
Enfermedades crónicas
Ponderales: desnutrición severa y obesidad mórbida
Yatrogenia
Farmacológica
Radioterápica
Quirúrgica
Hiperprolactinemia
Ejercicio físico de alta competición
Retraso constitucional de la pubertad (?)
BIBLIOGRAFÍA SELECCIONADA:
Henkin RI, Bartter FC Studies on olfactory thresholds in normal man and in patients
with adrenal cortical insufficiency: The role of adrenal cortical steroids and of serum
sodium concentration. J Clin Invest 1966; 45: 1631-1639. 2. Schwanzel-Fukuda M,
Abraham S, Crossin KL, Edelman GM, Pfaff DW. Immunocytochemical demonstration
of neural cell adhesion molecule (NCAM) along the migration route of luteinizing
hormone-releasing hormone (LHRH) neurons in mice J Comp Neurol 1992; 321: 1-18.
3. Schwanzel-Fukuda M, Bick D, Pfaff DW: Luteinizing hormone-releasing hormone
(LHRH)-expressing cells do not migrate normally in an inherited hypogonadal
(Kallmann) syndrome. Mol Brain Res 1989; 6: 311-326.
9.- Se trata de paciente femenino de 18 años de edad la cual acude a consulta por presentar
amenorrea acompañada de profundas alteraciones del olfato. De los siguientes el diagnóstico
más probable es:
a) Síndrome de amenorrea-galactorrea.
b) Amenorrea de causa uterina.
c) Síndrome de ovario poliquístico.
d) Amenorrea por alteración hipotalámica.
• AMENORREA HIPOTALAMICA. (Hipogonadotrópica)
– Psicógena. (Stress emocional)
– Anorexia nerviosa. (Deficiencia nutricional)
– Ejercicio excesivo. (Carrera de fondo, natación, gimnasia, ballet)
– Fármacos. (Fenotiazina, reserpina, bloqueadores ganglionares,
anticonceptivos)
– Pseudociésis.
– Síndrome de Kallman (Deficiencia selectiva de gonadotropinas y anosmia).
El ejemplo clásico de la alteración hipotalámica que lleva a desórdenes del ciclo menstrual es
el Síndrome de Kallman.
Lectura Recomendada:
Etiopatogenia de la amenorrea hipotalámica funcional Interacción de las respuestas
hormonales del Sistema Nervioso Central y Neuropéptidos Periféricos
Revista Argentina de Endocrinología y Metabolismo
Copyright © 2008 por la Sociedad Argentina de Endocrinología y Metabolismo
Vol 45 • No. 2
10.- Una mujer de 27 años G/4, P/0 con 6 SDG acude a su primera visita prenatal. Su historia
obstétrica pasada es importante porque tiene tres pérdidas de producto en el segundo
trimestre. Refiere que en las tres ocasiones al presentarse al hospital presentaba dilación
cervical completa. No recuerda haber tenido contracciones dolorosas. Niega antecedentes
médicos y quirúrgicos. El examen físico es normal incluyendo un examen pélvico que muestra
un cervix largo y cerrado. Después de una larga discusión con la paciente ella pide que se le
practique un cerclaje durante este embarazo. El momento más apropiado para realizarlo es:
a) Inmediatamente
b) 13 a 17 semanas
c) 24 a 28 semanas
d) 32 a 36 semanas
El cerclaje cervical tiene sus indicaciones en la profilaxis y tratamiento de la incompetencia
cervical.
La incompetencia o insuficiencia cervical representa un 10% de las causas de parto pretérmino
y está asociada a una importante morbimortalidad neonatal.
Las modificaciones cervicales en el segundo trimestre de gestación son causa de parto
prematuro y pueden deberse a:
1) Incompetencia cervical.
2) Pérdida de tejido conectivo tras una cirugía cervical (conización).
3) Defectos congénitos como la hipoplasia cervical tras exposición a dietilestilbestrol.
4) Infección intrauterina. Hasta un 51.5% de las pacientes con clínica compatible con
incompetencia cervical enmascaran un cuadro de infección intraamniótica subclínica.
Diferenciamos tres tipos de cerclaje:
1. El cerclaje se considera profiláctico o electivo (o primario) cuando se realiza de forma
electiva por historia previa de incompetencia cervical antes de evidenciar cambios en el cerviz
y generalmente suele realizarse entre las 13 y 17 semanas de gestación.
2. El cerclaje terapéutico secundario que se realiza tras la detección, en el seguimiento
obstétrico, de modificaciones en el cérvix antes de las 26 semanas de gestación. Se realiza en
pacientes con un riesgo potencial de parto pretérmino.
3. El cerclaje terapéutico terciario, en caliente, de rescate o “emergent cerclage” que se
realiza en pacientes que presentan la membrana amniótica visible a través del orificio cervical
externo o en vagina.
GUIA CLÍNICA:
INDICACIONES DEL CERCLAJE
Unitat de Prematuritat. Servei de Medicina Maternofetal.
Institut Clínic de Ginecologia, Obstetrícia i Neonatologia, Hospital Clínic de Barcelona
Responsables del protocolo: T.Cobo, M. López, M. Palacio
Creación: 24/01/07
Modificaciones: 05/09/07
Última actualización: 17/01/10
11.- Se trata de femenino de 67 años refiere intenso prurito vulvar y sensación quemante, al
examen el introito vaginal se encuentra estenótico. De los siguientes es el tratamiento más
apropiado es:
a) 5-fluoracilo
b) Testosterona tópica
c) Corticoesteroides fluorados
d) Estrógeno tópico
Vulvovaginitis atrófica
El hipoestrogenismo conduce a atrofia de la vagina y el vestíbulo vulvar, que los hace
fácilmente irritables y susceptibles a infecciones secundarias. Las pacientes refieren sensación
de quemadura, prurito, disuria, hipersensibilidad y dispareunia. Puede encontrarse al examen
físico atrofia, fisuras superficiales, y un flujo vaginal acuoso1. Hay disminución del tamaño del
introito2, pérdida de la rugosidad y la vagina toma una apariencia lisa y brillante.
Los hallazgos histológicos revelan un epitelio vaginal delgado, disminución de los lechos
capilares, y la citología muestra, a medida que la atrofia progresa, aumento de las células
basales y disminución o ausencia de las células superficiales2.
Se aconseja evitar el uso de jabones y demás irritantes de la piel. Se pueden utilizar lubricantes
simultáneamente con los estrógenos o como terapia única, si hay alguna contraindicación a las
hormonas.
El tratamiento con estrógenos por vía sistémica o transvaginal mejora y restaura los signos y
síntomas, y una a dos semanas después de iniciar el tratamiento los cambios de atrofia
empiezan a mejorar rápidamente, se reduce el pH y se induce maduración vaginal y de la
mucosa uretral, reduciendo la frecuencia de las infecciones urinarias3. La dosis y vía de
administración debe ser debidamente individualizada4. Contraindicaciones al tratamiento con
estrógenos, incluyen: la presencia de tumores estrógenosensibles, falla hepática terminal y
antecedentes de tromboembolización relacionada con ellos.
Menopausia y Piel. Parte II: Manifestaciones clínicas
dermatológicas durante la menopausia
MARÍA ISABEL BARONA C. Docente adjunto. Dermatóloga Universidad del Valle-Cali.
12.- Mujer de 26 años, es atendida en consulta en la clínica de displasias por papanicolau con
lesión NIC I. Antecedentes: menarca 14 años, ritmo 30x5 eumenorreica, inicio de vida sexual a
los 15 años, 2 parejas sexuales, método de planificación familiar oclusión tubaria bilateral,
gestas 3 partos 3, cérvix con lesión acetoblanca con extensión lineal de 2 cm. Para confirmar
el diagnóstico se debe realizar:
a) Crioterapia de lesión.
b) Captura de híbridos.
c) Repetir colposcopía
d) Biopsia de la lesión.
9.5.2 Las pacientes a quienes se les realizó citología cervical, cuyo resultado es LEIBG
(infección por VPH, displasia leve o NIC 1); LEIAG (displasia moderada y grave o NIC 2 y 3) o
cáncer deben enviarse a una clínica de colposcopía, para realizar estudio colposcópico.
9.5.3 Si el resultado de la citología es LEIBG, la colposcopía es satisfactoria y sin evidencia de
LEIBG, se realizará control citológico en un año (Apéndice Normativo A)
9.5.4 Si la citología es de LEIBG, la colposcopía es satisfactoria y existe evidencia de
lesión, se debe tomar una biopsia dirigida.
9.5.4.1 Si la biopsia dirigida es negativa, se realizará nueva colposcopía para verificar el
diagnóstico y en caso necesario, tomar nueva biopsia dirigida y revalorar.
9.5.4.2 Si la biopsia dirigida es reportada como LEIBG se podrá dar tratamiento conservador:
criocirugía, electrocirugía o laserterapia (sólo si cumple con las condiciones referidas en el
Apéndice 1) o se podrá mantener a la paciente en vigilancia en la clínica de colposcopía, con
colposcopía y estudio citológico cada seis meses, durante 24 meses.
Jueves 31 de mayo de 2007 DIARIO OFICIAL (Primera Sección)
9.5.4.3 Si la biopsia dirigida es reportada como LEIAG (Lesión Intraepitelial Escamosa de Alto
Grado) se realizará tratamiento conservador (electrocirugía o laserterapia). En las mujeres
posmenopáusicas, dependiendo de las condiciones anatómicas del cérvix, se realizará
tratamiento conservador en la clínica de colposcopía o tratamiento quirúrgico (histerectomía
extrafascial) en el servicio que corresponda.
9.5.4.4 Si la biopsia dirigida reporta cáncer microinvasor o invasor, la paciente se transferirá a
un Servicio o Centro Oncológico para su tratamiento correspondiente.
9.5.4.5 Si la citología reporta LEIBG y la colposcopía es no satisfactoria, se tomará cepillado
endocervical (Apéndice Normativo A)
9.6 En caso de colposcopía no satisfactoria, negativa a LEIBG y con cepillado endocervical
negativo, se continuará su control en la clínica de colposcopía en seis meses, con colposcopía
y citología.
9.6.1.1 Si el cepillado endocervical reporta LEIBG se tratará a la paciente como LEIAG, con
métodos conservadores escisionales.
Jueves 31 de mayo de 2007 DIARIO OFICIAL (Primera Sección)
Modificación a la Norma Oficial Mexicana NOM-014-SSA2-1994, Para la prevención,
detección, diagnóstico, tratamiento, control y vigilancia epidemiológica del cáncer
cérvico uterino.
Al margen un sello con el Escudo Nacional, que dice: Estados Unidos Mexicanos.- Secretaría
de Salud.
MODIFICACION A LA NORMA OFICIAL MEXICANA NOM-014-SSA2-1994, PARA LA
PREVENCION,
DETECCION, DIAGNOSTICO, TRATAMIENTO, CONTROL Y VIGILANCIA EPIDEMIOLOGICA
DEL CANCER CERVICO UTERINO.
7.3 El resultado del estudio citológico es descriptivo y debe ser informado de la siguiente
manera:
a.- Negativo a cáncer.
b.- Negativo con proceso inflamatorio.
c.- Displasia leve (NIC 1).
d.- Displasia moderada (NIC 2).
e.- Displasia grave (NIC 3).
f.- Cáncer del cuello del útero in situ (NIC 3).
g.- Cáncer microinvasor e invasor.
h.- Adenocarcinoma.
i.- Maligno no especificado
13.- Se trata de paciente de 33 años que cursa con 39 SDG; a la exploración física reflejos
patelares hiperactivos, inquieta, se reportan cifras de TA 145/95, se realiza laboratorio que
reporta proteinuria 2+,. El diagnóstico más probable es:
a) Glomerulonefritis aguda
b) Hipertensión esencial
c) feocromocitoma
d) Preeclampsia
La hipertensión es la complicación médica más común del embarazo , aunque para algunos
autores es la segunda complicación médica del embarazo sólo después de la anemia; es más
frecuente en jóvenes durante el primer embarazo y en nulíparas de mayor edad, hipertensas
previas y diabéticas.
En México, también es la complicación más frecuente del embarazo, la incidencia es de 47.3
por cada 1 000 nacimientos y es además, la primera causa de ingreso de pacientes
embarazadas a las unidades de terapia intensiva (debido a hemorragia masiva, para recibir
soporte hemodinámico), según la secretaría de salud (2001) la mortalidad por complicaciones
del embarazo ocupa el 15º lugar en la mortalidad hospitalaria en general. Además, la tasa de
preeclampsia se ha incrementado 40% en el periodo entre 1990 y 1999 y constituye hasta 40%
de los partos prematuros iatrogénicos.
.
Preeclampsia
La preeclampsia es un síndrome clínico caracterizado por hipertensión con disfunción orgánica
múltiple, proteinuria, edemas.
Es definida como un incremento de al menos 140/90 mmHg después de la semana 20 de
gestación, un incremento en la presión sanguínea diastólica de al menos 15 mmHg respecto a
un nivel previo a la semana 20 combinado con proteinuria (> 300 mg en 24 horas). Las
mediciones de la presión arterial citadas deben ser medidas al menos 2 ocasiones con por lo
menos 6 horas de separación. La proteinuria puede ser una toma simple de orina al azar que
indique al menos 30 mg/dL 3 ó ++ en dos muestras de orina1 según el tipo de prueba. El criterio
del incremento de 30 mmHg en la presión sistólica y/o 15 mmHg en la presión diastólica
respecto a valores previos a la semana 20 de gestación ha sido eliminado por ser poco
específico15
Myers JE, Baker PN. Hupertensive diseases and eclampsia. Curr Opin Obstet Gynecol 2002;
14: 119-125
2. Tierney, McPhee, Papadakis. Diagnóstico clínico y tratamiento 2003. 38ª ed, México, Manual
Moderno, 2003: 770-773
3. Wilson MI, Goodwin TM, Pan VI, Ingles SA. Molecular epidemiology of preeclampsia. Obstet
and Gynecol Survey 2003; 58(1):39-66
4. Burrow GM. Complicaciones médicas durante el embarazo. 4ª ed, México, McGraw-Hill
panamericana: 1996: 1-25
5. Guyton AC, Hall JE. Embarazo y lactancia en: Tratado de fisiología médica, 10ª ed, México,
McGraw-Hill Interamericana 2001: 1135-45
6. Vaticon D. Fisiología de la fecundación, embarazo, parto y lactancia, en: Tresguerres JAF.
Fisiología Humana. México, Interamericana McGraw-Hill, 1992: 1086-1109
7. Pridjian G, Puschett JB. Preeclampisa. Part 1: Clinical and Pathophysiologic Considerations.
Obstet and Gynecol Survey 2002; 57 (9): 598-618
8. Pridjian G, Puschett JB. Preeclampisa. Part I1: Experimental and Genetic Considerations.
Obstet and Gynecol Survey 2002; 57 (9): 619-40
9. IMSS. Embarazo de alto riesgo. Guía diagnóstica terapéutica. Rev Med IMSS 1998;
36(1):45-60
14.- Femenino de 36 años, con presencia de hiper y polimenorreas o pérdidas de sangre
contínuas sin conservación del ciclo, se sospecha de miomatosis uterina, éste tipo alteración
se presenta con mayor frecuencia en los miomas de localización:
a) Submucoso
b) Intramural
d) Intraligamentaria
e) cervical
Descripción
Los miomas son tumores monoclonales benignos de las células del músculo liso del miometrio.
Están compuestos por grandes cantidades de matriz extracelular que contiene colágeno,
fibronectina y proteoglicanos. El colágeno tipo I y tipo II es abundante, pero las fibrillas de
colágeno son anormales y están desorganizadas, de modo similar a lo que se observa en la
formación de queloides.
Los miomas submucosos son los menos frecuentes, constituyendo únicamente el 5% de la
totalidad de los miomas, a menudo producen un aumento del sangrado menstrual en forma de
hiper y polimenorreas, e incluso hemorragias importantes que exigen tratamiento de urgencia.
Las metrorragias también son habituales en este tipo de miomas. Según Novak, el peligro de
degeneración sarcomatosa es mucho mayor en los miomas submucosos, e igualmente es
causa de dismenorreas más intensas y frecuentes.
BIBLIOGRAFÍA
1. De la Fuente U. Tratado de Obstetricia y Ginecología. Mc Graw-Hill. (Madrid). 1998.
Volumen II.
2. DI SAIA S. Tratado de Obstetricia y Ginecología de Danforth. Sexta. Mc Graw-Hill. (Nueva
York). 1990. Sexta Edición.
3. González-Merlo J. Tratado de Obstetricia y Ginecología. Salvat Editores S.A. (Barcelona).
1990. Quinta Edición.
4. Aller J., Pages G. Obstetricia Moderna. Tercera Edición. Mc Graw-Hill. (Caracas). 1999.
5. Formación Médica Continuada en Atención Primaria. Marzo 1995. Vo. 5, Nº (3).
15.- Se trata de mujer de 26 años, es atendida en consulta por secreción láctea bilateral hace 4
meses y menstruaciones cada 21 días, en escasa cantidad desde hace 6 meses.
Antecedentes: hace 6 meses padece gastritis tratada con cimetidina durante 2 meses y
posteriormente ha tomado el medicamento de forma irregular. a.g.o.: menarca 12 años, ritmo
30/7, núbil. e.f.: talla 1.63, peso 54 kg. Al efectuar compresión en glándulas mamarias se
produce salida de secreción láctea bilateral, resto normal.
El estudio de elección en esta paciente para confirmar el diagnóstico es:
a) perfil hormonal ginecológico.
b) prolactina sérica.
c) radiografía de silla turca.
d) ultrasonido mamario.
La hiperprolactinemia (hiperprl) es un trastorno frecuente, más en la mujer que en el hombre, y
puede ser la manifestación de un adenoma hipofisario (prolactinoma). La medición de
prolactina (prl) en la sangre es ahora un procedimiento de rutina en la detección de trastornos
de la menstruación y galactorrea que se resuelven efectivamente con el uso de fármacos con
actividad dopaminérgica (bromocriptina y cabergolina). La causa más frecuente de hiperprl es
por el uso de fármacos que inhiben la acción de dopamina, pero se debe descartar inicialmente
hipotiroidismo y un prolactinoma mediante resonancia magnética.
Los prolactinomas son fácilmente controlados con dopaminérgicos y excepcionalmente se
requiere cirugía. En caso de infertilidad por exceso de prl existe una respuesta favorable con
bromocriptina o cabergolina, aunque en la actualidad se prefiere la última por su efectividad y
menos efectos indeseables.
Hiperprolactinemia. Guía terapéutica y diagnóstica Arturo Zárate*
* Unidad de investigación de endocrinología, diabetes y metabolismo,
Centro médico nacional IMSS y hospital ángeles México.
16.- Se trata de femenino de 31 años nuligesta pero con actividad sexual regular, sin método
de planificación familiar, con ciclos regulares, sin leucorrea, refiere dispareunia profunda,
sangrado intermestrual y dismenorrea secundaria ocacionalmente presenta urgencia urinaria,
el método de elección para confirmar el diagnóstico clínico de ésta paciente es:
a) Cultivo de exudado vaginal
b) Ultrasonido
c) Biopsia de endometrio
d) Laparoscopia con biopsias dirigidas
Endometriosis
Examinación bimanual
Dolor en los ligamentos utero‐sacros
Nodularidad en el fondo de saco
Induración del septo rectovaginal
Útero en retroversión fijo
Masa anexial
Endometriosis
Laparoscopia
visualización directa
estadificación
toma de muestras
Estadio I Minimo
Estadio II Leve
Estadio III Moderado
Estadio IV Severo
Referencias bibliográficas
1. Ruiz V. Endometriosis y fertilidad. Ed. Acosta y Warman, pp. 99
2. López, VH. Palomo E. Incidencias de endometriosis en una población infértil. XXI Congreso
nacional de Ginecología y Obstetricia. Guatemala, 1993.
3. El-Eoley, et al. Danazol but not ginadotropin releasing hormone agonists suppresses
autoantibodies in endomeriosis. Fertil Steril 1990; 54:725
4. Acosta AA. Buttram VC Jr. Besch PK, Malinak LR, Van Der Heyden J. A.proposed
classfication of pelvic endometriosis. Obstet Gynecol 1973;42:19.
5. Buttran VC Jr. Evolution of the revised American Fertility classification of endometriosis. Fert.
Steril 1985; 43: 347
6. López VH. Tratamiento médico-quirúrgico de la endometriosis. Simposio El rostro cambiante
de la endometriosis panamá 3. 12. 1993.
7. Steinleitner A. Heterolous transplation of activated murine peritonel macrophages inhibitis
gamete interaction in vivo; A paradigm fo endometriosis associted subfertility. Fertil Steril 1990;
54:725.
8. Damewood M. Effect of serum from patients with minimal to mild endometriosis on mouse
embryo growth. Fertil Steril 1990; 54: 917
9. Proug S. Peritoneal fluid fracctions from patients with endometriosis do not promote two-cell
mouse embryo growth. Fertil Steril 1990; 54: 927.
17.- Femenino de 22 años, que presenta una tumoración de 2 cm de diámetro en el cuadrante
ínfero-externo de la mama izquierda, indolora, de consistencia firme, superficie lisa, forma
ovoidea, móvil y bien delimitada del parénquima vecino, sin antecedentes de derrame por el
pezón, sin “piel de naranja” ni retracción del pezón. El diagnóstico más probable es:
a) Fibroadenoma.
b) Carcinoma.
c) Ectasia de los conductos mamarios.
d) Quiste solitario.
FIBROADENOMA MAMARIO
Tumor benigno más frecuente en las mujeres entre los 20 y 35 años.
ETIOLOGIA
Existen múltiples teorías siendo la más aceptada la hormonal, generalmente son únicos, solo el
20% son múltiples o bilaterales. De tamaño variable hasta de 10 cm. Ocupa el 13.6% de la
patología mamaria benigna.
CUADRO CLÍNICO
Lesión nodular de consistencia dura, de larga evolución y no dolorosa. Normalmente llegan a
los 3 cm. De diámetro. Durante la fase tardía del ciclo menstrual el tumor suele presentar un
leve aumento de tamaño. Durante la menopausia presentan regresión hasta la calcificación
(signo de palomitas de maíz).
DIAGNOSTICO
Es clínico, se presenta como un tumor bien delimitado, desplazable, no adherido a piel ni a
planos profundos, liso o multilobulado en ocasiones. Se localiza frecuentemente en cuadrantes
externos.
EXAMENES DIAGNOSTICOS
ULTRASONIDO MAMARIO .- Identifica un nódulo sólido, bien delimitado de bordes regulares .
TRATAMIENTO.-
Conservador con vigilancia estrecha dependiendo del tamaño y en caso de ser necesario
exéresis del nódulo para estudio histopatológico
hospitalgeneral.salud.gob.mx/
BIBLIOGRAFIA
1. Sánchez BC. Tratado de Enfermedades de la glándula mamaria. Ed. Manual Moderno. Cap.
13- 15.
2.- De Vita V. Cancer of the Breast. In Cancer: Principles and Practice of Oncology: Fifth Ed.
Philadelphia: Lippincott-Raven, Chapter 36; pp: 1521-1616.
3.-Consenso Nacional Acerca del Tratamiento de Cáncer de Mama. En Tumores de mama:
Diagnóstico y Tratamiento. 2ª Ed. McGraw-Hill Interamericana; pp: 119-126.
4.-Eberlein T. Current management of carcinoma of the breast. Ann Surgery 1994; 220: 121-
136.
18.- A 26-year-old woman presents with malodorus gray-wellow discharge. You take a wet
mount preparation and observe “Clue cells”. The agent of this infection and its treatment is:
a) Gardnerella vaginalis / clindamycin
b) Gardnerella vaginalis / Ketoconazole
c) Trichomona vaginalis / metronidazole / treat the partner
d) Candida albicans / nistatin
Gardnerella vaginalis fue clasificada como una sola especie y fue establecida como agente
causal de la vaginosis (antes conocida como vaginitis inespecífica). El cuadro clínico que
presenta es caracterizado por una secreción blanca o blanco-grisácea que se percibe
generalmente después de la relación sexual con olor fétido aminado (pescado). El diagnóstico
certero es la base para evitar posibles complicaciones como la enfermedad inflamatoria
pelviana y las complicaciones del embarazo. El tratamiento se basa principalmente en los
fármacos como: metronidazol y clindamicina, debido a su efectividad y espectro, pero como
todos se deben emplear con adecuada prudencia debido a su toxicidad. Además de que se
deben corregir o modificar los factores predisponentes, ya que esta patología va en aumento
convirtiéndose por su frecuencia en un problema de salud pública.
Bibliografía:
1. Hernández F. Gardnerella vaginalis mobiluncus en la etiología de la vaginosis bacteriana.
Rev Costarricense Ciencias Médicas 1998; 19: 57-61.
2. Hansen EA. Gardnerella. Rev Ginecol 2005; 25: 99.
3. Espinosa I, Lorenzo M, Bentancourt A, Riverón Y, Romero M. Caracterización bioquímica y
antigénica de diferentes aislamientos de Gardnerella vaginalis.
Rev Cubana Invest Biomed 2005; 24: 22-7.
4. Taylor F. Vaginal flora morphotypic profiles and assessment of bacterial vaginosis in women
at risk for HIV infection.Infect Dis Obstet Gynecol
2004; 12: 121-6.
19.- Una mujer embarazada, puede afectar al feto y hacerlo contraer lesiones importantes
durante el embarazo o al salir al exterior (atravesando el canal de parto), sí la gestante se
encuentra afectada de la siguiente patología:
a) Tricomonas
b) Gardenerella
c) Herpes genital
d) Gonococos
Herpes genital
La prevalencia de herpes simplex genital o tipo 2 (VHS-2) en mujeres embarazadas varía entre
7 y 33% en distintas series. La prevalencia ha experimentado un sostenido aumento durante
los últimos años. Se estima que aproximadamente 1 a 3% de las mujeres adquiere cada año la
infección. En el caso de parejas discordantes, la tasa de adquisición aumenta hasta 10 a 30%
anual. La adquisición durante el embarazo es ~2%.
La transmisión al hijo ocurre principalmente cuando la mujer embarazada adquiere una
infección primaria. La transmisión es de 30 a 50% cuando la infección primaria ocurre cerca del
momento del parto. La mayor transmisión (85%) ocurre durante el parto. Sin embargo, también
puede ocurrir transmisión intrauterina (5-8%) y post-natal (8-10%). Los factores que inciden en
la transmisión son: infección primaria mucho mayor eficiencia que infección recurrente, parejas
discordantes, títulos de anticuerpos maternos y procedimientos obstétricos invasores, (los que
están absolutamente contraindicados).
Las manifestaciones en la mujer embarazada son principalmente bajo la forma de herpes
genital localizado, muy raramente ocurre diseminación cutánea y visceral, situación de elevada
mortalidad (50%). La infección en el niño, si ocurre en las primeras 20 semanas del embarazo,
puede provocar aborto en 25%, malformaciones cerebrales, cicatrices, corioretinitis, RCIU. Si
ocurre después de las 20 semanas, puede causar parto prematuro, RCIU, o herpes neonatal.
Esta condición clínica tiene tres formas de presentación, las dos primeras de elevada
mortalidad y secuelas: herpes diseminado y encefalitis herpética o infección localizada en piel,
ojo y boca.
Pass R, Weber T, Whitley RJ. Herpesvirus infections in pregnancy.
Recommendations from the International Herpes Management Forum. Management
Strategies Workshop and 7th Annual Meeting.
Whitley R J. Varicella - Zoster virus. Mandell, Douglas and Bennett's Principles and
Practice of Infectious Diseases. Mandell G, Bennett J, Dolin R, editors. Fifth edition,
2000 Churchill Livingstone, Philadelphia, pp: 1586-98.
Abarca K, Cohen B J, Vial P. Seroprevalence of parvovirus B19 in urban Chilean children and
young adults, 1990 and 1996. Epidemiol Infect 2002; 128: 59-62.
20.- Femenino de 20 años, atendida en sala de urgencias ginecoobstétricas, Antecedente:
cursa embarazo de 38 SDG. Exploración Física: en trabajo de parto. Repentinamente presenta
sangrado profuso transvaginal y dolor abdominal. A la paciente se le coloca un monitor fetal
externo. El tono uterino está incrementado y hay desaceleraciones variables y ocasionales de
la FCF que llegan a 90.
El manejo apropiado en esta paciente es:
a) Tocolisis con un agonista beta adrenérgico
b) Continuar el monitoreo de la madre y el producto
c) Amniotomía
d) Cesárea
El manejo de éstas pacientes es estricto y urgente:
1- Ingreso inmediato de la paciente en vigilancia a sala de partos.
2- Venopuntura. Si existen signos de shock, instaurar preferentemente dos vías, una central y
otra periférica.
3- Solución de Ringer.
4- Monitoréo de signos vitales cada 15’.
5- Sonda vesical y monitoréo de diuresis. Diuresis de 50 ml/h asegura perfusión periférica
correcta.
Evitar diuresis inferiores a 30 ml/h.
6- Monitoréo contínuo de LCF.
7- Laboratorio de urgencia: hemograma, ionograma, urea y creatinina, coagulograma.
8- Sangre como mínimo 4U para reserva (concentrado de glóbulos rojos o sangre entera).
9- Ecografía obstétrica para localizar y medir el hematoma y descartar otras causas de
sangrado.
Dependiendo del grado de desprendimiento realizar:
1- Desprendimiento leve: sin compromiso materno ni fetal y el embarazo es de pretérmino,
conducta expectante. Uteroinhibición y maduración pulmonar fetal.
2- Desprendimientos moderados a severos: si hay compromiso materno o fetal u óbito fetal,
terminar inmediatamente la gestación.
3- Independientemente de la vía del parto, es preceptiva la práctica de la amniotomía, para
reducir la presión intraamniótica y disminuir el paso de tromboplastina hística a la circulación y
la extravasación de sangre al miometrio.
Las principales complicaciones son:
1- Hipotensión, shock hipovolémico.
2- Coagulación intravascular diseminada.
3- Necrosis cortical y necrosis tubular aguda renal.
Obstetricia. Scwarcz, Sala, Duverges. 7ª edic. Edit. El Ateneo. (Biblioteca Fac. Med.
UNNE).
21.- Mujer de 27 años a quien después de estudios de laboratorio y exploración física se le
diagnostica hirsutismo asociado a ovario poliquístico, el siguiente fármaco que usted elige para
su tratamiento por ser el más adecuado es:
a) Clomifeno
b) Estrógenos
c) Corticoide
d) Acetato de ciproterona
El acetato de ciproterona parece ser más efectivo que otros fármacos para el hirsutismo en
mujeres causado por la producción ovárica excesiva de andrógenos
Una de las causas de hirsutismo (crecimiento piloso excesivo) en mujeres es la
hiperproducción de andrógenos a partir del ovario. Varios fármacos pueden utilizarse para
contrarrestar los efectos del andrógeno. El acetato de ciproterona es un fármaco
antiandrogénico. Los efectos adversos informados con su uso fueron aumento de peso,
depresión, fatiga, síntomas mamarios y disfunción sexual. La revisión de los ensayos encontró
que el acetato de ciproterona parece ejercer un efecto en el hirsutismo similar a otros fármacos
utilizados para el tratamiento del hirsutismo por exceso de andrógenos. No existen pruebas
suficientes para comparar los efectos adversos de las opciones de tratamiento.
Van der Spuy ZM, le Roux PA. Acetato de ciproterona para el hirsutismo (Revisión Cochrane
traducida). En: La Biblioteca Cochrane Plus, número 4, 2007. Oxford, Update Software Ltd.
Disponible en: http://www.update-software.com. (Traducida de The Cochrane Library, 2007
Issue 4. Chichester, UK: John Wiley & Sons, Ltd.).
22.- Mujer de 36 años nulípara tras 3 años de relaciones sexuales sin contracepción, desde
hace 1 año inició presentando dismenorrea, dispareunia y sangrado vaginal intermenstrual. El
diagnóstico más probable de ésta paciente es:
a) Insuficiencia luteínica.
b) Enfermedad inflamatoria pélvica.
c) Dismenorrea funcional.
d) Endometriosis.
La endometriosis consiste en la aparición y crecimiento de tejido endometrial fuera del útero,
sobre todo en la cavidad pélvica como en los ovarios, detrás del útero, en los ligamentos
uterinos, en la vejiga urinaria o en el intestino. Es menos frecuente que la endometriosis
aparezca fuera del abdomen como en los pulmones o en otras partes del cuerpo.
La endometriosis es una enfermedad relativamente frecuente, que puede afectar a cualquier
mujer en edad fértil, desde la menarquia hasta la menopausia, aunque algunas veces, la
endometriosis puede durar hasta después de la menopausia. La endometriosis altera la calidad
de vida de las mujeres que la padecen, afectando a sus relaciones de pareja, familiares,
laborales y de reproducción.
Síntomas
Los síntomas clásicos son la dismenorrea, dolor pélvico, dispareunia, sangrados
intermestruales y en muchos casos, esterilidad.
El dolor no tiene que ver con el tamaño y la severidad de la lesión; generalmente cuanto
menor es la lesión mayor dolor produce. El dolor se agrava con las menstruaciones y en los
casos en que la lesión ocupa el fondo de saco de Douglas, puede dar dispareunia. Existe un
aumento de la PGF2 alfa y PGE2 y un aumento de las contracciones uterinas que podría
deberse a un depósito de endometrio en la cavidad peritoneal.
La esterilidad debido a la endometriosis podría deberse a distintas causas de acuerdo a la
severidad de la patología. En los casos de endometriosis severa puede haber un factor
tuboperitoneal con adherencias y alteración en la anatomía de la pelvis que interfiera con el
transporte del esperma y el óvulo. En los casos de endometriosis leve hay varios mecanismos
propuestos que justifican su relación con la infertilidad: foliculogénesis alterada, fase lútea
inadecuada, fagocitosis espermática, mala calidad ovocitaria, embriotoxicidad y alteración a
nivel de la implantación.. La producción de prostaglandinas por el endometrio ectópico puede
afectar la motilidad tubaria, la foliculogénesis y la función del cuerpo lúteo. Puede haber un
aumento de la activación de los macrófagos peritoneales en la endometriosis que cause la
fagocitosis de los espermas o la secreción de citoquinas que pueden ser tóxicas para el
embrión. Según algunos investigadores habría un 60% de las mujeres con endometriosis que
presentan un síndrome de Folículo Luteinizado no roto (LUF) en el cual el folículo no se rompe
en la ovulación y el óvulo queda atrapado.
Referencias bibliográficas
1. Ruiz V. Endometriosis y fertilidad. Ed. Acosta y Warman, pp. 99
2. Lópes,VH. Palomo E. Incidencias de endometriosis en una población infértil. XXI Congreso
nacional de Ginecología y Obtetricia. Guatemala, 1993.
3. El-Eoley, et al. Danazol but not ginadotropin releasing hormone agonists suppresses
autoantibodies in endomeriosis. Fertil Steril 1990; 54:725
4. Acosta AA. Buttram VC Jr. Besch PK, Malinak LR, Van Der Heyden J. A.proposed
classfication of pelvic endometriosis. Obstet Gynecol 1973;42:19.
5. Buttran VC Jr. Evolution of the revised American Fertility classification of endometriosis. Fert.
Steril 1985; 43: 347
6. López VH. Tratamiento médico-quirúrgico de la endometriosis. Simposio El rostro cambiante
de la endometriosis panamá 3. 12. 1993.
7. Steinleitner A. Heterolous transplation of activated murine peritonel macrophages inhibitis
gamete interaction in vivo; A paradigm fo endometriosis associted subfertility. Fertil Steril 1990;
54:725.
8. Damewood M. Effect of serum from patients with minimal to mild endometriosis on mouse
embryo growth. Fertil Steril 1990; 54: 917
9. Proug S. Peritoneal fluid fracctions from patients with endometriosis do not promote two-cell
mouse embryo growth. Fertil Steril 1990; 54: 927.
23.-Se trata de femenino de 37 años de edad, a quien se realiza diagnóstico de placenta
percreta, el tratamiento e elección en esta patología es:
a) Histerectomía.
b) Ergonovina a dosis altas.
c) Hemostasia con puntos transfictivos.
d) Taponamiento uterino.
La placenta anormalmente adherida es poco común y tiene importancia clínica por su
morbimortalidad, a consecuencia de hemorragia, perforación, invasión y lesión de las vías
urinarias. Esta adherencia anormal está asociada con la implantación placentaria sobre
cicatrices de cesárea previa, incisiones uterinas o legrados. La placenta percreta consiste en la
penetración del tejido placetario a través de toda la pared uterina, traspasando la serosa de la
misma. La identificación de esta anormalidad antes del parto es posible mediante métodos de
imagen (escala de grises por ultrasonido, ecografía Doppler color pulsado o resonancia
magnética nuclear). El tratamiento conservador se acompaña de elevada morbilidad en
muchos casos, por lo que el tratamiento quirúrgico se convierte en el definitivo. La
literatura sugiere un aumento previsto en la incidencia de esta condición con base en el
incremento del número de cesáreas, por lo que la histerectomía postcesárea será una decisión
que enfrentarán los especialistas con mayor frecuencia.
Perucca E, Domínguez C, Yahng Ch, García R. Placenta previa percreta con invasión vesical.
Rev Chil Obstet Ginecol 1997; 62(3): 206-10.
2. Abbas F, Talati J, Wasti S et al. Placenta percreta with bladder invasion as a cause of life
threatening hemorrhage. J Urol 2000; 164: 1270-4.
3. Perucca E, Cazenave H, Barra A, Ochoa N, Villagrán G, Espinoza R, Estay R, Bustamante
R, Siebert A. Placenta previa percreta con invasión vesical. Rev Chil Obstet Ginecol 2002;
67(5): 364-7.
4. Price F, Resnik E, Heller K, Christopherson W. Placenta previa percreta involving de urinary
bladder. A report of two cases and review of the literature. Obstet Gynecol 1991; 78(3): 508-11.
24.- Se trata de paciente femenino de 37 años de edad, G-3, C-2. Es ingresada a hospital
presentando cefalea, acúfenos, fosfenos y epigastralgia en barra con embarazo de 34
semanas. E.F T/A 160/110, FC 84 x´, FR 18 x´, no presenta fiebre, somnolienta, sin agregados
cardioventilatorios, hepatalgia. F.U. de 25 cm. Producto único vivo. FCF 110 lpm, genitales sin
pérdidas ni modificaciones cervicales. Laboratorio: hb 9.8 g/dl, plaquetas de 54 mil, TP 11 seg
TPT 27, TGO 160 ng/dl TGP 160 ng/dl, hiperbilirrubinemia indirecta, albuminuria 300 mg/dl,
Ácido Úrico de 8.1 mg/dl, creatinina de 1.5 mg/dl.
El diagnóstico más probable es:
a) Sindrome de hellp.
b) Sx anticuerpos antifisfolípidos
c) Púrpura trombocitopénica trombótica
d) Hígado graso
DEFINICIÓN:
• Es una complicación de la preeclampsia en la cual además de la Hipertensión Arterial y
proteinuria hay presencia de anemia hemolítica, enzimas hepáticas elevadas y recuento bajo
de plaquetas
MANIFESTACIONES CLINICAS:
• Malestar general, fatiga y molestias inespecíficas 90%
• Cefalea 70%
• Epigastralgia 64%
• Vómito 22%
• Fosfenos 15%
• Visión Borrosa 11%
• Acùfenos 3%
• Ictericia
• Anemia no explicada
• Oliguria
BIBLIOGRAFIA:
Sibai baha, El síndrome HELLP. Universidad de Valencia , revista quincenal de Obstetricia
clínica y ginecología, Octubre 2003.
V. Cararach, Síndrome de HELLP y Repercusiones maternas. X curso intensivo de formación
continuada materno fetal. Enero de 2003.
Toirac, Abelardo. Síndrome de Weistein HELLP Hospital Ginecoobstetrico Tamara Bunke.
Junio 2002
De la Fuente, David. Síndrome HELLP. Medicina Universitária 2003; 5 (19): 101 -9
Andrea G. Witlin, DO, Baha M. Sibai, MD. Diagnosis and Management of women with
Hemolysis Elevate Liver Enzymes, and Pletelet Count (HELLP) syndrome. Hospital Physician.
Febrero 1999.
CIFUENTES B, Rodrigo. Ginecología y obstetricia.
25.- Se trata de femenino de 24 años, G-1, que cursa con embarazo de 37 semanas de
gestación, presenta pérdida del estado de alerta posterior a crisis convulsivas tónico-clónicas,
signos vitales con T-A 170.120mmhg Fc 95x´, reflejos osteotendinosos aumentados, se aprecia
una Fc fetal de 132x´ y edema importante de miembros inferiores, no se aprecian datos de
trabajo de parto ni modificaciones cervicales, El diagnóstico más probable es:
a) Pre eclampsia severa
b) Crisis epileptica de gran mal
c) Hipertensión inducida por el embarazo
d) Eclampsia
CUADRO 1. DIAGNÓSTICO*
Preeclampsia Leve: Se presenta después de la semana 20 de gestación, durante el parto, o
en las primeras 6 semanas después de éste
Presión sistólica ≥ a 140 mm Hg o presión diastólica ≥ 90 mm Hg
Proteinuria ≥ a 300 mg / orina de 24 hrs o su equivalente en tira reactiva
Preeclampsia Severa: Se presenta después de la semana 20 de gestación, durante el parto, o
en las primeras 6 semanas después de éste
Presión sistólica ≥ a 160 mm Hg o presión diastólica ≥ 110 mm Hg
Proteinuria ≥ a 2 gr en orina de 24 horas o su equivalente en tira reactiva
Creatinina sérica > a 1.2 mg/dl
Trombocitopenia ≤ 150 000 cel/mm3
Incremento de la deshidrogenasa láctica ≥ a 600 UI
Elevación al doble de los valores de TGO/AST o TGP/ALT
Cefalea, alteraciones visuales o auditivas
Epigastralgia
Oliguria ≤ a 500 ml en 24 horas 7
Edema agudo de pulmón
Dolor en hipocondrio derecho
Restricción en el crecimiento intrauterino
Oligohidramnios
Eclampsia Preeclampsia mas convulsiones sin otra causa. Se presenta después de la semana
20 de gestación, durante el parto, o en las primeras 6 semanas después de éste.
Síndrome de HELLP Criterios para establecer el diagnóstico del síndrome de HELLP:
Plaquetas < 100 000/mm3 TGO/AST ≥ 70U/L DHL ≥ 600U/LBilirrubina total > 1.2 mg/dl
Se presenta después de la semana 20 de gestación, durante el parto, o en las primeras 6
semanas después de éste.
Hipertensión Crónica: Se diagnostica cuando existe hipertensión arterial ≥ a 140/90 mm Hg
antes de la semana 20 de gestación o si persiste después de doce semanas posteriores al
parto.
Las pacientes con hipertensión crónica deben ser evaluadas antes del embarazo para
determinar la severidad de la hipertensión y facilitar la planeación de un embarazo mediante el
cambio de medicamentos y de hábitos higiénicos y dietéticos para evitar complicaciones.
Hipertensión Gestacional: Presencia de hipertensión arterial ≥ a 140/90 mm Hg después de
la semana 20 de gestación y se mantiene hasta las doce semanas después del parto
Ausencia de proteinuria
Presencia o no de cefalea, acúfenos y fosfenos
Después de 12 semanas de la interrupción del embarazo se revalorará la presencia de
hipertensión, si continúa, se reclasifica como hipertensión crónica: es un diagnóstico
retrospectivo. 8 Si no hay, se clasifica como hipertensión transitoria.
1. Aagaard-Tillery KM, Belfort MA. Eclampsia: morbidity, mortality, and management. Clin
Obstet Gynecol 48:12-23, 2005.
2. Atallah AN, Hofmeyr GJ, Duley L. Calcium supplementation during pregnancy for preventing
hypertensive disorders and related problems. Cochrane Database Syst Rev 1:CD001059, 2001.
3. Barton JR, Sibai BM. Diagnosis and management of hemolysis, elevated liver enzymes, and
low platelets syndrome. Clin Perinatol 31:807-33, 2004.
4. Baxter JK, Weinstein L. HELLP syndrome: the state of the art. Obstet Gynecol Surv 59:838-
45, 2004.
5. Cetin A. Eclampsia. In Mohler III ER, Townsend RR. Advanced therapy in hypertension and
vascular disease. Ontario: B.C. Decker Inc. pp. 407-15, 2006.
6. Cetin A. Hemolysis, elevated liver enzymes, and low platelets (HELLP). In Mohler III ER,
Townsend RR. Advanced therapy in hypertension and vascular disease. Ontario: B.C. Decker
Inc. pp. 416-20, 2006.
7. Chappell LC, Seed PT, Briley AL, Kelly FJ, Lee R, Hunt BJ, Parmar K, Bewley SJ, Shennan
AH, Steer PJ, Poston L. Effect of antioxidants on the occurrence of pre-eclampsia in women at
increased risk: a randomised trial. Lancet 354:810-16, 1999.
26.- Femenino que cursa con 12 semanas de gestación, refiere tres días con sangrando por
genitales, con náuseas continuas y constantes, útero mayor aumentado de tamaño y valores
de beta HCG muy elevados, debemos sospechar de:
a) Amenaza de aborto.
b) Aborto diferido.
c) Mola hidatídica.
d) Amenaza de aborto en un útero con miomas.
La enfermedad trofoblástica gestacional agrupa a diferentes entidades interrelacionadas: mola
completa, generalmente diploide con origen cromosómico paterno, mola parcial generalmente
triploide, tumor trofoblástico del lecho placentario y coriocarcinoma, con tendencias variables a
la invasión local y a las metástasis, cuyo denominador común es la hipersecreción de hCG. El
coriocarcinoma es diploide y proviene de ambos progenitores, excluyendo probablemente su
origen directo en la mola completa. El tumor trofoblástico del lecho placentario está constituido
por trofoblasto mononuclear intermedio no conteniendo vellosidades coriónicas e
inmunohistoquimicamente caracterizado por expresar muchas de sus células hPL y unas pocas
hCG.
Cuadro clínico
Tras un periodo de amenorrea, y a partir del segundo mes, hay un aumento de los síntomas
subjetivos del embarazo, sobre todo náuseas y vómitos o hiperémesis en un 30%. Hay
metrorragias irregulares en el 96% de los casos, en principio de escasa cantidad, pero que se
va incrementando. Dicha hemorragia no es continua sino que se repite cada dos o tres días, de
color roja o negruzca, que se produce por la ruptura de vasos maternos al separarse las
vesículas de la decidua.
Ocasionalmente, hay expulsión de restos molares, que lo refiere la paciente como expulsión de
vesículas en el 11 % de los casos y que es patognomónico pero aparece tardíamente. También
puede presentar mal estado general, dolor difuso en hipogastrio y anemia. Esta última en
relación con las metrorragias.
Pueden haber signos y síntomas de hipertiroidismo, como taquicardia, sudoración, y temblores,
en el 7%, y es debido a que la fracción β-hCG es similar a la hormona TSH. Existen signos de
preeclampsia o hipertensión gestacional del primer trimestre hasta en el 50% de los casos, y
signos de insuficiencia respiratoria aguda en el 2% por embolismo pulmonar de las células
trofoblásticas, o por la asociación entre hipertiroidismo e hipertensión arterial. Como
complicaciones pueden aparecer coagulopatías y metástasis.
A la exploración el útero esta aumentado de tamaño, en el 50% de los casos, por encima de lo
que correspondería a la edad gestacional,3 de consistencia blanda, sin signos de actividad fetal
a partir de la semana 12 y siempre que se trate de una mola total. También puede que la
paciente presente un tamaño uterino menor al esperado para la edad gestacional.3 El cuello
está cerrado, con metrorragia en cantidad variable, y raramente se observa la expulsión de
vesículas. Pueden haber quistes teca-luteínicos bilaterales en ovario en cerca del 20% de los
casos,3 debido al estímulo de la β-hCG.
La enfermedad trofoblástica maligna va a cursar con metrorragias por lo general intensas, hay
una elevación de la β-hCG y en la ecografía se objetiva la cavidad uterina con signos de
ocupación atípica.
Diagnóstico
Por la clínica, y pruebas complementarias como la determinación de la β-hCG y la ecografía.
La determinación de la β-hCG se basa en que el trofoblasto produce la hormona gonadotropina
coriónica, presentando cifras elevadas, y su cuantificación va a servir para diagnóstico, valorar
el pronóstico, y el seguimiento postratamiento. La ecografía revela un útero aumentado de
tamaño que no corresponde con la amenorrea, con ecos en su interior, puntiformes que
corresponderían a las vesículas y que asemejan copos de nieve o panal de abeja. No se
aprecia saco gestacional ni estructuras fetales y, en ambos ovarios se aprecian quistes teca-
luteínicos como formaciones ováricas redondas, econegativas, con múltiples tabiques en su
interior
BIBLIOGRAFÍA
1. Mazur MT, Kurman RJ. Gestational trophoblastic disease and related lesions. En:
Kurman RJ editor. Blaunstein’s pathology of the female genital tract. 4th ed. New York:
Springer-Verlag. 1994, p. 1049-93.
2. Kurman RJ, Young RH, Norris HJ, Main CS, Lawrence WD, Scully RE.
Immunocytochemical localization of placental lactogen and chorionic gonadotrophin in
the normal placenta and trophoblastic tumors, with emphasis on intermediate
trophoblast and the placental site trophoblastic tumor. Int J Gynecol Pathol 1984; 3:
101-21.
3. Berkowitz RS, Golstein DP. The management of molar pregnancy and gestational
trophoblastic tumours. En Knapp RC, Berkowitz RS, editores. Gynecologic Oncology,
2nd ed. New York: Mc Graw-Hill 1992, p. 328-38.
4. De Agustín P, Ruiz A, López F, Contreras F. Patología de la enfermedad trofoblástica.
Simposio Enfermedad Trofoblástica 1972; 79-98.
5. Salem S. Ultrasound diagnosis of trophoblastic disease. En: Sanders RC, James AE(Jr)
editores. Ultrasonography in Obstetrics and Gynaecology. New York: Appleton-Century
Crofts: 1977; p. 255-66.
6. Silverberg SG, Kurman RJ. Tumors of the uterine corpus and gestational trophoblastic
disease. En: Rosai J, Sobin LJ, editores. Atlas of tumor pathology: tumors of the uterine
corpus and gestational trophoblastic disease, fasc. 3, ser. 3. Washington DC: Armed
Forces Institute of Pathology; 1992, p. 219-85.
Szulman AE, Surti U The syndromes of hydatiform mole II. Morphologic evolution of the
complete and partial mole. Am J Obstet Gynecol. 1978; 132: 20-7.
27.- Mujer que acude al servicio de ginecología por referir ciclos opso-menorreicos desde el
inicio de su menarquia, ha incrementado 15 Kg. de lo que pesaba habitualmente, se aprecia
una gran cantidad de acné, pero además refiere depilarse el área del bigote cada semana, y
cree que esto le sensibiliza la piel para que aumente el acné.
En el caso de ovario poliquístico el dato clínico que con más frecuencia les acompaña es:
a) anovulación y esterilidad
b) Hirsutismo
c) Amenorrea
d) Obesidad
El síndrome de ovarios poliquísticos (SOPQ) afecta aproximadamente a un 4% de mujeres en
edad reproductiva y se caracteriza por anovulación crónica e hiperandrogenismo. Es la causa
más común de infertilidad en mujeres.
Se caracteriza clínicamente por acné, alopecia, hirsutismo, irregularidades menstruales e
infertilidad.
Los hallazgos de laboratorio más frecuentes son: aumento de la hormona luteinizante (LH),
aumento de la relación LH/FSH (hormona folículoestimulante), aumento de andrógenos (tanto
ováricos como adrenales) y de estrógenos circulantes. Otros hallazgos de laboratorio
habituales son una prueba tolerancia oral a la glucosa anormal y alteraciones en el perfil
lipídico.
Todo esto junto con las imágenes ecocardiográficas características define al síndrome.
La terapéutica permite dos grandes enfoques que pueden superponerse: la corrección de las
manifestaciones de hiperandrogenismo y el tratamiento de las alteraciones del eje reproductivo
(anovulación, esterilidad). Los antiandrógenos están fundamentalmente indicados para tratar
los síntomas virilizantes.
Las alternativas para inducir la ovulación son numerosas: al citrato de clomifeno y a la antigua
resección en cuña se agregan las gonadotrofinas humanas, pulsos de GnRH (hormona
liberadora de gonadotrofinas), medidas o fármacos para modificar los niveles de insulina, y
finalmente técnicas quirúrgicas endoscópicas para reducir la masa ovárica.
BIBLIOGRAFIA
1. Guzick D.Polycystic ovary syndrome: Symptomatology, pathophysiology, and
epidemiology. Am J Ostetric Gynecol 1998; 179 (6): 89-93.
2. Stephen Franks. Polycystic ovary syndrome. N Engl J Med 1995; 333(13): 853-861.
3. Gori J.R., Larusso A. Ginecología de Gori. 2ª Edición. Buenos Aires, Argentina.
Editorial El Ateneo. 2001.
4. Adams J., Polson D. W., Franks S. Prevalence of polycystic ovaries in women with
anovulation and idiopathic hirsutism. Br Med J 1986; 293: 355-9.
5. Copeland L. J . Ginecología. Buenos Aires, Argentina. Editorial Panamericana. 1ª
Edición. 1994.
6. Ehrmann D.A., Rosenfield R.L., Barnes R.B., Brigell D.F., Sheikh Z. Detection of
functional ovarian hyperandrogenism in women with androgen excess. N Engl J Med
1992; 327:157-162.
7. Kahasar-Miller M., Conway Myers B., Boots L., Azziz R. Steroidogenic acute regulatory
protein (StAR) in the ovaries of healthy women and those with polycystic ovary
syndrome. Am J Obstet Gynecol 2001; 185(6): 1381-7.
8. Pérez Sánchez A. Ginecología. Santiago de Chile. Publicaciones Técnicas
Mediterráneo. 3ª Edición. 1995.
9. Velázquez E., Mendoza S., Hamer T., Sosa F., Glucck C. Metformin therapy in women
with polycistic ovary syndrome reduces hiperinsulinemia, insulin resistence,
hyperandrogenemia, and systolic blood pressure, while facilitating menstrual regularity
and pregnancy. Metabolism 1994 ; 43: 647-655.
28.- Femenino de 29 años. Acude al servicio de urgencias por presentar salida de líquido
vaginal. Antecedentes: G2, P1, cursa embarazo de 36 semanas de gestación exploración
física: cervix con 10% de borramiento, 1 cm de dilatación y Tarnier positivo.
La complicación más frecuente en esta paciente es:
a) Corioamnioitis.
b) Parto pre término.
c) Sepsis neonatal.
d) Endometritis.
Corioamnioitis: El diagnóstico de la infección intraamniótica (IIA) es básicamente clínico. La
corioamnionitis se debe descartar en toda gestante que presente fiebre sin foco aparente,
sobre todo si se sospecha o se ha confirmado una rotura de membranas. Los criterios más
empleados para el diagnóstico son: fiebre materna y, al menos, 2 de los siguientes signos:
taquicardia materna, taquicardia fetal, irritabilidad uterina, leucocitosis materna o líquido
amniótico purulento o maloliente.
Progresos de obstetricia y ginecología: revista oficial de la Sociedad española de ginecología y
obstetricia, ISSN 0304-5013, Vol. 48, Nº. 6, 2005 , pags. 316-317
29.- Mujer de 23 años diagnosticada de E. Ectópico a nivel ampular, con saco gestacional
menor de 3 cm., sin actividad cardíaca embrionaria, asintomática y con niveles de B-HCG
menores para su edad gestacional. El tratamiento más indicado es:
a) Salpingocentesis
b) Resección segmentaria
c) Tratamiento médico con Metrotexate.
d) Histerectomía total con doble anexectomía.
TRATAMIENTO
 METOTREXATE
 Ácido 4 amino 10 metil folínico, antagonista del ácido fólico
 Glucosa hiperosmolar
 Prostaglandina F2a
 Actinomicina D
 Mifespristona
TRATAMIENTO CON METOTREXATO
 Mayor éxito:
 Embarazo menor a 6 SDG
 Masa tubaria menor de 3.5cm
 Feto sin latido cardíaco
PROTOCOLO CON DOSIS ÚNICA DE METOTREXATE
Día 0 hGC, Biometría hemática, transaminasas, creatinina, grupo sanguíneo
Día 1 hGC
Día 4 hGC
Día 7 hGC
 Disminución de hCG <15%. Segunda dosis
 Si la hCG declina seguirla semanalmente
 Si la hCG está en meseta o en ascenso, segunda dosis
 El raspado endometrial se realiza sólo en pacientes con hCG <2000 mIU/mL al
momento de comenzar el tratamiento
Graczykowski JW, Mishell DR. Methotrexate prophylaxis for persistent ectopic pregnancy after
conservative treatment by salpingostomy. Obstet Gynecol.
30.- Femenino de 19 años acude a consulta por padecer un cuadro de dolor abdominal de
inicio súbito, refiere alteraciones en su ciclo menstrual. La exploración física revela una
tumoración dolorosa en el anexo izquierdo. La prueba de embarazo es negativa. La radiografía
muestra una masa opaca en la fosa ilíaca izquierda con áreas de calcificación. El diagnóstico
clínico más probable es:
a) Cistadenoma mucinoso
b) Teratoma quístico
c) Quiste folicular
d) Cistadenoma seroso
Los tumores de células germinales constituyen casi el 20% de los tumores de ovario y de ellos
un 95% son benignos, siendo el tipo más frecuente el teratoma maduro benigno o también
denominado quiste dermoide (1).
Aproximadamente el 80% se presentan en mujeres en edad fértil. Se originan a partir de
células embrionarias pluripotenciales presentes habitualmente en ovario, testículo, mediastino,
retroperitoneo y región sacrocoxígea, esto explica que la coloración de las faneras del quiste
coincida con el fenotipo del paciente (2).
El teratoma quístico benigno con relativa frecuencia es un tumor bilateral (del 7 al 25% según
los autores) (1) y se caracteriza por una cápsula gruesa, bien formada, revestida por epitelio
plano estratificado. Bajo este se pueden encontrar una variedad de apéndices cutáneos que
incluyen glándulas sudoríparas, apocrinas y sebáceas. La cavidad se llena de los detritus de
éste y sus anexos, que es de color amarillo pálido, grasoso, espeso y suele contener pelo.
Otros tejidos que se pueden encontrar son dientes, cartílago, plexos coroideos, falanges, tejido
nervioso y en ocasiones tejido tiroideo (struma ovarii) con potencial tirotóxico o de
degeneración maligna tiroidea.
La mayor parte de los quistes dermoides son asintomáticos y la forma más frecuente de
presentación son el dolor abdominal (48%) y hemorragia uterina anormal o concomitante (15%)
o aumento del volumen abdominal (15%). La rotura de un quiste dermoide es rara, entre el 1-
1,2% y constituye una urgencia quirúrgica (3).
El tratamiento es quirúrgico, siendo posible la mayor parte de las veces una resección del
mismo, respetando el resto del ovario.
La ecografía constituye el modelo diagnóstico de elección y la combinación de ecografía con
radiografía simple de abdomen proporciona un diagnóstico más exacto en la mayoría de los
casos, siendo la resonancia magnética o la TAC el que aporte el diagnóstico diferencial.
Las Rx de quiste dermoide se caracteriza por una cápsula bastante radioopaca y el líquido
oleoso que contiene es radiolúcido, esta conjugación presenta muchas veces una
característica de aspécto radiológico de calcificación en la pared del mismo.
Cólico nefrítico, teratoma ovárico y radiografía simple de aparato urinario Romero Pérez P,
Martínez Hernández MªC.
Servicio de Urología. Policlínico San Carlos Denia (Alicante).Actas Urol Esp. 2007:31(8):936-
937ACTAS UROLÓGICAS ESPAÑOLAS SEPTIEMBRE 2007
31.- Se trata de femenino de 33 años que acude al servicio cursa en éste momento con
diagnóstico de preclampsia , el fármaco de elección que se administra en esta patología es:
a) Nifedipina.
b) Inhibidores de la enzima convertidora de angiotensina.
c) Clonidinas.
d) Alfametildopa.
• Prevenir complicaciones a corto plazo de las mujeres con PA elevada que comprometa
el bienestar fetal
• Cuando la PAS es mayor o igual a 150 mmHg y la PAD mayor o igual a 100 mmHg.
 El propósito es alcanzar cifras de TA alrededor de 140/90.
 La medicación antihipertensiva se reserva para los casos en que la PAD ≥ 100 mmHg.
 Se recomienda continuar el tratamiento antihipertensivo previo al embarazo,
exceptuando el uso de IECA.
 La alfametildopa y la hidralazina vía oral son los fármacos de elección dado su uso
extensivo con seguridad y eficacia y sin efectos colaterales para el feto (excepto
hidralazina en lupus).
• ALFA METILDOPA
500-2000 MG/DÍA
• HIDRALAZINA
50-200 MG/DÍA
• LABETALOL
100-400 MG/DIA
• ATENOLOL
50-200 MG/DÍA
• NIFEDIPINA
10-30 MG/DÍA
1. Aagard K, Belfort M. Eclampsia: Morbility, mortality, and management. Clin
Obstet Gynecolol. 2005; 48: 12-23.
2. Oyarzún E. Síndrome hipertensivo del embarazo en Oyarzún E. Ed. Embarazo de alto
riesgo. Ediciones Universidad Católica de Chile. Santiago. 1997: 157-
175.
3. Roberts J, Redman C. Pre-eclamsia: More than pregnancy induced hypertens
32.- Se trata de femenino de 56 años de edad refiere aumento de volumen a nivel abdominal,
con predominio de hemiabdomen inferior con la siguiente sintomatología: plenitud,
estreñimiento, se acompaña de USG pélvico que demuestra imagen quística en ovario
derecho de 15 por 15 cms. El diagnóstico más probable es:
a) Teratoma quístico.
b) Disgerminoma.
c) Endometrioma.
d) Cistadenoma seroso.
Los Tumores de Ovario son una patología frecuente dentro del contexto de la patología
femenina. Por esta causa consultan un grupo elevado de mujeres, tanto las consultas de
ginecología como las de Cirugía propiamente dicha. Las edades oscilan desde las tempranas
hasta las ya avanzadas, siendo el riesgo de degeneración maligna muy variable y relacionado
con le edad. La experiencia de la clínica revela la alta incidencia de tumores de ovario en la
etapa del climaterio, comprendida entre los 35 y 65 años de edad 1.
El cistoadenoma seroso de ovario (CSO) es un tipo de tumor derivado del epitelio superficial
(celómico), formado por áreas quísticas. El cistoadenoma seroso de ovario es el tumor más
frecuente de aquellos que provienen del epitelio celómico superficial. Hay tumores pequeños
macroscópicamente y tumores masivos que ocupan toda la pelvis e incluso la cavidad
abdominal. Estas frecuentes neoplasias quísticas uniloculares están tapizadas por células
epiteliales altas, cilíndricas y ciliadas, llenas de un líquido seroso claro y de superficie lisa con
abundantes vasos. Las variedades benigna, limítrofe y maligna representan, en conjunto, 30%
aproximadamente de todos los tumores del ovario. El riesgo de presentar tumores epiteliales se
incrementa con el paso de la edad, ya que pese a que la declinación de la función ovárica
marca el envejecimiento gonadal progresivo, el ovario humano nunca pierde su capacidad para
generar tumores. Por lo general, cuando es detectado, su tamaño es grande, en donde la
imagenología puede ayudarnos a considerar su diagnóstico
1. Capítulo 22 Tumores Benignos de Ovario. En: Novak ER, Jones G., Jokes HW. Tratado
de Ginecología. 9 ed. Ciudad de la Habana. Editorial Científico Técnica; 1977.p.432 – 66.
2. MedlinePlus Enciclopedia Médica en Español: Quistes Ováricos. Disponible en:
http://vsearch.nlm.nih.gov/vivisimo/cgibin/querymeta?v%3Aproject=medlineplusspanish&spell=s
pell&query=Quistes+Ov%C3%A1ricos Acceso: Actualizado 20/6/06.
Capítulo XL Tumores Ováricos En: Llusiá Botella J, Núñez Clavero JA. Tratado de Ginecología.
Ciudad de la Habana. Editorial Científico Técnica. 1983; T 3.1; p. 751 – 803.
33.- Paciente femenino de 27 años de edad con deseo de un embarazo, antecedentes de
G3 A2 - P1 se le realiza una histerosalpingografía, se constata que existe un síndrome de
Asherman. Ello significa que se trata de:
a) Útero bicorne
b) Endometriosis en la trompa
c) Sinequias uterinas
d) Insuficiencia istmico cervical
El síndrome de Asherman es una enfermedad ginecológica rara que se caracteriza por la
presencia de sinequias (adherencias) intrauterinas que pueden ocasionar amenorrea (ausencia
de períodos menstruales regulares) e infertilidad.
En 1894 Heinrich Fritsch describe por primera vez la presencia de sinequias intrauterinas de
tipo postraumático, en una paciente que desarrolló una amenorrea secundaria a un curetaje.
Posteriormente en 1927 Bass informó de veinte casos de atresia (oclusión de una abertura
natural) cervical tras abortos inducidos, pero no fue hasta 1948, cuando Joseph G. Asherman
recopiló la información hasta entonces existente y acuñó el nombre con el que se conoce
actualmente a la enfermedad.
Asherman describió originalmente dos tipos diferentes de amenorrea secundaria, en función de
su etiología (estudio de las causas de las enfermedades): la amenorrea traumática atrética,
debida a estenosis del orificio cervical interno y la amenorrea debida a adherencias
intrauterinas. Posteriormente ambas entidades se agruparon en una única entidad bajo el
nombre de síndrome de Asherman.
Suele presentarse en mayor proporción tras dilataciones y curetajes uterinos de repetición y
sobre todo si se realizan durante el embarazo o si existe infección uterina en el momento en el
que se realizan estas intervenciones.
Las adherencias intrauterinas pueden producirse debido a cualquier factor que lleve a una
destrucción de las paredes del miometrio (capa muscular de la pared del útero). Sin embargo,
hay que distinguir entre factores predisponentes, siendo el principal de ellos el embarazo y
factores causales, entre los que se encuentran: traumatismos uterinos, intervenciones
quirúrgicas que afecten al útero, agentes físicos o químicos e infecciones uterinas por
tuberculosis o esquistosomiasis. En cualquier caso, el factor más importante es el trauma
uterino en el momento del parto o el puerperio.
El cuadro clínico es muy variable y las manifestaciones clínicas varían con el grado de oclusión
de la cavidad uterina y la severidad de las adherencias, pudiendo presentarse: esterilidad
cuando la oclusión de la cavidad uterina incluye porciones proximales (más cerca de un centro,
tronco o línea media) de las trompas de Falopio o cuando las adherencias impiden la nidación
del huevo; las pacientes presentan con frecuencia amenorrea, oligomenorrea (disminución de
la frecuencia de las menstruaciones), dismenorrea (menstruación dolorosa) y abortos
repetidos.
1. Hysteroscopic treatment of severe Asherman's syndrome and subsequent fertility. Capella-
Allouc S; Hum Reprod, 1999 May.
34.- Se trata de femenino de 31 años de edad, la cual inicia con hiperemesis gravídica de
difícil control, así como sangrado trasvaginal. Se realiza el diagnóstico de mola hidatiforme. La
primera opción de tratamiento indicado en esta patología es:
a) Histerectomía total abdominal.
b) Legrado por aspiración.
c) Metotrexate y seguimientos radiográficos.
d) Legrado uterino instrumental
La enfermedad trofoblástica gestacional agrupa a diferentes entidades interrelacionadas: mola
completa, generalmente diploide con origen cromosómico paterno, mola parcial generalmente
triploide, tumor trofoblástico del lecho placentario y coriocarcinoma, con tendencias variables a
la invasión local y a las metástasis, cuyo denominador común es la hipersecreción de hCG. El
coriocarcinoma es diploide y proviene de ambos progenitores, excluyendo probablemente su
origen directo en la mola completa. El tumor trofoblástico del lecho placentario está constituido
por trofoblasto mononuclear intermedio no conteniendo vellosidades coriónicas e
inmunohistoquimicamente caracterizado por expresar muchas de sus células hPL y unas pocas
hCG
Tratamiento
Hay que tratar las complicaciones como la hiperémesis, anemia, hipertensión, y alteraciones
electrolíticas, coagulopatías, alteraciones cardio-respiratorias y preeclampsia, procediendo a
evacuar la mola lo antes posible, con lo que se producirán menos malignizaciones.
La evacuación del contenido uterino se realiza mediante dilatación, y legrado por aspiración.
Además se pauta profilaxis antibiótica y oxitócicos.3
La histerectomía, con la mola en su interior, está indicada en pacientes de edad superior a 40
años o en mujeres con más de tres hijos, ya que en ambos grupos se ha demostrado una
mayor incidencia de malignización.
. Tras la cirugía, se mide la concentración de gonadotropina coriónica humana para determinar
si la extirpación ha sido completa. Si es así, el valor de esta hormona vuelve a la normalidad,
en unas 8 semanas, y se mantiene en esos valores. Si una mujer a la que se le ha extirpado
una mola queda embarazada, es difícil interpretar un valor alto de gonadotropina coriónica
humana, porque podría estar causado tanto por el embarazo como por una parte de la mola
que no se ha extirpado. En consecuencia, a las mujeres a las que se les ha extirpado una mola
se les recomienda no quedar embarazadas durante un año. Las molas hidatiformes benignas
no necesitan quimioterapia, pero las malignas sí. Los fármacos que se usan para este
tratamiento son el metotrexato, la dactinomicina o una combinación de ambos.
Silverman L,Romero Zambrano F, Saldaño S. Enfermedad molar. Diagnóstico, tratamiento y
seguimiento, 1987.
4- Puertas A, López Fernandez J et al. Enfermedad trofoblástica. Casuística del Hospital Virgen
de las Nieves de
Granada. Clín Invest Gin Obs 1993; 20: 98-103.
5- Enfermedad trofoblástica gestacional. Propuesta Normativa Perinatológica y Ginecológica de
Alto Riesgo.
Ministerio de Asuntos Sociales. Tucumán. 1996-1997; 19:171-183.
6- Jones. Enfermedad Trofoblástica Gestacional: qué hemos aprendido en la última década.
Am J Gynecol Obstet
1990;162: 1286-1292.
7- Resúmen del simposio Enfermedad Troblástica Gestacional del Segundo Congreso Nacional
de AGORA, 1990.
8- Gonzalez Merlo et al. Protocolos de diagnóstico y tratamiento en Obstetricia y Ginecología,
Barcelona: editorial
Salvat, 7:35-45.
35.- Se trata de femenino de 33 años que ingresa a la sala de urgencias, inicia con
convulsiones por presentar preclampsia, decide iniciar tratamiento, el fármaco elección en
ésta patología es:
a) Diacepam.
b) Fenitoína.
c) Donadores de óxido nítrico.
d) Sulfato de magnesio.
Manejo de la Preeclampsia
1. Manejo ambulatorio: HTA sin proteinuria significativa, se recomienda el reposo en cama.
Monitoreo de TA, peso, presencia de proteínas en orina. Ecografías periódicas para ver el feto
y evaluar posibles retardo de crecimiento.
2. Manejo hospitalario: para mujeres con HTA inducida por el embarazo y 2+ o más o
proteinuria significativa y en quienes falló el manejo ambulatorio.
3. Laboratorio y evaluación del peso: debe realizarse diariamente. Evaluación de la dinámica
fetal. Monitoreo de síntomas como cefalea, alteraciones visuales y dolor epigástrico.
4. El parto es el tratamiento de elección: el cual debe realizarse cuando el feto está maduro
pero puede realizarse en forma temprana si la salud de la madre está en peligro o si hay
evidencia de distress fetal. El parto está indicado cuando la paciente cumple con los criterios de
preeclampsia severa. Betametasona 12.5 mg IM dos veces por día puede estimular la
maduración de los pulmones fetales.
5. Terapia antihipertensiva: está indicada sólo si la TA es persistentemente > 160/110 , es
importante disminuir la TA hasta una diastólica de 90 a 100 mmHg porque la presión normal
podría resultar en hipoperfusión de la placenta. Los diuréticos nunca están indicados, estas
pacientes ya son hipovolémicas. Los IECA no deben ser usados durante el embarazo. Las
medicaciones de largo plazo, incluyen alfa metildopa, atenolol y labetalol.
6. Terapia anticonvulsivante:
A- Profilaxis de las convulsiones: está indicada en todas las pacientes pre-eclámpticas
durante el trabajo de parto y el parto y por un mínimo de 24 hs luego del mismo. Algunos
mantienen la terapia con magnesio hasta que comienza la diuresis. El Sulfato de Magnesio es
la droga de elección. La dosis profiláctica es de 4 a 6 g de sulfato de magnesio IV y continúa
con 2 g c/ hora.
B- Tratamiento de las convulsiones: Sulfato de Magnesio 1 g/min IV hasta controlar las
convulsiones hasta un máximo de 4 a 6 g. El nivel terapéutico es de 4 meq/l. Toxicidad del
magnesio: ausencia de reflejo patelar, debilidad muscular, parálisis respiratoria y depresión
cardíaca, 10 ml al 10 % de gluconato de calcio puede ser administrada IV. La terapia con
sulfato de magnesio continúa por lo menos 24 horas en el post parto, la terapia puede
detenerse si la excreción urinaria es > 200 ml/h por cuatro horas consecutivas.
C- Prevención: 81 mg de aspirina diarios pueden ser administrados luego del primer
trimestre en mujeres con hipertensión crónica o historia previa de preeclampsia, sin embargo la
eficacia de esta indicación ha sido cuestionada.
Myers JE, Baker PN. Hupertensive diseases and eclampsia. Curr Opin Obstet Gynecol 2002;
14: 119-125
2. Tierney, McPhee, Papadakis. Diagnóstico clínico y tratamiento 2003. 38ª ed, México, Manual
Moderno, 2003: 770-773
3. Wilson MI, Goodwin TM, Pan VI, Ingles SA. Molecular epidemiology of preeclampsia. Obstet
and Gynecol Survey 2003; 58(1):39-66
4. Burrow GM. Complicaciones médicas durante el embarazo. 4ª ed, México, McGraw-Hill
panamericana: 1996: 1-25
5. Guyton AC, Hall JE. Embarazo y lactancia en: Tratado de fisiología médica, 10ª ed, México,
McGraw-Hill Interamericana 2001: 1135-45
6. Vaticon D. Fisiología de la fecundación, embarazo, parto y lactancia, en: Tresguerres JAF.
Fisiología Humana. México, Interamericana McGraw-Hill, 1992: 1086-1109
7. Pridjian G, Puschett JB. Preeclampisa. Part 1: Clinical and Pathophysiologic Considerations.
Obstet and Gynecol Survey 2002; 57 (9): 598-618
8. Pridjian G, Puschett JB. Preeclampisa. Part I1: Experimental and Genetic Considerations.
Obstet and Gynecol Survey 2002; 57 (9): 619-40
9. IMSS. Embarazo de alto riesgo. Guía diagnóstica terapéutica. Rev Med IMSS 1998;
36(1):45-60
36.- Se trata de femenino de 47 años que presenta sinusorrragia de 3 meses, sin alteraciones
menstruales previas, por lo demás asintomática, el diagnóstico más probable a la
especuloscopía es:
a) Adenomiosis
b) Malformaciones Müllerianas
c) Pólipo endometrial
d) Pólipo endocervical
Pólipo es toda formación sésil o pediculada que se fija a la cavidad uterina. Las
formaciones polipoideas dentro del aparato genital femenino se dividen en endometriales y
cervicales. Dentro de los pólipos cevicales se diferencian los ectocervicales y los
endocervicales; estos últimos son considerados como los verdaderos por estar constituidos
por epitelio cilíndricco. No se conocen con certeza los factores asociados a su histogénesis
(teoría más admitida: hiperplasia focal de la mucosa endocervical por estímulo estrogénico)
(1). Los pólipos cervicales se consideran la forma más frecuente de tumor benigno de
cérvix.
Constituyen de un 3 a un 10% de las consultas ginecológicas; presentan máxima incidencia
entre la 4° y 5° década de la vida. Algunos estudios demuestran que hasta en el 56% de
casos de mujeres postmenopáusicas se encuentran asociados pólipos cervicales y
endometriales; es mucho menor esta frecuencia antes de la menopausia (2). La clínica de
presentación más común es la hemorragia intermenstrual, espontánea o durante el coito (3).
También pueden ser asintomáticos (hallazgo casual en estudio ecográfico) o, en ocasiones,
alcanzar grandes tamaños: llegan a aflorar por el orificio cervical externo; se denominan
"pólipos gigantes" cuando superan 2 cm (4). Para el diagnóstico habitualmente se utiliza la
ecografía. Los casos sintomáticos se consideran de indicación quirúrgica por su riesgo de
degeneración a adenocarcinoma cervical.
Los pólipos cervicales constituyen una patología frecuente en la consulta ginecológica, sobre
todo en mujeres mayores de 20 años que han tenido hijos y son poco comunes antes de la
menarquia. En la mayoría de los casos, solamente se presenta un pólipo, aunque en ocasiones
se pueden encuentrar 2 ó 3. La época más frecuente es al final de su vida reproductiva y
después de la menopausia. En las primeras, las lesiones suelen ser más grandes, difíciles de
tratar y con frecuencia recidivan.
La causa de los pólipos cervicales aún no se ha comprendido completamente, pero con
frecuencia son el resultado de una infección crónica, una respuesta local anormal a los niveles
de estrógeno o a una congestión local de los vasos sanguíneoscervicales. Clínicamente se
manifiesta con sangrado genital fuera de la menstruación, sobre todo durante la relación
sexual. Son benignos, no se malignizan pero deben ser extirpados para controlar la
sinusorragia.
37.- A 23-year-old nulligravid female has not menstruated in the past 4 months. Previously, her
menstrual cycles were regular. She is otherwise well and denies recent onset of stress, change
in exercise routine, headaches, visual field alterations, or galactorrhea.. She has a body mass
index of 24, blood pressure of 120/78 mm Hg, and does not appear hirsute. No adnexal masses
can be palpated. Laboratory investigations reveal a negative beta human chorionic
gonadotropin (β-hCG), normal thyroid-stimulating hormone (TSH), and prolactin levels.
What is the next best step in the management of this patient?
a) Measurement of luteinizing hormone.
b) CT scan of the sella turca.
c) Prescribe oral estrogen for 21 days followed by 7 days of medroxyprgesterona and
reevaluate.
d) Preescribe 7 days of medroxyprgesterona and reevaluate.
La Prueba de respuesta a progestágenos se basa en la observación de que el tratamiento con
progestágeno (acetato de medroxiprogesterona 10 mg por 5 a 6 días) solo induce la
menstruación en las mujeres con concentraciones normales de estrógenos circulantes. Una
prueba positiva (hemorragia después de concluir el tratamiento con progestágenos) señala
cifras normales de producción de estrógenos y una prueba negativa (sin hemorragia por
privación), hipogonadismo franco.
Danforth, Tratado de Obstetricia y Ginecologia, 9ª Edición, Ed. Mc Graw Hill Interamericana,
Pág 668
38.- Se trata de femenino de 27 años, Gesta 1, Para 1. Con dos citologías “lesión de alto
grado”, prueba de Schiller positiva y biopsia de cérvix que demuestra carcinoma “In Situ”. La
conducta es:
a) Conización.
b) Histerectomía total abdominal.
c) Histerectomía y salpingooforectomía bilateral.
d) Electrocauterización del cérvix.
La conización cervical es el tratamiento de elección en pacientes con cáncer cervicouterino
microinvasor y más si existedeseo de fertilidad. Asimismo, la histerectomía extrafasciales un
método adecuado en lesiones de 0.5 a 3 mm de invasión.Además se propone que, para
pacientes con lesiones de 3.1 a 5 mm de invasión, a partir de la membrana inicial sinfactores
de mal pronóstico como invasión vascular y linfática, sean tratadas con histerectomía
extrafascial, ya que en aquellas a las que se realizó linfadenectomía pélvica, con este tipo de
lesión, no se encontró metástasis a ganglios linfáticos.
Resultados del tratamiento en cáncer cervicouterino microinvasor en el Instituto Nacional de
Cancerología de México (1980-1999)
1.- Mestwerdt G. Fruhdiagnose des Kollumkarzinoms. Zentralb Gynaekol, 1947 ;69 :326.
2. - Morrow CP, Curtin JP. Surgery for cervical neoplasia. In Gynecologic Cancer Surgery. New
York, Churchill Livingstone, 1996, p 472.3.
3. - Burghardt E, Holzer E. Diagnosis and treatment of microinvasive carcinoma of the cervix
uteri. J Obstet and Gynecol 1977; 49:641-653.
39.- Se trata de femenino de 35 años la cual presenta un nódulo mamario palpable de
aparición brusca. La ecografía revela un nódulo anecogénico, de limites muy precisos,
morfología regular y refuerzo posterior, único de 3.5 cms. de diámetro. El diagnóstico más
probable es:
a) Cáncer.
b) Displasia fibrosa.
c) Fibroadenoma.
d) Quiste
Quistes. Los quistes mamarios son fáciles de detectar con la ecosonografía. Pueden ser
lesiones únicas o múltiples que se observan como imágenes redondeadas, anecogénicas, de
paredes delgadas, contornos bien definidos, con importante reforzamiento acústico posterior y
sombras laterales delgadas. Pueden presentar septos intraquísticos y, en ocasiones, se
pueden observar ecos internos que sugieren detritus celulares o proceso inflamatorio. Se debe
descartar la presencia de lesiones intraquísticas o la coexistencia de otras alteraciones
benignas o malignas. En caso de ser sintomáticos, el tratamiento adecuado es la punción y
aspiración de la lesión con aguja guiada por palpación o ecosonografía de acuerdo con ell
tamaño, profundidad y características del contenido. El uso del ultrasonido garantiza el
vaciamiento completo.
REFERENCIAS:
Barth V, Prechtel K. Mama normal. En: Barth V, Prechtel K, editores. Atlas de patología de la
glándula mamaria.
2da ed. Madrid: Editorial Médica Panamericana, 1991.
Bush H, McCredie A. Carcinoma of the breast during pregnancy and lactation. In: Allen HH,
Nisker JA. Cancer in pregnancy. New York: Futura Publishing Co. Inc., 1986.
Byrd BF, Bayer DS, Robertson JC, Stephenson JE Jr.
Treatment of breast tumor associated with pregnancy and lactation. Ann Surg. 1962; 155:940-7.
40.- Femenino de 36 años, es atendida en consulta externa con reporte de papanicolaou que
reporta un NIC I, la especuloscopía se observa cérvix con ectropión periorificiario.
El agente etiológico más probable causante de esta infección es:
a) neisseria gonorreae.
b) clamidya trachomatis.
c) virus del papiloma humano.
d) treponema pallidum.
Examen ginecologia o_20196
Examen ginecologia o_20196
Examen ginecologia o_20196
Examen ginecologia o_20196
Examen ginecologia o_20196
Examen ginecologia o_20196
Examen ginecologia o_20196
Examen ginecologia o_20196
Examen ginecologia o_20196
Examen ginecologia o_20196
Examen ginecologia o_20196
Examen ginecologia o_20196
Examen ginecologia o_20196
Examen ginecologia o_20196
Examen ginecologia o_20196
Examen ginecologia o_20196
Examen ginecologia o_20196
Examen ginecologia o_20196
Examen ginecologia o_20196
Examen ginecologia o_20196
Examen ginecologia o_20196
Examen ginecologia o_20196
Examen ginecologia o_20196
Examen ginecologia o_20196
Examen ginecologia o_20196
Examen ginecologia o_20196
Examen ginecologia o_20196
Examen ginecologia o_20196
Examen ginecologia o_20196
Examen ginecologia o_20196
Examen ginecologia o_20196
Examen ginecologia o_20196
Examen ginecologia o_20196
Examen ginecologia o_20196
Examen ginecologia o_20196
Examen ginecologia o_20196
Examen ginecologia o_20196
Examen ginecologia o_20196
Examen ginecologia o_20196
Examen ginecologia o_20196
Examen ginecologia o_20196
Examen ginecologia o_20196
Examen ginecologia o_20196
Examen ginecologia o_20196
Examen ginecologia o_20196
Examen ginecologia o_20196
Examen ginecologia o_20196
Examen ginecologia o_20196

Weitere ähnliche Inhalte

Was ist angesagt? (20)

Hemorragia Uterina Anormal
Hemorragia Uterina AnormalHemorragia Uterina Anormal
Hemorragia Uterina Anormal
 
CONTRACCIONES UTERINAS
CONTRACCIONES UTERINASCONTRACCIONES UTERINAS
CONTRACCIONES UTERINAS
 
ROTURA UTERINA
ROTURA UTERINAROTURA UTERINA
ROTURA UTERINA
 
Perfil Biofísico Fetal
Perfil Biofísico FetalPerfil Biofísico Fetal
Perfil Biofísico Fetal
 
Ok amenaza de parto prematuro
Ok amenaza de parto prematuroOk amenaza de parto prematuro
Ok amenaza de parto prematuro
 
Amenorrea
AmenorreaAmenorrea
Amenorrea
 
Enfermedad pelvica inflamatoria
Enfermedad pelvica inflamatoriaEnfermedad pelvica inflamatoria
Enfermedad pelvica inflamatoria
 
Parto pretérmino
Parto pretérminoParto pretérmino
Parto pretérmino
 
Hemorragia uterina anormal
Hemorragia uterina anormalHemorragia uterina anormal
Hemorragia uterina anormal
 
Enfermedad Trofoblástica Gestacional
Enfermedad Trofoblástica GestacionalEnfermedad Trofoblástica Gestacional
Enfermedad Trofoblástica Gestacional
 
Puerperio fisiológico (Introducion)
Puerperio fisiológico (Introducion) Puerperio fisiológico (Introducion)
Puerperio fisiológico (Introducion)
 
Hemorragia uterina completo2
Hemorragia uterina completo2Hemorragia uterina completo2
Hemorragia uterina completo2
 
Ruptura Prematura de Membrana
Ruptura Prematura de MembranaRuptura Prematura de Membrana
Ruptura Prematura de Membrana
 
Amenaza de parto pretermino
Amenaza de parto pretermino Amenaza de parto pretermino
Amenaza de parto pretermino
 
Embarazo Ectópico. Dr. Jesús Martínez
Embarazo Ectópico. Dr. Jesús MartínezEmbarazo Ectópico. Dr. Jesús Martínez
Embarazo Ectópico. Dr. Jesús Martínez
 
AMEU
AMEUAMEU
AMEU
 
Altura de presentacion fetal y parto 1 aaa
Altura de presentacion fetal y parto 1 aaaAltura de presentacion fetal y parto 1 aaa
Altura de presentacion fetal y parto 1 aaa
 
Amenaza de Parto Pretermino
Amenaza de Parto PreterminoAmenaza de Parto Pretermino
Amenaza de Parto Pretermino
 
Enfermedad trofoblástica gestacional
Enfermedad trofoblástica gestacionalEnfermedad trofoblástica gestacional
Enfermedad trofoblástica gestacional
 
Puerperio
PuerperioPuerperio
Puerperio
 

Ähnlich wie Examen ginecologia o_20196

Ähnlich wie Examen ginecologia o_20196 (20)

CES2019-02: Cáncer de ovario (por René Pareja)
CES2019-02: Cáncer de ovario (por René Pareja)CES2019-02: Cáncer de ovario (por René Pareja)
CES2019-02: Cáncer de ovario (por René Pareja)
 
Enfermedad Inflamatoria Pelvica
Enfermedad Inflamatoria PelvicaEnfermedad Inflamatoria Pelvica
Enfermedad Inflamatoria Pelvica
 
GINECOLOGIA PRIMERA UNIDAD.pptx
GINECOLOGIA PRIMERA UNIDAD.pptxGINECOLOGIA PRIMERA UNIDAD.pptx
GINECOLOGIA PRIMERA UNIDAD.pptx
 
Dismenorrea
DismenorreaDismenorrea
Dismenorrea
 
Dismenorrea
DismenorreaDismenorrea
Dismenorrea
 
Cazares elenes orlando
Cazares elenes orlandoCazares elenes orlando
Cazares elenes orlando
 
Endometriosis final
Endometriosis finalEndometriosis final
Endometriosis final
 
Endometriosis Cedip
Endometriosis CedipEndometriosis Cedip
Endometriosis Cedip
 
Onfalocele
Onfalocele Onfalocele
Onfalocele
 
Dismenorrea
DismenorreaDismenorrea
Dismenorrea
 
Enfermedad trofoblastica gestacional y embarazo ectópico
Enfermedad trofoblastica gestacional y embarazo ectópicoEnfermedad trofoblastica gestacional y embarazo ectópico
Enfermedad trofoblastica gestacional y embarazo ectópico
 
Endometriosis
EndometriosisEndometriosis
Endometriosis
 
Cancer De Cervix Avanzado
Cancer De Cervix Avanzado Cancer De Cervix Avanzado
Cancer De Cervix Avanzado
 
Endometriosis
EndometriosisEndometriosis
Endometriosis
 
Abdomen agudo gineco obstetrico
Abdomen agudo gineco obstetricoAbdomen agudo gineco obstetrico
Abdomen agudo gineco obstetrico
 
Cáncer cervicouterino en el primer nivel
Cáncer cervicouterino en el primer nivelCáncer cervicouterino en el primer nivel
Cáncer cervicouterino en el primer nivel
 
05 09 05 cancer de cuello uterino www.gftaognosticaespiritual.org
05 09 05 cancer de cuello uterino www.gftaognosticaespiritual.org05 09 05 cancer de cuello uterino www.gftaognosticaespiritual.org
05 09 05 cancer de cuello uterino www.gftaognosticaespiritual.org
 
SANGRADO UTERINO ANORMAL.pptx
SANGRADO UTERINO ANORMAL.pptxSANGRADO UTERINO ANORMAL.pptx
SANGRADO UTERINO ANORMAL.pptx
 
Cancer de ovario
Cancer de ovarioCancer de ovario
Cancer de ovario
 
endometriosis HCSC.pptx
endometriosis HCSC.pptxendometriosis HCSC.pptx
endometriosis HCSC.pptx
 

Kürzlich hochgeladen

10. Protocolo de atencion a victimas de violencia sexual.pptx
10. Protocolo de atencion a victimas de violencia sexual.pptx10. Protocolo de atencion a victimas de violencia sexual.pptx
10. Protocolo de atencion a victimas de violencia sexual.pptxKatherineReyes36006
 
TEXTO PRN 8VA ESPAÑOL.pdf reanimacion neonatal
TEXTO PRN 8VA ESPAÑOL.pdf reanimacion neonatalTEXTO PRN 8VA ESPAÑOL.pdf reanimacion neonatal
TEXTO PRN 8VA ESPAÑOL.pdf reanimacion neonatalJanKarlaCanaviriDelg1
 
Histología del pelo o cabello-Medicina.pptx
Histología del pelo o cabello-Medicina.pptxHistología del pelo o cabello-Medicina.pptx
Histología del pelo o cabello-Medicina.pptx Estefa RM9
 
Presentación de las glandulas endocrinas del páncreas
Presentación de las glandulas endocrinas del páncreasPresentación de las glandulas endocrinas del páncreas
Presentación de las glandulas endocrinas del páncreasanabel495352
 
asincronias ventilatorias-ventilacion mecanica
asincronias ventilatorias-ventilacion mecanicaasincronias ventilatorias-ventilacion mecanica
asincronias ventilatorias-ventilacion mecanicaAlexaSosa4
 
Cuadro comparativo de las enfermedades exantematicas 2022.docx
Cuadro comparativo de las enfermedades exantematicas 2022.docxCuadro comparativo de las enfermedades exantematicas 2022.docx
Cuadro comparativo de las enfermedades exantematicas 2022.docxandreapaosuline1
 
Presentación ojo anatomía Quiroz en pdf
Presentación ojo anatomía Quiroz en pdfPresentación ojo anatomía Quiroz en pdf
Presentación ojo anatomía Quiroz en pdfORONARAMOSBARBARALIZ
 
Clase 17 Artrologia MMII 3 de 3 (Pie) 2024 (1).pdf
Clase 17 Artrologia MMII 3 de 3 (Pie) 2024 (1).pdfClase 17 Artrologia MMII 3 de 3 (Pie) 2024 (1).pdf
Clase 17 Artrologia MMII 3 de 3 (Pie) 2024 (1).pdfgarrotamara01
 
MÚSCULOS DEL CUELLO DESCRIPCIÓN ORIGEN INSERCIÓN E INERVACION
MÚSCULOS DEL CUELLO DESCRIPCIÓN ORIGEN INSERCIÓN E INERVACIONMÚSCULOS DEL CUELLO DESCRIPCIÓN ORIGEN INSERCIÓN E INERVACION
MÚSCULOS DEL CUELLO DESCRIPCIÓN ORIGEN INSERCIÓN E INERVACIONPinedaValderrabanoAi
 
1. HISTORIA DE LA FISIOTERAPIA EN EL MUNDO.pptx
1. HISTORIA DE LA FISIOTERAPIA EN EL MUNDO.pptx1. HISTORIA DE LA FISIOTERAPIA EN EL MUNDO.pptx
1. HISTORIA DE LA FISIOTERAPIA EN EL MUNDO.pptxSarayAcua2
 
ANAMNESIS Y EXAMEN FISICO DEL SISTEMA RENAL.pptx
ANAMNESIS Y EXAMEN FISICO DEL SISTEMA  RENAL.pptxANAMNESIS Y EXAMEN FISICO DEL SISTEMA  RENAL.pptx
ANAMNESIS Y EXAMEN FISICO DEL SISTEMA RENAL.pptxCENTRODESALUDCUNCHIB
 
glucólisis anaerobia.pdf
glucólisis                 anaerobia.pdfglucólisis                 anaerobia.pdf
glucólisis anaerobia.pdfLuzElena608762
 
Músculos de la pierna y el pie-Anatomía.pptx
Músculos de la pierna y el pie-Anatomía.pptxMúsculos de la pierna y el pie-Anatomía.pptx
Músculos de la pierna y el pie-Anatomía.pptx Estefa RM9
 
1 mapa mental acerca del virus VIH o sida
1 mapa mental acerca del virus VIH o sida1 mapa mental acerca del virus VIH o sida
1 mapa mental acerca del virus VIH o sidagsandovalariana
 
Atlas de Hematología para estudiantes univbersitarios.pdf
Atlas de Hematología para estudiantes univbersitarios.pdfAtlas de Hematología para estudiantes univbersitarios.pdf
Atlas de Hematología para estudiantes univbersitarios.pdfCarlosNichoRamrez
 
Introduccion a la Consejeria Pastoral.pptx
Introduccion a la Consejeria Pastoral.pptxIntroduccion a la Consejeria Pastoral.pptx
Introduccion a la Consejeria Pastoral.pptxOlgaRedchuk
 
Anticoncepcion actualización 2024 según la OMS
Anticoncepcion actualización 2024 según la OMSAnticoncepcion actualización 2024 según la OMS
Anticoncepcion actualización 2024 según la OMSferblan28071
 
ESCALAS DE VALORACION EN ENFERMERIA.pptx
ESCALAS DE VALORACION EN ENFERMERIA.pptxESCALAS DE VALORACION EN ENFERMERIA.pptx
ESCALAS DE VALORACION EN ENFERMERIA.pptxmfy7bkb299
 
Corazon parte 1 introducción - Latarjet.
Corazon parte 1 introducción - Latarjet.Corazon parte 1 introducción - Latarjet.
Corazon parte 1 introducción - Latarjet.AdrianaBohrquez6
 
Psorinum y sus usos en la homeopatía y la dermatología
Psorinum y sus usos en la homeopatía y la dermatologíaPsorinum y sus usos en la homeopatía y la dermatología
Psorinum y sus usos en la homeopatía y la dermatologíaFelixGutirrez3
 

Kürzlich hochgeladen (20)

10. Protocolo de atencion a victimas de violencia sexual.pptx
10. Protocolo de atencion a victimas de violencia sexual.pptx10. Protocolo de atencion a victimas de violencia sexual.pptx
10. Protocolo de atencion a victimas de violencia sexual.pptx
 
TEXTO PRN 8VA ESPAÑOL.pdf reanimacion neonatal
TEXTO PRN 8VA ESPAÑOL.pdf reanimacion neonatalTEXTO PRN 8VA ESPAÑOL.pdf reanimacion neonatal
TEXTO PRN 8VA ESPAÑOL.pdf reanimacion neonatal
 
Histología del pelo o cabello-Medicina.pptx
Histología del pelo o cabello-Medicina.pptxHistología del pelo o cabello-Medicina.pptx
Histología del pelo o cabello-Medicina.pptx
 
Presentación de las glandulas endocrinas del páncreas
Presentación de las glandulas endocrinas del páncreasPresentación de las glandulas endocrinas del páncreas
Presentación de las glandulas endocrinas del páncreas
 
asincronias ventilatorias-ventilacion mecanica
asincronias ventilatorias-ventilacion mecanicaasincronias ventilatorias-ventilacion mecanica
asincronias ventilatorias-ventilacion mecanica
 
Cuadro comparativo de las enfermedades exantematicas 2022.docx
Cuadro comparativo de las enfermedades exantematicas 2022.docxCuadro comparativo de las enfermedades exantematicas 2022.docx
Cuadro comparativo de las enfermedades exantematicas 2022.docx
 
Presentación ojo anatomía Quiroz en pdf
Presentación ojo anatomía Quiroz en pdfPresentación ojo anatomía Quiroz en pdf
Presentación ojo anatomía Quiroz en pdf
 
Clase 17 Artrologia MMII 3 de 3 (Pie) 2024 (1).pdf
Clase 17 Artrologia MMII 3 de 3 (Pie) 2024 (1).pdfClase 17 Artrologia MMII 3 de 3 (Pie) 2024 (1).pdf
Clase 17 Artrologia MMII 3 de 3 (Pie) 2024 (1).pdf
 
MÚSCULOS DEL CUELLO DESCRIPCIÓN ORIGEN INSERCIÓN E INERVACION
MÚSCULOS DEL CUELLO DESCRIPCIÓN ORIGEN INSERCIÓN E INERVACIONMÚSCULOS DEL CUELLO DESCRIPCIÓN ORIGEN INSERCIÓN E INERVACION
MÚSCULOS DEL CUELLO DESCRIPCIÓN ORIGEN INSERCIÓN E INERVACION
 
1. HISTORIA DE LA FISIOTERAPIA EN EL MUNDO.pptx
1. HISTORIA DE LA FISIOTERAPIA EN EL MUNDO.pptx1. HISTORIA DE LA FISIOTERAPIA EN EL MUNDO.pptx
1. HISTORIA DE LA FISIOTERAPIA EN EL MUNDO.pptx
 
ANAMNESIS Y EXAMEN FISICO DEL SISTEMA RENAL.pptx
ANAMNESIS Y EXAMEN FISICO DEL SISTEMA  RENAL.pptxANAMNESIS Y EXAMEN FISICO DEL SISTEMA  RENAL.pptx
ANAMNESIS Y EXAMEN FISICO DEL SISTEMA RENAL.pptx
 
glucólisis anaerobia.pdf
glucólisis                 anaerobia.pdfglucólisis                 anaerobia.pdf
glucólisis anaerobia.pdf
 
Músculos de la pierna y el pie-Anatomía.pptx
Músculos de la pierna y el pie-Anatomía.pptxMúsculos de la pierna y el pie-Anatomía.pptx
Músculos de la pierna y el pie-Anatomía.pptx
 
1 mapa mental acerca del virus VIH o sida
1 mapa mental acerca del virus VIH o sida1 mapa mental acerca del virus VIH o sida
1 mapa mental acerca del virus VIH o sida
 
Atlas de Hematología para estudiantes univbersitarios.pdf
Atlas de Hematología para estudiantes univbersitarios.pdfAtlas de Hematología para estudiantes univbersitarios.pdf
Atlas de Hematología para estudiantes univbersitarios.pdf
 
Introduccion a la Consejeria Pastoral.pptx
Introduccion a la Consejeria Pastoral.pptxIntroduccion a la Consejeria Pastoral.pptx
Introduccion a la Consejeria Pastoral.pptx
 
Anticoncepcion actualización 2024 según la OMS
Anticoncepcion actualización 2024 según la OMSAnticoncepcion actualización 2024 según la OMS
Anticoncepcion actualización 2024 según la OMS
 
ESCALAS DE VALORACION EN ENFERMERIA.pptx
ESCALAS DE VALORACION EN ENFERMERIA.pptxESCALAS DE VALORACION EN ENFERMERIA.pptx
ESCALAS DE VALORACION EN ENFERMERIA.pptx
 
Corazon parte 1 introducción - Latarjet.
Corazon parte 1 introducción - Latarjet.Corazon parte 1 introducción - Latarjet.
Corazon parte 1 introducción - Latarjet.
 
Psorinum y sus usos en la homeopatía y la dermatología
Psorinum y sus usos en la homeopatía y la dermatologíaPsorinum y sus usos en la homeopatía y la dermatología
Psorinum y sus usos en la homeopatía y la dermatología
 

Examen ginecologia o_20196

  • 1. Universidad La Salle. Facultad Mexicana de Medicina. Curso de Extensión Universitaria para la Preparación del Examen Nacional para Aspirantes a Residencias Médicas. Examen de Ginecología y Obstetrícia. 1.- Se trata de paciente femenino de 29 años de edad con tumor anexial de 6 cm líquido, dolor abdominal, fiebre, leucorrea, con historia de cervicovaginitis de repetición y dispareunia crónica, última menstruación hace una semana. El diagnóstico más probable de ésta paciente es: a) Cistoadenoma b) Enfermedad pélvica inflamatoria c) Embarazo ectópico d) Quiste de ovario La EIP puede cursar con los siguientes síntomas:  Dolor abdominal bajo (incluyendo dolor anexial, dispareunia). Es el síntoma más frecuente (95%)  Aumento del flujo vaginal, flujo de características anormales (74%)  Sangrado anormal (intermestrual, poscoital) (45%)  Síntomas urinarios (35%)  Vómitos (14%)  Es posible la ausencia de síntomas Y en ella podemos encontrar estos signos:  Dolor a la movilización del cuello, dolor anexial en la exploración vaginal bimanual (99%)  En el examen con espéculo observamos cervicitis y descarga endocervical purulenta (74%)  Fiebre (> 38º C) (menos del 47%).  Masa pélvica: sugiere abceso tuboovárico (ATO)  Peritonitis CRITERIOS CLINICOS PARA EL DIAGNOSTICO DE SALPINGITIS a. Dolor abdominal con o sin rebote. b. Sensibilidad a la movilización del cérvix. c. Sensibilidad anexial. Los tres criterios anteriores son necesarios para establecer el diagnóstico, con uno o más de los siguientes:
  • 2. a. Extendido de Gram de endocérvix positivo, para diplococos gram negativos intracelulares b. Temperatura mayor de 38°C c. Leucocitosis (mayor de 10.000 por c.c.) d. Material purulento (positivo para leucocitos) en la cavidad peritoneal obtenido por culdocentesis o laparoscopia. Establecido el diagnóstico clínico de EPI, se debe hacer la definición del estado clínico y anatómico de la patología pélvica: a) No complicada (limitada a trompas u ovarios) 1) Sin peritonitis pélvica 2) Con peritonitis pélvica b) Complicada (masa inflamatoria o absceso que compromete trompa (s) u ovario (s) 1) Sin peritonitis pélvica 2) Con peritonitis pélvica o Beigi RH, Wiesenfeld HC. Pelvic inflammatory disease: new diagnostic criteria and treatment. Obstet Gynecol Clin Norh Am. 2003; 30 (4): 777 – 93 o Center for Disease Control. Guidelines for treatment of sexually transmited diseases. MMWR Recomm Rep. 2002 May 10;51(RR-6):1-78 o Center for Disease Control. Guidelines for prevention and management (MMWR. 40: 1 - 25 1991) Pelvic inflammatory disease: guidelines for prevention and management. MMWR Recomm Rep. 1991 Apr 26;40(RR-5):1-25. o Hager WD, Eschenbach DA, Spence MR, Sweet RL. Criteria for diagnosis and grading of salpingitis. Obstet Gynecol. 1983 Jan;61(1):113-4. o Prodigy Guidance. Pelvic inflammatory disease. [Internet]. UK : NHS, Department of Health; 2003. [Acceso 18 de Junio de 2005]. Disponible en: o Ross J. Pelvic inflammatory disease. Clin Evid. 2004 Dec;(12):2259-65. o Royal College of Obstetricians and Gynecologists. Pelvic Inflammatory Disease. Guideline nº 32. [Internet]. RCOG; Mayo 2003. [Acceso 18 de Junio de 2005]. 2.-Femenino de 25 años con antecedentes de G/2, P/1, C/1 acude al servicio de consulta externa, refiere que presenta una secreción transvaginal bastante líquida, de baja viscosidad, maloliente de color amarillo y gris, espumoso. El tratamiento de elección para esta entidad es: a) Metronidazol 500 mg. VO c/12 por 7 a 10 días b) Ampicilina 1g VO c/ 6 hrs. c) Clotrimazol + Metronidazol 500 mg VO c/12 hrs. por 7 a días d) Clindamicina 300 mg VO cada 12 hrs. x 5 días.
  • 3. Referencias bibliográficas: 1. Secretaría de Salud. Norma Oficial Mexicana NOM -039-SSA2-2002, Para la prevención y control de las infecciones de transmisión sexual. D.O.F. 19 de Septiembre 2003. 2. Kettler H, White K, Hawkes S. Mapping the landscape for sexually transmitted infections: key findings and recommendations. Geneva, TDR (TDR/STI/ IDE/04.1). 3. CDC. Trends in Reportable Sexually Transmitted Diseases in the United States. CDC, National Report. 2004 4. Distribución de los casos nuevos de enfermedades por mes Estados Unidos Mexicanos 2004. Sistema Único de Información para la Vigilancia Epidemiológica/Dirección General de Epidemiología/SSA 5. Aral S O. Sexual risk behavior and infection: epidemiological considerations. Sex. Transm. Inf. 2004;80:8-12 3.- Femenino de 25 años, con embarazo de término, sin antecedentes de control prenatal. G 3. C-1. Se ingresa al servicio de obstetricia por presentar actividad uterina regular y dolorosa. Ef.: Deambulante, tranquila, adecuada coloración de tegumentos, abdomen con fondo uterino a 32 cm. con producto único vivo en situación transversa dorso inferior FCF 144, al tacto vaginal cérvix dilatado a 3 cm. y membranas íntegras. Se realiza cesárea con retención de placenta e invasión a vejiga. Estudio de gabinete de elección previo al evento obstétrico es: a) prueba sin estrés b) perfil biofísico c) simple de abdomen d) ultrasonido doppler PLACENTA:  Se divide en:  Placenta acreta.  Placenta increta.  Placenta percreta.
  • 4.  ACRETA: Las vellosidades se adhieren al miometrio.  INCRETA: Penetran más de la mitad del espesor del miometrio.  PERCRETA: Atraviesa todo el espesor del miometrio, llegando a la serosa, incluso atravesándola y adhiriéndose a órganos vecinos. Factores:  Endometriósis previa.  Tumores submucosos. (Miomas)  Cicatríz uterina previa. (Cesárea, miomectomía)  Implantación baja. (Placenta previa)  Malformaciones placentarias. (Placenta extracorial)  Legrado enérgico previo.  Extracción manual previa de una placenta. Diagnóstico Pre-parto  Ultrasonografía Döppler.  Resonancia Magnética. Diagnóstico transparto-:  Placenta retenida por más de 20 minutos.  Imposibilidad para encontrar un plano de separación placentaria cuando se intenta su extracción manual.  Hemorragia incontrolable después de la pseudoextracción.  El diagnóstico histopatológico corrobora el diagnóstico clínico.  Escenario menos deseable. Tratamiento:  Histerectomía Obstétrica.  Constituye una cirugía no planeada y secundaria al hallazgo del acretismo placentario con sangrado incohercible.  Cesárea-Histerectomía. (Con diagnóstico previo)  Cirugía planificada ante un correcto diagnóstico prenatal.  Recomendación ACOG:  Maduración pulmonar intrauterina.  Inyectar al cordón umbilical 50 mg de metrotexate.  Ligar el cordón en el nacimiento placentario y dejar la placenta in-situ.  Embolización inmediata de arterias uterinas bilaterales, así como de ramas de la división anterior de la arteria iliaca interna con alcohol polivinílico.  Continuar con 5 dosis I.M. de 50 mg de metrotexate y cuantificar niveles de βhCG.  Programar Histerectomía Total Radical Abdominal y/o Cistectomía parcial y/o resección pared anterior recto. Lee et al. Conservative Management of Placenta Percreta. Obstet Gynecol, 112(2):421-424
  • 5. 4.- Mujer que acude al servicio de ginecología por referir ciclos opso-menorreicos desde el inicio de su menarquia, ha incrementado 15 Kg. de lo que pesaba habitualmente, se aprecia una gran cantidad de acné, pero además refiere depilarse el área del bigote cada semana, y cree que esto le sensibiliza la piel para que aumente el acné. El diagnóstico más probable en esta paciente es: a) Sx. De Asherman b) Sx. Stein Leventhall c) Sx. Amenorrea Galactorrea d) Sx. Karman Síndrome de Ovario Poliquístico (SOP) es uno de los más comunes trastornos endocrinos que afectan a las mujeres alrededor del 5% al 10% de las mujeres en edad reproductiva (12-45 años) y se piensa que es una de las principales causas de la infertilidad femenina. Las características principales son la obesidad, anovulación (dando lugar a la menstruación irregular) o amenorrea, acné, y las cantidades excesivas o los efectos de androgénicos (masculinizantes) hormonas. Los síntomas y la severidad del síndrome varían mucho entre las mujeres. Si bien las causas son desconocidas, resistencia a la insulina, la diabetes y la obesidad están fuertemente correlacionadas con el SOP. Bulun SE, Adashi EY. The physiology and pathology of the female reporductive axis. In: Kronenberg HM, Melmed S, Polonsky KS, Larsen PR, eds. Williams Textbook of Endocrinology. 11th ed. Philadelphia, Pa: Saunders Elsevier; 2008:chap 16. 5.- Se trata de mujer de 43 años, G- 4, P-3, A-1, con diagnóstico de anemia ferropénica, de 9.5 g/dl, refiere ciclos menstruales de 31,32 x 8,9 días de duración, acompañados de coágulos, los cuales aparecieron después del nacimiento de su segundo hijo hace 13 años. E.F.: Buen estado general, TA 130/80, genitales con evidencia de sangrado activo, al tacto vaginal se detecta útero de consistencia firme voluminosa, irregular, aproximadamente de 12 cm. anexos libres. El diagnóstico más probable es: a) Adenomiosis uterina. b) Cáncer cervicouterino. c) Miomatosis uterina. d) Hiperpalsia adenomatosa de endometrio. MIOMATOSIS UTERINA Definición: Tumor benigno que se origina en el miometrio, por lo que su componente histológico predominante es el tejido muscular y, en menor medida, el conectivo y fibroso. El único tratamiento efectivo es el quirúrgico; sin embargo, sólo requieren ser tratados aquellos que producen síntomas. Evaluación y Diagnóstico: Historia: 1. El síntoma más frecuente suele ser la hemorragia uterina. 2. Los síntomas principales están relacionados con el crecimiento del tumor. 3. La paciente puede notar una masa en hipogastrio o abdomen inferior.
  • 6. 4. La masa se puede asociar a dolor pélvico, o manifestaciones por compresión de órganos o estructuras vecinas. 5. Puede haber alteración de la fertilidad. Examen Físico: 1. Se debe realizar con la vejiga y el recto vacuo. 2. El hallazgo primordial es el aumento de volumen y consistencia del útero, el cual puede ser simétrico (nódulos submucosos) o irregular (nódulos intramurales o subserosos). Exámenes Auxiliares: 1. Papanicolau cérvico–vaginal: Indicado siempre; permite descartar neoplasia epitelial cervical o cáncer infiltrante de cérvix. 2. Ultrasonido transabdominal y transvaginal: Indicado siempre; permite evaluar la localización, tamaño y número aproximado de miomas. 3. Hemoglobina, hematocrito: Indicado cuando hay historia de sangrado; orienta en la severidad del sangrado y anemia. 4. Hemograma y VSG: Indicado cuando hay historia de fiebre; si es anormal sugiere infección o necrosis del mioma (puede ser apropiado descartar infección de otro órgano o sistema). 5. Grupo sanguíneo y factor Rh: Si hay anemia severa o en el preoperatorio. 6. Perfil de coagulación (tiempo de protrombina y de tromboplastina parcial, recuento de plaquetas): Si hay historia de sangrado exagerado. 7. Gonadotrofina coriónica (sub–unidad sérica): Permite descartar posible embarazo en casos de ciclos irregulares, retraso menstrual o útero de consistencia blanda. Diagnóstico Diferencial: 1. Embarazo. 2. Tumor de ovario. 3. Enfermedad inflamatoria pélvica, complejo inflamatorio anexial pélvico. 4. Endometriosis. 5. Adenomiosis. 6. Tumor extragenital: colon, retroperitoneo. Referencias Bibliográficas: 1. Hillard PA. Benign Diseases of the Female Reproductive Tract: Symptoms and Signs. En: Berek JS, Adashi EY, Hillard PA, eds. Novak's Gynecology. Baltimore: Williams and Wilkins, 1996:331-97. 1. Hutchins FL, Greenber MD. Miomas Uterinos: Diagnóstico e Indicaciones de Tratamiento. Clinicas de Ginecología y Obstetricia. Temas Actuales. 1995;5:609-14. 2. Davis KM, Sclass WD. Tratamiento Médico para Miomatosis Uterina. Clinicas de Ginecología y Obstetricia. Temas Actuales. 1995;5:671-81. 1. Selwyn P, Oskowitz MB. Leiomyomata Uteri. En: Friedman EA, ed. Gynecological Decision Making. St. Louis: Mosby, 1983:148-9. 1. Diaz Huamán V. Tumores Benignos del Aparato Reproductor Femenino. En: Ludmir A, Cervantes R, Castellano C, eds. Ginecología y Obstetricia, Prevención - Diagnóstico - Tratamiento. Lima: Concytec, 1996:907-25.
  • 7. 6.- Femenino de 34 años que cursa con 38.5 semanas de gestación que ingresa al servicio con trabajo de parto. En el transcurso de trabajo de parto durante la dilatación presenta dolor intenso y brusco. A la exploración usted observa metrorragia escasa y aumento del tono uterino a la palpación abdominal que resulta muy doloroso. El diagnóstico más probable es: a) Placenta previa. b) Rotura de vasos previos. c) Crioamnionitis hemorrágica. d) Desprendimiento de placenta. Fisiopatología de la hemorragia La hemorragia es el signo fundamental que domina el cuadro clínico de la placenta previa. Para explicar su mecanismo existen distintas teorías: Mecanismo de Jacquemier: Se produce crecimiento armónico de la placenta y del útero hasta la semana 26, 28. Después el segmento inferior crece más deprisa y favorece el despegamiento lo que origina la hemorragia en el embarazo. Mecanismo de Schroeder: Las contracciones uterinas en el parto traccionan del segmento inferior hacia arriba y empujan al feto hacia abajo despegando la placenta. Mecanismo de Pinard: Explica las hemorragias gestacionales y del parto. El estiramiento de las membranas de la zona de menor radio (orificio interno cervical) como consecuencia de las contracciones tira de la placenta y la desprenden. Mecanismo de Bartholomew: Explica la hemorragia en los casos de placenta previa central. La zona placentaria que reviste el orificio interno es un área isquémica ya que no recibe vasos deciduales. A este nivel disminuye la presión sanguínea, por lo que la sangre tiende a dirigirse hacia esta zona y escapa por la cara materna. En el alumbramiento también puede haber una hemorragia importante producida por un doble mecanismo: -desprendimiento parcial antes de la expulsión en los casos de placenta oclusiva. -atonía uterina en la zona de inserción después de expulsada la placenta y vascularización anómala. Manifestaciones clínicas de la placenta previa en el embarazo - Síntomas: Principalmente la hemorragia. Toda hemorragia vaginal acontecida en el tercer trimestre debe hacer pensar en una placenta previa. Las hemorragias suelen ser espontáneas, no acompañadas de dolor, de sangre roja y se presentan de forma intermitente, con intervalos variables entre las mismas. Progresivamente se van haciendo más frecuentes y más graves. La primera hemorragia suele aparecer en forma inesperada generalmente nocturna, cesando en menos de media hora. Las hemorragias ulteriores son más graves y más precoces. - Signos: La consecuencia fundamental es la anemia materna que depende de la cuantía de la hemorragia (la sangre es de origen materna ya que procede de espacios intervellosos). - Exploración: Exploración general para valorar la existencia de signos de anemia. Exploración obstétrica: valorar el tamaño del útero (adecuado para la edad gestacional), es blando e indoloro. A menudo la estética fetal está alterada (transverso, oblicuo, nalgas). No debe efectuarse tacto vaginal cuando haya existido hemorragia en embarazo avanzado por el riesgo de infección y de despegamiento y aumentar así la hemorragia. Auscultación fetal normal. Exploración ecográfica: es una técnica fundamental en el diagnóstico de la placenta previa. Permite determinar la localización placentaria y la variedad de la placenta previa. En general, la placenta puede identificarse a partir de la 9ª semana. No obstante, a lo largo de la gestación
  • 8. por crecimiento uterino se produce un cambio en sus relaciones con el útero ("emigración placentaria", imagen de desplazamiento). Así el diagnóstico de certeza de límites placentarios solo puede establecerse hacia la semana 34. Siempre será necesario hacer una adecuada identificación del orificio cervical interno (más fácil con sonda transvaginal). Añadiendo al estudio Doppler color se observa la vascularización y las zonas que sangran. 7.- Se trata de femenino de 24 años de edad gesta 1, tuvo un parto vaginal espontáneo con un producto con peso de 4,350 g. después de 5 minutos de tracción suave del cordón umbilical se expulsó la placenta, que parece estar intacta. Se inició el masaje del fondo uterino y se pidió a la enfermera que administrara 20 unidades de oxitocina en 100 ml de solución Ringer lactato. Después de una inspección cuidadosa del canal del parto se observa una laceración de segundo grado y una laceración de 2 cm en la pared vaginal izquierda que se intentó reparar. En la exploración física se encuentra un fondo uterino blando y atónico. Los signos vitales son: temperatura 37.1°C, TA 164/92, FC 130x’, FR 18 X’. El tratamiento más adecuado en éste caso es: (a) Oxitocina 10 unidades directas en goteo intravenoso (b) Metilergonovina 0.2 mg IM (c) Prostaglandina F 0.25 mg IM (d) Legrado Morgan M, Siddighi S. Ginecología y obstetricia, National Medical Series. 5° edición. Mc Graw Hill. Pp. 28. La atonía uterina es la causa más común de hemorragia puerperal. El masaje energético y la oxitocina diluida no han sido útiles para interrumpir la hemorragia y por tanto el siguiente paso es agregar un fármaco uterotónico. La metilergonovina está contraindicada porque la paciente se encuentra hipertensa a pesar de la hemorragia intensa, el siguiente fármaco es la prostaglandina. La administración de oxitocina no diluida, 10 UI por vía IV podría causar hipotensión grave. La exploración manual podría ser apropiada si se sospecha laceración como causa de hemorragia. El legrado es apropiado para la hemorragia puerperal tardía, cuando se sospecha retención de los productos de la concepción. 8.- Se trata de paciente femenino de 20 años de edad, que acude a consulta refiriendo presentar 6 meses sin regla. Se manifiesta preocupada por su sobrepeso, ha estado a dieta y ha perdido 6 kg en 8 meses. Actualmente pesa 46 Kg. con talla de 1,65 mts. Signos vitales dentro de sus parámetros normales. Se realiza prueba de embarazo con resultado negativa. La causa más probable de su amenorrea es: a) Disgenesia gonadal. b) Hipogonadismo hipogonadotropo. c) Síndrome de ovario poliquístico. d) Adenoma hipofisario. Sigue siendo práctica y eficaz la diferenciación de los hipogonadismos en hipergonadotrópicos (con fallo de función en la gónada) e hipogonadotrópicos (con fallo en hipotalámo y/o hipófisis). La deficiencia puede ser total o parcial y afectar a una o ambas funciones gonadales (producción de esteroides y germinal). Las causas son muy variadas. En la tabla 1 se han recopilado las principales de origen hipogonadotrópico, aunque algunas de ellas raramente se observan durante la infancia (p. ej., hiperprolactinemia) y en la tabla 2 las de origen hipergonadotrópico. Dentro del apartado de “idiopático” poco a poco se van dilucidando las
  • 9. causas y, así, con la ayuda de la biología molecular se encuentran mutaciones del gen del receptor de GnRH, de LH o alteraciones del gen DAX 1. Por otra parte debe tenerse en cuenta que en diferentes síndromes puede aparecer un hipogonadismo hipogonadotrópico (tabla 3). MANIFESTACIONES CLÍNICAS Y ANALÍTICAS La ausencia de datos clínicos de hipogonadismo durante la infancia es la regla. Tan sólo la presencia de micropene o criptorquidia en algún varón puede hacer sospechar de la existencia de un hipogonadismo, pero ni es patognomónica (se da también en el déficit de GH y en otras situaciones no hormonalmente deficitarias), ni su ausencia excluye la deficiencia gonadal. Por lo tanto, y si se descarta la sospecha del hipogonadismo por otras características asociadas (p. ej., diversos síndromes, alteraciones evidentes: tumoraciones o radioterapia hipotálamo-hipofisaria), sólo el retraso de aparición de la pubertad será la primera indicación de una deficiencia de estas características. Sin embargo, aquí se produce el principal problema para alcanzar un diagnóstico temprano al resultar muy difícil distinguirlo de una variante normal del desarrollo: el retraso constitucional del crecimiento y adolescencia. Respecto al hipogonadismo hipogonadotrópico los estudios hormonales son escasamente discriminatorios en la época previa a la pubertad por lo que el momento en que se realiza el diagnóstico, y por lo tanto el inicio del tratammiento, suele ser tardío. La ausencia de caracteres sexuales secundarios, junto con un tamaño infantil de las gónadas (por palpación testicular o ecografía ovárica), es suficiente para valorar una situación prepuberal. En todo caso, la confirmación, según el sexo, de bajos niveles de testosterona circulante, o de estrógenos (directamente o por citología vaginal/vesical), permiten una sencilla, fácil y rápida comprobación. No debe olvidarse que en relación a la causa del hipogonadismo (p. ej., una tumoración hipofisacualquier edad, es objetivar una anosmia (síndrome de Kallmann), más aún si es parcial. En los casos de hipogonadismo hipergonadotrópico puede encontrarse en ciertos periodos a lo largo de la infancia niveles elevados de gonadotrofinas basales y que tras estímulo con GnRH muestran una respuesta más elevada de lo normal. Sin embargo, lo más habitual es encontrarse con niveles propios de la infancia al realizar las determinaciones basales. Recientemente se ha demostrado por medio de una técnica muy sensible que no era cierta la pretendida ausencia de estradiol en las niñas, por lo que si se consigue generalizar su empleo será un arma valiosa de diagnóstico durante la infancia. TABLA 1. Causas de hipogonadismo hipogonadotrópico Panhipopituitarismo Idiopático Tumoración hipofisaria o hipotalámica Misceláneo Enfermedad granulomatosa Traumatismo Vasculitis Infarto Hemocromatosis Deficiencia aislada de gonadotrofinas Síndrome de Kallman y variantes Hipogonadismo hipotalámico idiopático Deficiencia aislada de LH Deficiencia aislada de FSH Hipoplasia adrenal congénita ligada al cromosoma X Defectos de desarrollo de la línea media Síndromes con diversas malformaciones (tabla 2) Prader-Willi Laurence-Moon-Bardet-Biedl Alteraciones sistémicas Deficiencias nutritivas Enfermedades crónicas Ponderales: desnutrición severa y obesidad mórbida Yatrogenia Farmacológica Radioterápica Quirúrgica Hiperprolactinemia
  • 10. Ejercicio físico de alta competición Retraso constitucional de la pubertad (?) BIBLIOGRAFÍA SELECCIONADA: Henkin RI, Bartter FC Studies on olfactory thresholds in normal man and in patients with adrenal cortical insufficiency: The role of adrenal cortical steroids and of serum sodium concentration. J Clin Invest 1966; 45: 1631-1639. 2. Schwanzel-Fukuda M, Abraham S, Crossin KL, Edelman GM, Pfaff DW. Immunocytochemical demonstration of neural cell adhesion molecule (NCAM) along the migration route of luteinizing hormone-releasing hormone (LHRH) neurons in mice J Comp Neurol 1992; 321: 1-18. 3. Schwanzel-Fukuda M, Bick D, Pfaff DW: Luteinizing hormone-releasing hormone (LHRH)-expressing cells do not migrate normally in an inherited hypogonadal (Kallmann) syndrome. Mol Brain Res 1989; 6: 311-326. 9.- Se trata de paciente femenino de 18 años de edad la cual acude a consulta por presentar amenorrea acompañada de profundas alteraciones del olfato. De los siguientes el diagnóstico más probable es: a) Síndrome de amenorrea-galactorrea. b) Amenorrea de causa uterina. c) Síndrome de ovario poliquístico. d) Amenorrea por alteración hipotalámica. • AMENORREA HIPOTALAMICA. (Hipogonadotrópica) – Psicógena. (Stress emocional) – Anorexia nerviosa. (Deficiencia nutricional) – Ejercicio excesivo. (Carrera de fondo, natación, gimnasia, ballet) – Fármacos. (Fenotiazina, reserpina, bloqueadores ganglionares, anticonceptivos) – Pseudociésis. – Síndrome de Kallman (Deficiencia selectiva de gonadotropinas y anosmia). El ejemplo clásico de la alteración hipotalámica que lleva a desórdenes del ciclo menstrual es el Síndrome de Kallman. Lectura Recomendada: Etiopatogenia de la amenorrea hipotalámica funcional Interacción de las respuestas hormonales del Sistema Nervioso Central y Neuropéptidos Periféricos Revista Argentina de Endocrinología y Metabolismo Copyright © 2008 por la Sociedad Argentina de Endocrinología y Metabolismo Vol 45 • No. 2
  • 11. 10.- Una mujer de 27 años G/4, P/0 con 6 SDG acude a su primera visita prenatal. Su historia obstétrica pasada es importante porque tiene tres pérdidas de producto en el segundo trimestre. Refiere que en las tres ocasiones al presentarse al hospital presentaba dilación cervical completa. No recuerda haber tenido contracciones dolorosas. Niega antecedentes médicos y quirúrgicos. El examen físico es normal incluyendo un examen pélvico que muestra un cervix largo y cerrado. Después de una larga discusión con la paciente ella pide que se le practique un cerclaje durante este embarazo. El momento más apropiado para realizarlo es: a) Inmediatamente b) 13 a 17 semanas c) 24 a 28 semanas d) 32 a 36 semanas El cerclaje cervical tiene sus indicaciones en la profilaxis y tratamiento de la incompetencia cervical. La incompetencia o insuficiencia cervical representa un 10% de las causas de parto pretérmino y está asociada a una importante morbimortalidad neonatal. Las modificaciones cervicales en el segundo trimestre de gestación son causa de parto prematuro y pueden deberse a: 1) Incompetencia cervical. 2) Pérdida de tejido conectivo tras una cirugía cervical (conización). 3) Defectos congénitos como la hipoplasia cervical tras exposición a dietilestilbestrol. 4) Infección intrauterina. Hasta un 51.5% de las pacientes con clínica compatible con incompetencia cervical enmascaran un cuadro de infección intraamniótica subclínica. Diferenciamos tres tipos de cerclaje: 1. El cerclaje se considera profiláctico o electivo (o primario) cuando se realiza de forma electiva por historia previa de incompetencia cervical antes de evidenciar cambios en el cerviz y generalmente suele realizarse entre las 13 y 17 semanas de gestación. 2. El cerclaje terapéutico secundario que se realiza tras la detección, en el seguimiento obstétrico, de modificaciones en el cérvix antes de las 26 semanas de gestación. Se realiza en pacientes con un riesgo potencial de parto pretérmino. 3. El cerclaje terapéutico terciario, en caliente, de rescate o “emergent cerclage” que se realiza en pacientes que presentan la membrana amniótica visible a través del orificio cervical externo o en vagina. GUIA CLÍNICA: INDICACIONES DEL CERCLAJE Unitat de Prematuritat. Servei de Medicina Maternofetal. Institut Clínic de Ginecologia, Obstetrícia i Neonatologia, Hospital Clínic de Barcelona Responsables del protocolo: T.Cobo, M. López, M. Palacio Creación: 24/01/07 Modificaciones: 05/09/07 Última actualización: 17/01/10
  • 12. 11.- Se trata de femenino de 67 años refiere intenso prurito vulvar y sensación quemante, al examen el introito vaginal se encuentra estenótico. De los siguientes es el tratamiento más apropiado es: a) 5-fluoracilo b) Testosterona tópica c) Corticoesteroides fluorados d) Estrógeno tópico Vulvovaginitis atrófica El hipoestrogenismo conduce a atrofia de la vagina y el vestíbulo vulvar, que los hace fácilmente irritables y susceptibles a infecciones secundarias. Las pacientes refieren sensación de quemadura, prurito, disuria, hipersensibilidad y dispareunia. Puede encontrarse al examen físico atrofia, fisuras superficiales, y un flujo vaginal acuoso1. Hay disminución del tamaño del introito2, pérdida de la rugosidad y la vagina toma una apariencia lisa y brillante. Los hallazgos histológicos revelan un epitelio vaginal delgado, disminución de los lechos capilares, y la citología muestra, a medida que la atrofia progresa, aumento de las células basales y disminución o ausencia de las células superficiales2. Se aconseja evitar el uso de jabones y demás irritantes de la piel. Se pueden utilizar lubricantes simultáneamente con los estrógenos o como terapia única, si hay alguna contraindicación a las hormonas. El tratamiento con estrógenos por vía sistémica o transvaginal mejora y restaura los signos y síntomas, y una a dos semanas después de iniciar el tratamiento los cambios de atrofia empiezan a mejorar rápidamente, se reduce el pH y se induce maduración vaginal y de la mucosa uretral, reduciendo la frecuencia de las infecciones urinarias3. La dosis y vía de administración debe ser debidamente individualizada4. Contraindicaciones al tratamiento con estrógenos, incluyen: la presencia de tumores estrógenosensibles, falla hepática terminal y antecedentes de tromboembolización relacionada con ellos. Menopausia y Piel. Parte II: Manifestaciones clínicas dermatológicas durante la menopausia MARÍA ISABEL BARONA C. Docente adjunto. Dermatóloga Universidad del Valle-Cali. 12.- Mujer de 26 años, es atendida en consulta en la clínica de displasias por papanicolau con lesión NIC I. Antecedentes: menarca 14 años, ritmo 30x5 eumenorreica, inicio de vida sexual a los 15 años, 2 parejas sexuales, método de planificación familiar oclusión tubaria bilateral, gestas 3 partos 3, cérvix con lesión acetoblanca con extensión lineal de 2 cm. Para confirmar el diagnóstico se debe realizar: a) Crioterapia de lesión. b) Captura de híbridos. c) Repetir colposcopía d) Biopsia de la lesión.
  • 13. 9.5.2 Las pacientes a quienes se les realizó citología cervical, cuyo resultado es LEIBG (infección por VPH, displasia leve o NIC 1); LEIAG (displasia moderada y grave o NIC 2 y 3) o cáncer deben enviarse a una clínica de colposcopía, para realizar estudio colposcópico. 9.5.3 Si el resultado de la citología es LEIBG, la colposcopía es satisfactoria y sin evidencia de LEIBG, se realizará control citológico en un año (Apéndice Normativo A) 9.5.4 Si la citología es de LEIBG, la colposcopía es satisfactoria y existe evidencia de lesión, se debe tomar una biopsia dirigida.
  • 14. 9.5.4.1 Si la biopsia dirigida es negativa, se realizará nueva colposcopía para verificar el diagnóstico y en caso necesario, tomar nueva biopsia dirigida y revalorar. 9.5.4.2 Si la biopsia dirigida es reportada como LEIBG se podrá dar tratamiento conservador: criocirugía, electrocirugía o laserterapia (sólo si cumple con las condiciones referidas en el Apéndice 1) o se podrá mantener a la paciente en vigilancia en la clínica de colposcopía, con colposcopía y estudio citológico cada seis meses, durante 24 meses. Jueves 31 de mayo de 2007 DIARIO OFICIAL (Primera Sección) 9.5.4.3 Si la biopsia dirigida es reportada como LEIAG (Lesión Intraepitelial Escamosa de Alto Grado) se realizará tratamiento conservador (electrocirugía o laserterapia). En las mujeres posmenopáusicas, dependiendo de las condiciones anatómicas del cérvix, se realizará tratamiento conservador en la clínica de colposcopía o tratamiento quirúrgico (histerectomía extrafascial) en el servicio que corresponda. 9.5.4.4 Si la biopsia dirigida reporta cáncer microinvasor o invasor, la paciente se transferirá a un Servicio o Centro Oncológico para su tratamiento correspondiente. 9.5.4.5 Si la citología reporta LEIBG y la colposcopía es no satisfactoria, se tomará cepillado endocervical (Apéndice Normativo A) 9.6 En caso de colposcopía no satisfactoria, negativa a LEIBG y con cepillado endocervical negativo, se continuará su control en la clínica de colposcopía en seis meses, con colposcopía y citología. 9.6.1.1 Si el cepillado endocervical reporta LEIBG se tratará a la paciente como LEIAG, con métodos conservadores escisionales. Jueves 31 de mayo de 2007 DIARIO OFICIAL (Primera Sección) Modificación a la Norma Oficial Mexicana NOM-014-SSA2-1994, Para la prevención, detección, diagnóstico, tratamiento, control y vigilancia epidemiológica del cáncer cérvico uterino. Al margen un sello con el Escudo Nacional, que dice: Estados Unidos Mexicanos.- Secretaría de Salud. MODIFICACION A LA NORMA OFICIAL MEXICANA NOM-014-SSA2-1994, PARA LA PREVENCION, DETECCION, DIAGNOSTICO, TRATAMIENTO, CONTROL Y VIGILANCIA EPIDEMIOLOGICA DEL CANCER CERVICO UTERINO. 7.3 El resultado del estudio citológico es descriptivo y debe ser informado de la siguiente manera: a.- Negativo a cáncer. b.- Negativo con proceso inflamatorio. c.- Displasia leve (NIC 1). d.- Displasia moderada (NIC 2). e.- Displasia grave (NIC 3). f.- Cáncer del cuello del útero in situ (NIC 3). g.- Cáncer microinvasor e invasor. h.- Adenocarcinoma. i.- Maligno no especificado
  • 15. 13.- Se trata de paciente de 33 años que cursa con 39 SDG; a la exploración física reflejos patelares hiperactivos, inquieta, se reportan cifras de TA 145/95, se realiza laboratorio que reporta proteinuria 2+,. El diagnóstico más probable es: a) Glomerulonefritis aguda b) Hipertensión esencial c) feocromocitoma d) Preeclampsia La hipertensión es la complicación médica más común del embarazo , aunque para algunos autores es la segunda complicación médica del embarazo sólo después de la anemia; es más frecuente en jóvenes durante el primer embarazo y en nulíparas de mayor edad, hipertensas previas y diabéticas. En México, también es la complicación más frecuente del embarazo, la incidencia es de 47.3 por cada 1 000 nacimientos y es además, la primera causa de ingreso de pacientes embarazadas a las unidades de terapia intensiva (debido a hemorragia masiva, para recibir soporte hemodinámico), según la secretaría de salud (2001) la mortalidad por complicaciones del embarazo ocupa el 15º lugar en la mortalidad hospitalaria en general. Además, la tasa de preeclampsia se ha incrementado 40% en el periodo entre 1990 y 1999 y constituye hasta 40% de los partos prematuros iatrogénicos. . Preeclampsia La preeclampsia es un síndrome clínico caracterizado por hipertensión con disfunción orgánica múltiple, proteinuria, edemas. Es definida como un incremento de al menos 140/90 mmHg después de la semana 20 de gestación, un incremento en la presión sanguínea diastólica de al menos 15 mmHg respecto a un nivel previo a la semana 20 combinado con proteinuria (> 300 mg en 24 horas). Las mediciones de la presión arterial citadas deben ser medidas al menos 2 ocasiones con por lo menos 6 horas de separación. La proteinuria puede ser una toma simple de orina al azar que indique al menos 30 mg/dL 3 ó ++ en dos muestras de orina1 según el tipo de prueba. El criterio del incremento de 30 mmHg en la presión sistólica y/o 15 mmHg en la presión diastólica respecto a valores previos a la semana 20 de gestación ha sido eliminado por ser poco específico15
  • 16. Myers JE, Baker PN. Hupertensive diseases and eclampsia. Curr Opin Obstet Gynecol 2002; 14: 119-125 2. Tierney, McPhee, Papadakis. Diagnóstico clínico y tratamiento 2003. 38ª ed, México, Manual Moderno, 2003: 770-773 3. Wilson MI, Goodwin TM, Pan VI, Ingles SA. Molecular epidemiology of preeclampsia. Obstet and Gynecol Survey 2003; 58(1):39-66 4. Burrow GM. Complicaciones médicas durante el embarazo. 4ª ed, México, McGraw-Hill panamericana: 1996: 1-25 5. Guyton AC, Hall JE. Embarazo y lactancia en: Tratado de fisiología médica, 10ª ed, México, McGraw-Hill Interamericana 2001: 1135-45 6. Vaticon D. Fisiología de la fecundación, embarazo, parto y lactancia, en: Tresguerres JAF. Fisiología Humana. México, Interamericana McGraw-Hill, 1992: 1086-1109
  • 17. 7. Pridjian G, Puschett JB. Preeclampisa. Part 1: Clinical and Pathophysiologic Considerations. Obstet and Gynecol Survey 2002; 57 (9): 598-618 8. Pridjian G, Puschett JB. Preeclampisa. Part I1: Experimental and Genetic Considerations. Obstet and Gynecol Survey 2002; 57 (9): 619-40 9. IMSS. Embarazo de alto riesgo. Guía diagnóstica terapéutica. Rev Med IMSS 1998; 36(1):45-60 14.- Femenino de 36 años, con presencia de hiper y polimenorreas o pérdidas de sangre contínuas sin conservación del ciclo, se sospecha de miomatosis uterina, éste tipo alteración se presenta con mayor frecuencia en los miomas de localización: a) Submucoso b) Intramural d) Intraligamentaria e) cervical Descripción Los miomas son tumores monoclonales benignos de las células del músculo liso del miometrio. Están compuestos por grandes cantidades de matriz extracelular que contiene colágeno, fibronectina y proteoglicanos. El colágeno tipo I y tipo II es abundante, pero las fibrillas de colágeno son anormales y están desorganizadas, de modo similar a lo que se observa en la formación de queloides. Los miomas submucosos son los menos frecuentes, constituyendo únicamente el 5% de la totalidad de los miomas, a menudo producen un aumento del sangrado menstrual en forma de hiper y polimenorreas, e incluso hemorragias importantes que exigen tratamiento de urgencia. Las metrorragias también son habituales en este tipo de miomas. Según Novak, el peligro de degeneración sarcomatosa es mucho mayor en los miomas submucosos, e igualmente es causa de dismenorreas más intensas y frecuentes.
  • 18. BIBLIOGRAFÍA 1. De la Fuente U. Tratado de Obstetricia y Ginecología. Mc Graw-Hill. (Madrid). 1998. Volumen II. 2. DI SAIA S. Tratado de Obstetricia y Ginecología de Danforth. Sexta. Mc Graw-Hill. (Nueva York). 1990. Sexta Edición. 3. González-Merlo J. Tratado de Obstetricia y Ginecología. Salvat Editores S.A. (Barcelona). 1990. Quinta Edición. 4. Aller J., Pages G. Obstetricia Moderna. Tercera Edición. Mc Graw-Hill. (Caracas). 1999. 5. Formación Médica Continuada en Atención Primaria. Marzo 1995. Vo. 5, Nº (3). 15.- Se trata de mujer de 26 años, es atendida en consulta por secreción láctea bilateral hace 4 meses y menstruaciones cada 21 días, en escasa cantidad desde hace 6 meses. Antecedentes: hace 6 meses padece gastritis tratada con cimetidina durante 2 meses y posteriormente ha tomado el medicamento de forma irregular. a.g.o.: menarca 12 años, ritmo 30/7, núbil. e.f.: talla 1.63, peso 54 kg. Al efectuar compresión en glándulas mamarias se produce salida de secreción láctea bilateral, resto normal. El estudio de elección en esta paciente para confirmar el diagnóstico es: a) perfil hormonal ginecológico. b) prolactina sérica. c) radiografía de silla turca. d) ultrasonido mamario. La hiperprolactinemia (hiperprl) es un trastorno frecuente, más en la mujer que en el hombre, y puede ser la manifestación de un adenoma hipofisario (prolactinoma). La medición de prolactina (prl) en la sangre es ahora un procedimiento de rutina en la detección de trastornos de la menstruación y galactorrea que se resuelven efectivamente con el uso de fármacos con actividad dopaminérgica (bromocriptina y cabergolina). La causa más frecuente de hiperprl es por el uso de fármacos que inhiben la acción de dopamina, pero se debe descartar inicialmente hipotiroidismo y un prolactinoma mediante resonancia magnética. Los prolactinomas son fácilmente controlados con dopaminérgicos y excepcionalmente se requiere cirugía. En caso de infertilidad por exceso de prl existe una respuesta favorable con bromocriptina o cabergolina, aunque en la actualidad se prefiere la última por su efectividad y menos efectos indeseables.
  • 19. Hiperprolactinemia. Guía terapéutica y diagnóstica Arturo Zárate* * Unidad de investigación de endocrinología, diabetes y metabolismo, Centro médico nacional IMSS y hospital ángeles México. 16.- Se trata de femenino de 31 años nuligesta pero con actividad sexual regular, sin método de planificación familiar, con ciclos regulares, sin leucorrea, refiere dispareunia profunda, sangrado intermestrual y dismenorrea secundaria ocacionalmente presenta urgencia urinaria, el método de elección para confirmar el diagnóstico clínico de ésta paciente es: a) Cultivo de exudado vaginal b) Ultrasonido c) Biopsia de endometrio d) Laparoscopia con biopsias dirigidas
  • 20. Endometriosis Examinación bimanual Dolor en los ligamentos utero‐sacros Nodularidad en el fondo de saco Induración del septo rectovaginal Útero en retroversión fijo Masa anexial Endometriosis Laparoscopia visualización directa estadificación toma de muestras Estadio I Minimo Estadio II Leve Estadio III Moderado Estadio IV Severo Referencias bibliográficas 1. Ruiz V. Endometriosis y fertilidad. Ed. Acosta y Warman, pp. 99 2. López, VH. Palomo E. Incidencias de endometriosis en una población infértil. XXI Congreso nacional de Ginecología y Obstetricia. Guatemala, 1993. 3. El-Eoley, et al. Danazol but not ginadotropin releasing hormone agonists suppresses autoantibodies in endomeriosis. Fertil Steril 1990; 54:725 4. Acosta AA. Buttram VC Jr. Besch PK, Malinak LR, Van Der Heyden J. A.proposed classfication of pelvic endometriosis. Obstet Gynecol 1973;42:19. 5. Buttran VC Jr. Evolution of the revised American Fertility classification of endometriosis. Fert. Steril 1985; 43: 347 6. López VH. Tratamiento médico-quirúrgico de la endometriosis. Simposio El rostro cambiante de la endometriosis panamá 3. 12. 1993.
  • 21. 7. Steinleitner A. Heterolous transplation of activated murine peritonel macrophages inhibitis gamete interaction in vivo; A paradigm fo endometriosis associted subfertility. Fertil Steril 1990; 54:725. 8. Damewood M. Effect of serum from patients with minimal to mild endometriosis on mouse embryo growth. Fertil Steril 1990; 54: 917 9. Proug S. Peritoneal fluid fracctions from patients with endometriosis do not promote two-cell mouse embryo growth. Fertil Steril 1990; 54: 927. 17.- Femenino de 22 años, que presenta una tumoración de 2 cm de diámetro en el cuadrante ínfero-externo de la mama izquierda, indolora, de consistencia firme, superficie lisa, forma ovoidea, móvil y bien delimitada del parénquima vecino, sin antecedentes de derrame por el pezón, sin “piel de naranja” ni retracción del pezón. El diagnóstico más probable es: a) Fibroadenoma. b) Carcinoma. c) Ectasia de los conductos mamarios. d) Quiste solitario. FIBROADENOMA MAMARIO Tumor benigno más frecuente en las mujeres entre los 20 y 35 años. ETIOLOGIA Existen múltiples teorías siendo la más aceptada la hormonal, generalmente son únicos, solo el 20% son múltiples o bilaterales. De tamaño variable hasta de 10 cm. Ocupa el 13.6% de la patología mamaria benigna. CUADRO CLÍNICO Lesión nodular de consistencia dura, de larga evolución y no dolorosa. Normalmente llegan a los 3 cm. De diámetro. Durante la fase tardía del ciclo menstrual el tumor suele presentar un leve aumento de tamaño. Durante la menopausia presentan regresión hasta la calcificación (signo de palomitas de maíz). DIAGNOSTICO Es clínico, se presenta como un tumor bien delimitado, desplazable, no adherido a piel ni a planos profundos, liso o multilobulado en ocasiones. Se localiza frecuentemente en cuadrantes externos. EXAMENES DIAGNOSTICOS ULTRASONIDO MAMARIO .- Identifica un nódulo sólido, bien delimitado de bordes regulares . TRATAMIENTO.- Conservador con vigilancia estrecha dependiendo del tamaño y en caso de ser necesario exéresis del nódulo para estudio histopatológico hospitalgeneral.salud.gob.mx/ BIBLIOGRAFIA
  • 22. 1. Sánchez BC. Tratado de Enfermedades de la glándula mamaria. Ed. Manual Moderno. Cap. 13- 15. 2.- De Vita V. Cancer of the Breast. In Cancer: Principles and Practice of Oncology: Fifth Ed. Philadelphia: Lippincott-Raven, Chapter 36; pp: 1521-1616. 3.-Consenso Nacional Acerca del Tratamiento de Cáncer de Mama. En Tumores de mama: Diagnóstico y Tratamiento. 2ª Ed. McGraw-Hill Interamericana; pp: 119-126. 4.-Eberlein T. Current management of carcinoma of the breast. Ann Surgery 1994; 220: 121- 136. 18.- A 26-year-old woman presents with malodorus gray-wellow discharge. You take a wet mount preparation and observe “Clue cells”. The agent of this infection and its treatment is: a) Gardnerella vaginalis / clindamycin b) Gardnerella vaginalis / Ketoconazole c) Trichomona vaginalis / metronidazole / treat the partner d) Candida albicans / nistatin Gardnerella vaginalis fue clasificada como una sola especie y fue establecida como agente causal de la vaginosis (antes conocida como vaginitis inespecífica). El cuadro clínico que presenta es caracterizado por una secreción blanca o blanco-grisácea que se percibe generalmente después de la relación sexual con olor fétido aminado (pescado). El diagnóstico certero es la base para evitar posibles complicaciones como la enfermedad inflamatoria pelviana y las complicaciones del embarazo. El tratamiento se basa principalmente en los fármacos como: metronidazol y clindamicina, debido a su efectividad y espectro, pero como todos se deben emplear con adecuada prudencia debido a su toxicidad. Además de que se deben corregir o modificar los factores predisponentes, ya que esta patología va en aumento convirtiéndose por su frecuencia en un problema de salud pública. Bibliografía: 1. Hernández F. Gardnerella vaginalis mobiluncus en la etiología de la vaginosis bacteriana. Rev Costarricense Ciencias Médicas 1998; 19: 57-61. 2. Hansen EA. Gardnerella. Rev Ginecol 2005; 25: 99. 3. Espinosa I, Lorenzo M, Bentancourt A, Riverón Y, Romero M. Caracterización bioquímica y antigénica de diferentes aislamientos de Gardnerella vaginalis. Rev Cubana Invest Biomed 2005; 24: 22-7. 4. Taylor F. Vaginal flora morphotypic profiles and assessment of bacterial vaginosis in women at risk for HIV infection.Infect Dis Obstet Gynecol 2004; 12: 121-6. 19.- Una mujer embarazada, puede afectar al feto y hacerlo contraer lesiones importantes durante el embarazo o al salir al exterior (atravesando el canal de parto), sí la gestante se encuentra afectada de la siguiente patología: a) Tricomonas b) Gardenerella c) Herpes genital d) Gonococos
  • 23. Herpes genital La prevalencia de herpes simplex genital o tipo 2 (VHS-2) en mujeres embarazadas varía entre 7 y 33% en distintas series. La prevalencia ha experimentado un sostenido aumento durante los últimos años. Se estima que aproximadamente 1 a 3% de las mujeres adquiere cada año la infección. En el caso de parejas discordantes, la tasa de adquisición aumenta hasta 10 a 30% anual. La adquisición durante el embarazo es ~2%. La transmisión al hijo ocurre principalmente cuando la mujer embarazada adquiere una infección primaria. La transmisión es de 30 a 50% cuando la infección primaria ocurre cerca del momento del parto. La mayor transmisión (85%) ocurre durante el parto. Sin embargo, también puede ocurrir transmisión intrauterina (5-8%) y post-natal (8-10%). Los factores que inciden en la transmisión son: infección primaria mucho mayor eficiencia que infección recurrente, parejas discordantes, títulos de anticuerpos maternos y procedimientos obstétricos invasores, (los que están absolutamente contraindicados). Las manifestaciones en la mujer embarazada son principalmente bajo la forma de herpes genital localizado, muy raramente ocurre diseminación cutánea y visceral, situación de elevada mortalidad (50%). La infección en el niño, si ocurre en las primeras 20 semanas del embarazo, puede provocar aborto en 25%, malformaciones cerebrales, cicatrices, corioretinitis, RCIU. Si ocurre después de las 20 semanas, puede causar parto prematuro, RCIU, o herpes neonatal. Esta condición clínica tiene tres formas de presentación, las dos primeras de elevada mortalidad y secuelas: herpes diseminado y encefalitis herpética o infección localizada en piel, ojo y boca. Pass R, Weber T, Whitley RJ. Herpesvirus infections in pregnancy. Recommendations from the International Herpes Management Forum. Management Strategies Workshop and 7th Annual Meeting. Whitley R J. Varicella - Zoster virus. Mandell, Douglas and Bennett's Principles and Practice of Infectious Diseases. Mandell G, Bennett J, Dolin R, editors. Fifth edition, 2000 Churchill Livingstone, Philadelphia, pp: 1586-98. Abarca K, Cohen B J, Vial P. Seroprevalence of parvovirus B19 in urban Chilean children and young adults, 1990 and 1996. Epidemiol Infect 2002; 128: 59-62. 20.- Femenino de 20 años, atendida en sala de urgencias ginecoobstétricas, Antecedente: cursa embarazo de 38 SDG. Exploración Física: en trabajo de parto. Repentinamente presenta sangrado profuso transvaginal y dolor abdominal. A la paciente se le coloca un monitor fetal externo. El tono uterino está incrementado y hay desaceleraciones variables y ocasionales de la FCF que llegan a 90. El manejo apropiado en esta paciente es: a) Tocolisis con un agonista beta adrenérgico b) Continuar el monitoreo de la madre y el producto c) Amniotomía d) Cesárea El manejo de éstas pacientes es estricto y urgente: 1- Ingreso inmediato de la paciente en vigilancia a sala de partos. 2- Venopuntura. Si existen signos de shock, instaurar preferentemente dos vías, una central y otra periférica. 3- Solución de Ringer. 4- Monitoréo de signos vitales cada 15’. 5- Sonda vesical y monitoréo de diuresis. Diuresis de 50 ml/h asegura perfusión periférica correcta.
  • 24. Evitar diuresis inferiores a 30 ml/h. 6- Monitoréo contínuo de LCF. 7- Laboratorio de urgencia: hemograma, ionograma, urea y creatinina, coagulograma. 8- Sangre como mínimo 4U para reserva (concentrado de glóbulos rojos o sangre entera). 9- Ecografía obstétrica para localizar y medir el hematoma y descartar otras causas de sangrado. Dependiendo del grado de desprendimiento realizar: 1- Desprendimiento leve: sin compromiso materno ni fetal y el embarazo es de pretérmino, conducta expectante. Uteroinhibición y maduración pulmonar fetal. 2- Desprendimientos moderados a severos: si hay compromiso materno o fetal u óbito fetal, terminar inmediatamente la gestación. 3- Independientemente de la vía del parto, es preceptiva la práctica de la amniotomía, para reducir la presión intraamniótica y disminuir el paso de tromboplastina hística a la circulación y la extravasación de sangre al miometrio. Las principales complicaciones son: 1- Hipotensión, shock hipovolémico. 2- Coagulación intravascular diseminada. 3- Necrosis cortical y necrosis tubular aguda renal. Obstetricia. Scwarcz, Sala, Duverges. 7ª edic. Edit. El Ateneo. (Biblioteca Fac. Med. UNNE). 21.- Mujer de 27 años a quien después de estudios de laboratorio y exploración física se le diagnostica hirsutismo asociado a ovario poliquístico, el siguiente fármaco que usted elige para su tratamiento por ser el más adecuado es: a) Clomifeno b) Estrógenos c) Corticoide d) Acetato de ciproterona El acetato de ciproterona parece ser más efectivo que otros fármacos para el hirsutismo en mujeres causado por la producción ovárica excesiva de andrógenos Una de las causas de hirsutismo (crecimiento piloso excesivo) en mujeres es la hiperproducción de andrógenos a partir del ovario. Varios fármacos pueden utilizarse para contrarrestar los efectos del andrógeno. El acetato de ciproterona es un fármaco antiandrogénico. Los efectos adversos informados con su uso fueron aumento de peso, depresión, fatiga, síntomas mamarios y disfunción sexual. La revisión de los ensayos encontró que el acetato de ciproterona parece ejercer un efecto en el hirsutismo similar a otros fármacos utilizados para el tratamiento del hirsutismo por exceso de andrógenos. No existen pruebas suficientes para comparar los efectos adversos de las opciones de tratamiento. Van der Spuy ZM, le Roux PA. Acetato de ciproterona para el hirsutismo (Revisión Cochrane traducida). En: La Biblioteca Cochrane Plus, número 4, 2007. Oxford, Update Software Ltd. Disponible en: http://www.update-software.com. (Traducida de The Cochrane Library, 2007 Issue 4. Chichester, UK: John Wiley & Sons, Ltd.).
  • 25. 22.- Mujer de 36 años nulípara tras 3 años de relaciones sexuales sin contracepción, desde hace 1 año inició presentando dismenorrea, dispareunia y sangrado vaginal intermenstrual. El diagnóstico más probable de ésta paciente es: a) Insuficiencia luteínica. b) Enfermedad inflamatoria pélvica. c) Dismenorrea funcional. d) Endometriosis. La endometriosis consiste en la aparición y crecimiento de tejido endometrial fuera del útero, sobre todo en la cavidad pélvica como en los ovarios, detrás del útero, en los ligamentos uterinos, en la vejiga urinaria o en el intestino. Es menos frecuente que la endometriosis aparezca fuera del abdomen como en los pulmones o en otras partes del cuerpo. La endometriosis es una enfermedad relativamente frecuente, que puede afectar a cualquier mujer en edad fértil, desde la menarquia hasta la menopausia, aunque algunas veces, la endometriosis puede durar hasta después de la menopausia. La endometriosis altera la calidad de vida de las mujeres que la padecen, afectando a sus relaciones de pareja, familiares, laborales y de reproducción. Síntomas Los síntomas clásicos son la dismenorrea, dolor pélvico, dispareunia, sangrados intermestruales y en muchos casos, esterilidad. El dolor no tiene que ver con el tamaño y la severidad de la lesión; generalmente cuanto menor es la lesión mayor dolor produce. El dolor se agrava con las menstruaciones y en los casos en que la lesión ocupa el fondo de saco de Douglas, puede dar dispareunia. Existe un aumento de la PGF2 alfa y PGE2 y un aumento de las contracciones uterinas que podría deberse a un depósito de endometrio en la cavidad peritoneal. La esterilidad debido a la endometriosis podría deberse a distintas causas de acuerdo a la severidad de la patología. En los casos de endometriosis severa puede haber un factor tuboperitoneal con adherencias y alteración en la anatomía de la pelvis que interfiera con el transporte del esperma y el óvulo. En los casos de endometriosis leve hay varios mecanismos propuestos que justifican su relación con la infertilidad: foliculogénesis alterada, fase lútea inadecuada, fagocitosis espermática, mala calidad ovocitaria, embriotoxicidad y alteración a nivel de la implantación.. La producción de prostaglandinas por el endometrio ectópico puede afectar la motilidad tubaria, la foliculogénesis y la función del cuerpo lúteo. Puede haber un aumento de la activación de los macrófagos peritoneales en la endometriosis que cause la fagocitosis de los espermas o la secreción de citoquinas que pueden ser tóxicas para el embrión. Según algunos investigadores habría un 60% de las mujeres con endometriosis que
  • 26. presentan un síndrome de Folículo Luteinizado no roto (LUF) en el cual el folículo no se rompe en la ovulación y el óvulo queda atrapado. Referencias bibliográficas 1. Ruiz V. Endometriosis y fertilidad. Ed. Acosta y Warman, pp. 99 2. Lópes,VH. Palomo E. Incidencias de endometriosis en una población infértil. XXI Congreso nacional de Ginecología y Obtetricia. Guatemala, 1993. 3. El-Eoley, et al. Danazol but not ginadotropin releasing hormone agonists suppresses autoantibodies in endomeriosis. Fertil Steril 1990; 54:725 4. Acosta AA. Buttram VC Jr. Besch PK, Malinak LR, Van Der Heyden J. A.proposed classfication of pelvic endometriosis. Obstet Gynecol 1973;42:19. 5. Buttran VC Jr. Evolution of the revised American Fertility classification of endometriosis. Fert. Steril 1985; 43: 347 6. López VH. Tratamiento médico-quirúrgico de la endometriosis. Simposio El rostro cambiante de la endometriosis panamá 3. 12. 1993. 7. Steinleitner A. Heterolous transplation of activated murine peritonel macrophages inhibitis gamete interaction in vivo; A paradigm fo endometriosis associted subfertility. Fertil Steril 1990; 54:725. 8. Damewood M. Effect of serum from patients with minimal to mild endometriosis on mouse embryo growth. Fertil Steril 1990; 54: 917 9. Proug S. Peritoneal fluid fracctions from patients with endometriosis do not promote two-cell mouse embryo growth. Fertil Steril 1990; 54: 927. 23.-Se trata de femenino de 37 años de edad, a quien se realiza diagnóstico de placenta percreta, el tratamiento e elección en esta patología es: a) Histerectomía. b) Ergonovina a dosis altas. c) Hemostasia con puntos transfictivos. d) Taponamiento uterino. La placenta anormalmente adherida es poco común y tiene importancia clínica por su morbimortalidad, a consecuencia de hemorragia, perforación, invasión y lesión de las vías urinarias. Esta adherencia anormal está asociada con la implantación placentaria sobre cicatrices de cesárea previa, incisiones uterinas o legrados. La placenta percreta consiste en la penetración del tejido placetario a través de toda la pared uterina, traspasando la serosa de la misma. La identificación de esta anormalidad antes del parto es posible mediante métodos de imagen (escala de grises por ultrasonido, ecografía Doppler color pulsado o resonancia magnética nuclear). El tratamiento conservador se acompaña de elevada morbilidad en muchos casos, por lo que el tratamiento quirúrgico se convierte en el definitivo. La
  • 27. literatura sugiere un aumento previsto en la incidencia de esta condición con base en el incremento del número de cesáreas, por lo que la histerectomía postcesárea será una decisión que enfrentarán los especialistas con mayor frecuencia. Perucca E, Domínguez C, Yahng Ch, García R. Placenta previa percreta con invasión vesical. Rev Chil Obstet Ginecol 1997; 62(3): 206-10. 2. Abbas F, Talati J, Wasti S et al. Placenta percreta with bladder invasion as a cause of life threatening hemorrhage. J Urol 2000; 164: 1270-4. 3. Perucca E, Cazenave H, Barra A, Ochoa N, Villagrán G, Espinoza R, Estay R, Bustamante R, Siebert A. Placenta previa percreta con invasión vesical. Rev Chil Obstet Ginecol 2002; 67(5): 364-7. 4. Price F, Resnik E, Heller K, Christopherson W. Placenta previa percreta involving de urinary bladder. A report of two cases and review of the literature. Obstet Gynecol 1991; 78(3): 508-11. 24.- Se trata de paciente femenino de 37 años de edad, G-3, C-2. Es ingresada a hospital presentando cefalea, acúfenos, fosfenos y epigastralgia en barra con embarazo de 34 semanas. E.F T/A 160/110, FC 84 x´, FR 18 x´, no presenta fiebre, somnolienta, sin agregados cardioventilatorios, hepatalgia. F.U. de 25 cm. Producto único vivo. FCF 110 lpm, genitales sin pérdidas ni modificaciones cervicales. Laboratorio: hb 9.8 g/dl, plaquetas de 54 mil, TP 11 seg TPT 27, TGO 160 ng/dl TGP 160 ng/dl, hiperbilirrubinemia indirecta, albuminuria 300 mg/dl, Ácido Úrico de 8.1 mg/dl, creatinina de 1.5 mg/dl. El diagnóstico más probable es: a) Sindrome de hellp. b) Sx anticuerpos antifisfolípidos c) Púrpura trombocitopénica trombótica d) Hígado graso DEFINICIÓN: • Es una complicación de la preeclampsia en la cual además de la Hipertensión Arterial y proteinuria hay presencia de anemia hemolítica, enzimas hepáticas elevadas y recuento bajo de plaquetas MANIFESTACIONES CLINICAS: • Malestar general, fatiga y molestias inespecíficas 90% • Cefalea 70% • Epigastralgia 64% • Vómito 22% • Fosfenos 15% • Visión Borrosa 11% • Acùfenos 3% • Ictericia • Anemia no explicada • Oliguria
  • 28. BIBLIOGRAFIA: Sibai baha, El síndrome HELLP. Universidad de Valencia , revista quincenal de Obstetricia clínica y ginecología, Octubre 2003. V. Cararach, Síndrome de HELLP y Repercusiones maternas. X curso intensivo de formación continuada materno fetal. Enero de 2003. Toirac, Abelardo. Síndrome de Weistein HELLP Hospital Ginecoobstetrico Tamara Bunke. Junio 2002 De la Fuente, David. Síndrome HELLP. Medicina Universitária 2003; 5 (19): 101 -9 Andrea G. Witlin, DO, Baha M. Sibai, MD. Diagnosis and Management of women with Hemolysis Elevate Liver Enzymes, and Pletelet Count (HELLP) syndrome. Hospital Physician. Febrero 1999. CIFUENTES B, Rodrigo. Ginecología y obstetricia.
  • 29. 25.- Se trata de femenino de 24 años, G-1, que cursa con embarazo de 37 semanas de gestación, presenta pérdida del estado de alerta posterior a crisis convulsivas tónico-clónicas, signos vitales con T-A 170.120mmhg Fc 95x´, reflejos osteotendinosos aumentados, se aprecia una Fc fetal de 132x´ y edema importante de miembros inferiores, no se aprecian datos de trabajo de parto ni modificaciones cervicales, El diagnóstico más probable es: a) Pre eclampsia severa b) Crisis epileptica de gran mal c) Hipertensión inducida por el embarazo d) Eclampsia CUADRO 1. DIAGNÓSTICO* Preeclampsia Leve: Se presenta después de la semana 20 de gestación, durante el parto, o en las primeras 6 semanas después de éste Presión sistólica ≥ a 140 mm Hg o presión diastólica ≥ 90 mm Hg Proteinuria ≥ a 300 mg / orina de 24 hrs o su equivalente en tira reactiva Preeclampsia Severa: Se presenta después de la semana 20 de gestación, durante el parto, o en las primeras 6 semanas después de éste Presión sistólica ≥ a 160 mm Hg o presión diastólica ≥ 110 mm Hg Proteinuria ≥ a 2 gr en orina de 24 horas o su equivalente en tira reactiva Creatinina sérica > a 1.2 mg/dl Trombocitopenia ≤ 150 000 cel/mm3 Incremento de la deshidrogenasa láctica ≥ a 600 UI Elevación al doble de los valores de TGO/AST o TGP/ALT Cefalea, alteraciones visuales o auditivas Epigastralgia Oliguria ≤ a 500 ml en 24 horas 7 Edema agudo de pulmón Dolor en hipocondrio derecho Restricción en el crecimiento intrauterino Oligohidramnios Eclampsia Preeclampsia mas convulsiones sin otra causa. Se presenta después de la semana 20 de gestación, durante el parto, o en las primeras 6 semanas después de éste. Síndrome de HELLP Criterios para establecer el diagnóstico del síndrome de HELLP: Plaquetas < 100 000/mm3 TGO/AST ≥ 70U/L DHL ≥ 600U/LBilirrubina total > 1.2 mg/dl Se presenta después de la semana 20 de gestación, durante el parto, o en las primeras 6 semanas después de éste. Hipertensión Crónica: Se diagnostica cuando existe hipertensión arterial ≥ a 140/90 mm Hg antes de la semana 20 de gestación o si persiste después de doce semanas posteriores al parto. Las pacientes con hipertensión crónica deben ser evaluadas antes del embarazo para determinar la severidad de la hipertensión y facilitar la planeación de un embarazo mediante el cambio de medicamentos y de hábitos higiénicos y dietéticos para evitar complicaciones. Hipertensión Gestacional: Presencia de hipertensión arterial ≥ a 140/90 mm Hg después de la semana 20 de gestación y se mantiene hasta las doce semanas después del parto Ausencia de proteinuria Presencia o no de cefalea, acúfenos y fosfenos Después de 12 semanas de la interrupción del embarazo se revalorará la presencia de hipertensión, si continúa, se reclasifica como hipertensión crónica: es un diagnóstico retrospectivo. 8 Si no hay, se clasifica como hipertensión transitoria. 1. Aagaard-Tillery KM, Belfort MA. Eclampsia: morbidity, mortality, and management. Clin Obstet Gynecol 48:12-23, 2005.
  • 30. 2. Atallah AN, Hofmeyr GJ, Duley L. Calcium supplementation during pregnancy for preventing hypertensive disorders and related problems. Cochrane Database Syst Rev 1:CD001059, 2001. 3. Barton JR, Sibai BM. Diagnosis and management of hemolysis, elevated liver enzymes, and low platelets syndrome. Clin Perinatol 31:807-33, 2004. 4. Baxter JK, Weinstein L. HELLP syndrome: the state of the art. Obstet Gynecol Surv 59:838- 45, 2004. 5. Cetin A. Eclampsia. In Mohler III ER, Townsend RR. Advanced therapy in hypertension and vascular disease. Ontario: B.C. Decker Inc. pp. 407-15, 2006. 6. Cetin A. Hemolysis, elevated liver enzymes, and low platelets (HELLP). In Mohler III ER, Townsend RR. Advanced therapy in hypertension and vascular disease. Ontario: B.C. Decker Inc. pp. 416-20, 2006. 7. Chappell LC, Seed PT, Briley AL, Kelly FJ, Lee R, Hunt BJ, Parmar K, Bewley SJ, Shennan AH, Steer PJ, Poston L. Effect of antioxidants on the occurrence of pre-eclampsia in women at increased risk: a randomised trial. Lancet 354:810-16, 1999. 26.- Femenino que cursa con 12 semanas de gestación, refiere tres días con sangrando por genitales, con náuseas continuas y constantes, útero mayor aumentado de tamaño y valores de beta HCG muy elevados, debemos sospechar de: a) Amenaza de aborto. b) Aborto diferido. c) Mola hidatídica. d) Amenaza de aborto en un útero con miomas. La enfermedad trofoblástica gestacional agrupa a diferentes entidades interrelacionadas: mola completa, generalmente diploide con origen cromosómico paterno, mola parcial generalmente triploide, tumor trofoblástico del lecho placentario y coriocarcinoma, con tendencias variables a la invasión local y a las metástasis, cuyo denominador común es la hipersecreción de hCG. El coriocarcinoma es diploide y proviene de ambos progenitores, excluyendo probablemente su origen directo en la mola completa. El tumor trofoblástico del lecho placentario está constituido por trofoblasto mononuclear intermedio no conteniendo vellosidades coriónicas e inmunohistoquimicamente caracterizado por expresar muchas de sus células hPL y unas pocas hCG. Cuadro clínico
  • 31. Tras un periodo de amenorrea, y a partir del segundo mes, hay un aumento de los síntomas subjetivos del embarazo, sobre todo náuseas y vómitos o hiperémesis en un 30%. Hay metrorragias irregulares en el 96% de los casos, en principio de escasa cantidad, pero que se va incrementando. Dicha hemorragia no es continua sino que se repite cada dos o tres días, de color roja o negruzca, que se produce por la ruptura de vasos maternos al separarse las vesículas de la decidua. Ocasionalmente, hay expulsión de restos molares, que lo refiere la paciente como expulsión de vesículas en el 11 % de los casos y que es patognomónico pero aparece tardíamente. También puede presentar mal estado general, dolor difuso en hipogastrio y anemia. Esta última en relación con las metrorragias. Pueden haber signos y síntomas de hipertiroidismo, como taquicardia, sudoración, y temblores, en el 7%, y es debido a que la fracción β-hCG es similar a la hormona TSH. Existen signos de preeclampsia o hipertensión gestacional del primer trimestre hasta en el 50% de los casos, y signos de insuficiencia respiratoria aguda en el 2% por embolismo pulmonar de las células trofoblásticas, o por la asociación entre hipertiroidismo e hipertensión arterial. Como complicaciones pueden aparecer coagulopatías y metástasis. A la exploración el útero esta aumentado de tamaño, en el 50% de los casos, por encima de lo que correspondería a la edad gestacional,3 de consistencia blanda, sin signos de actividad fetal a partir de la semana 12 y siempre que se trate de una mola total. También puede que la paciente presente un tamaño uterino menor al esperado para la edad gestacional.3 El cuello está cerrado, con metrorragia en cantidad variable, y raramente se observa la expulsión de vesículas. Pueden haber quistes teca-luteínicos bilaterales en ovario en cerca del 20% de los casos,3 debido al estímulo de la β-hCG.
  • 32. La enfermedad trofoblástica maligna va a cursar con metrorragias por lo general intensas, hay una elevación de la β-hCG y en la ecografía se objetiva la cavidad uterina con signos de ocupación atípica. Diagnóstico Por la clínica, y pruebas complementarias como la determinación de la β-hCG y la ecografía. La determinación de la β-hCG se basa en que el trofoblasto produce la hormona gonadotropina coriónica, presentando cifras elevadas, y su cuantificación va a servir para diagnóstico, valorar el pronóstico, y el seguimiento postratamiento. La ecografía revela un útero aumentado de tamaño que no corresponde con la amenorrea, con ecos en su interior, puntiformes que corresponderían a las vesículas y que asemejan copos de nieve o panal de abeja. No se aprecia saco gestacional ni estructuras fetales y, en ambos ovarios se aprecian quistes teca- luteínicos como formaciones ováricas redondas, econegativas, con múltiples tabiques en su interior
  • 33. BIBLIOGRAFÍA 1. Mazur MT, Kurman RJ. Gestational trophoblastic disease and related lesions. En: Kurman RJ editor. Blaunstein’s pathology of the female genital tract. 4th ed. New York: Springer-Verlag. 1994, p. 1049-93. 2. Kurman RJ, Young RH, Norris HJ, Main CS, Lawrence WD, Scully RE. Immunocytochemical localization of placental lactogen and chorionic gonadotrophin in the normal placenta and trophoblastic tumors, with emphasis on intermediate trophoblast and the placental site trophoblastic tumor. Int J Gynecol Pathol 1984; 3: 101-21. 3. Berkowitz RS, Golstein DP. The management of molar pregnancy and gestational trophoblastic tumours. En Knapp RC, Berkowitz RS, editores. Gynecologic Oncology, 2nd ed. New York: Mc Graw-Hill 1992, p. 328-38. 4. De Agustín P, Ruiz A, López F, Contreras F. Patología de la enfermedad trofoblástica. Simposio Enfermedad Trofoblástica 1972; 79-98. 5. Salem S. Ultrasound diagnosis of trophoblastic disease. En: Sanders RC, James AE(Jr) editores. Ultrasonography in Obstetrics and Gynaecology. New York: Appleton-Century Crofts: 1977; p. 255-66. 6. Silverberg SG, Kurman RJ. Tumors of the uterine corpus and gestational trophoblastic disease. En: Rosai J, Sobin LJ, editores. Atlas of tumor pathology: tumors of the uterine corpus and gestational trophoblastic disease, fasc. 3, ser. 3. Washington DC: Armed Forces Institute of Pathology; 1992, p. 219-85.
  • 34. Szulman AE, Surti U The syndromes of hydatiform mole II. Morphologic evolution of the complete and partial mole. Am J Obstet Gynecol. 1978; 132: 20-7. 27.- Mujer que acude al servicio de ginecología por referir ciclos opso-menorreicos desde el inicio de su menarquia, ha incrementado 15 Kg. de lo que pesaba habitualmente, se aprecia una gran cantidad de acné, pero además refiere depilarse el área del bigote cada semana, y cree que esto le sensibiliza la piel para que aumente el acné. En el caso de ovario poliquístico el dato clínico que con más frecuencia les acompaña es: a) anovulación y esterilidad b) Hirsutismo c) Amenorrea d) Obesidad El síndrome de ovarios poliquísticos (SOPQ) afecta aproximadamente a un 4% de mujeres en edad reproductiva y se caracteriza por anovulación crónica e hiperandrogenismo. Es la causa más común de infertilidad en mujeres. Se caracteriza clínicamente por acné, alopecia, hirsutismo, irregularidades menstruales e infertilidad. Los hallazgos de laboratorio más frecuentes son: aumento de la hormona luteinizante (LH), aumento de la relación LH/FSH (hormona folículoestimulante), aumento de andrógenos (tanto ováricos como adrenales) y de estrógenos circulantes. Otros hallazgos de laboratorio habituales son una prueba tolerancia oral a la glucosa anormal y alteraciones en el perfil lipídico. Todo esto junto con las imágenes ecocardiográficas características define al síndrome. La terapéutica permite dos grandes enfoques que pueden superponerse: la corrección de las manifestaciones de hiperandrogenismo y el tratamiento de las alteraciones del eje reproductivo (anovulación, esterilidad). Los antiandrógenos están fundamentalmente indicados para tratar los síntomas virilizantes. Las alternativas para inducir la ovulación son numerosas: al citrato de clomifeno y a la antigua resección en cuña se agregan las gonadotrofinas humanas, pulsos de GnRH (hormona liberadora de gonadotrofinas), medidas o fármacos para modificar los niveles de insulina, y finalmente técnicas quirúrgicas endoscópicas para reducir la masa ovárica. BIBLIOGRAFIA 1. Guzick D.Polycystic ovary syndrome: Symptomatology, pathophysiology, and epidemiology. Am J Ostetric Gynecol 1998; 179 (6): 89-93. 2. Stephen Franks. Polycystic ovary syndrome. N Engl J Med 1995; 333(13): 853-861. 3. Gori J.R., Larusso A. Ginecología de Gori. 2ª Edición. Buenos Aires, Argentina. Editorial El Ateneo. 2001. 4. Adams J., Polson D. W., Franks S. Prevalence of polycystic ovaries in women with anovulation and idiopathic hirsutism. Br Med J 1986; 293: 355-9. 5. Copeland L. J . Ginecología. Buenos Aires, Argentina. Editorial Panamericana. 1ª Edición. 1994. 6. Ehrmann D.A., Rosenfield R.L., Barnes R.B., Brigell D.F., Sheikh Z. Detection of functional ovarian hyperandrogenism in women with androgen excess. N Engl J Med 1992; 327:157-162. 7. Kahasar-Miller M., Conway Myers B., Boots L., Azziz R. Steroidogenic acute regulatory protein (StAR) in the ovaries of healthy women and those with polycystic ovary syndrome. Am J Obstet Gynecol 2001; 185(6): 1381-7. 8. Pérez Sánchez A. Ginecología. Santiago de Chile. Publicaciones Técnicas Mediterráneo. 3ª Edición. 1995. 9. Velázquez E., Mendoza S., Hamer T., Sosa F., Glucck C. Metformin therapy in women with polycistic ovary syndrome reduces hiperinsulinemia, insulin resistence,
  • 35. hyperandrogenemia, and systolic blood pressure, while facilitating menstrual regularity and pregnancy. Metabolism 1994 ; 43: 647-655. 28.- Femenino de 29 años. Acude al servicio de urgencias por presentar salida de líquido vaginal. Antecedentes: G2, P1, cursa embarazo de 36 semanas de gestación exploración física: cervix con 10% de borramiento, 1 cm de dilatación y Tarnier positivo. La complicación más frecuente en esta paciente es: a) Corioamnioitis. b) Parto pre término. c) Sepsis neonatal. d) Endometritis. Corioamnioitis: El diagnóstico de la infección intraamniótica (IIA) es básicamente clínico. La corioamnionitis se debe descartar en toda gestante que presente fiebre sin foco aparente, sobre todo si se sospecha o se ha confirmado una rotura de membranas. Los criterios más empleados para el diagnóstico son: fiebre materna y, al menos, 2 de los siguientes signos: taquicardia materna, taquicardia fetal, irritabilidad uterina, leucocitosis materna o líquido amniótico purulento o maloliente. Progresos de obstetricia y ginecología: revista oficial de la Sociedad española de ginecología y obstetricia, ISSN 0304-5013, Vol. 48, Nº. 6, 2005 , pags. 316-317 29.- Mujer de 23 años diagnosticada de E. Ectópico a nivel ampular, con saco gestacional menor de 3 cm., sin actividad cardíaca embrionaria, asintomática y con niveles de B-HCG menores para su edad gestacional. El tratamiento más indicado es: a) Salpingocentesis b) Resección segmentaria c) Tratamiento médico con Metrotexate. d) Histerectomía total con doble anexectomía. TRATAMIENTO  METOTREXATE  Ácido 4 amino 10 metil folínico, antagonista del ácido fólico  Glucosa hiperosmolar  Prostaglandina F2a  Actinomicina D  Mifespristona TRATAMIENTO CON METOTREXATO  Mayor éxito:  Embarazo menor a 6 SDG  Masa tubaria menor de 3.5cm  Feto sin latido cardíaco
  • 36. PROTOCOLO CON DOSIS ÚNICA DE METOTREXATE Día 0 hGC, Biometría hemática, transaminasas, creatinina, grupo sanguíneo Día 1 hGC Día 4 hGC Día 7 hGC  Disminución de hCG <15%. Segunda dosis  Si la hCG declina seguirla semanalmente  Si la hCG está en meseta o en ascenso, segunda dosis  El raspado endometrial se realiza sólo en pacientes con hCG <2000 mIU/mL al momento de comenzar el tratamiento Graczykowski JW, Mishell DR. Methotrexate prophylaxis for persistent ectopic pregnancy after conservative treatment by salpingostomy. Obstet Gynecol. 30.- Femenino de 19 años acude a consulta por padecer un cuadro de dolor abdominal de inicio súbito, refiere alteraciones en su ciclo menstrual. La exploración física revela una tumoración dolorosa en el anexo izquierdo. La prueba de embarazo es negativa. La radiografía muestra una masa opaca en la fosa ilíaca izquierda con áreas de calcificación. El diagnóstico clínico más probable es: a) Cistadenoma mucinoso b) Teratoma quístico c) Quiste folicular d) Cistadenoma seroso Los tumores de células germinales constituyen casi el 20% de los tumores de ovario y de ellos un 95% son benignos, siendo el tipo más frecuente el teratoma maduro benigno o también denominado quiste dermoide (1). Aproximadamente el 80% se presentan en mujeres en edad fértil. Se originan a partir de células embrionarias pluripotenciales presentes habitualmente en ovario, testículo, mediastino, retroperitoneo y región sacrocoxígea, esto explica que la coloración de las faneras del quiste coincida con el fenotipo del paciente (2). El teratoma quístico benigno con relativa frecuencia es un tumor bilateral (del 7 al 25% según los autores) (1) y se caracteriza por una cápsula gruesa, bien formada, revestida por epitelio plano estratificado. Bajo este se pueden encontrar una variedad de apéndices cutáneos que incluyen glándulas sudoríparas, apocrinas y sebáceas. La cavidad se llena de los detritus de éste y sus anexos, que es de color amarillo pálido, grasoso, espeso y suele contener pelo. Otros tejidos que se pueden encontrar son dientes, cartílago, plexos coroideos, falanges, tejido nervioso y en ocasiones tejido tiroideo (struma ovarii) con potencial tirotóxico o de degeneración maligna tiroidea. La mayor parte de los quistes dermoides son asintomáticos y la forma más frecuente de presentación son el dolor abdominal (48%) y hemorragia uterina anormal o concomitante (15%) o aumento del volumen abdominal (15%). La rotura de un quiste dermoide es rara, entre el 1- 1,2% y constituye una urgencia quirúrgica (3).
  • 37. El tratamiento es quirúrgico, siendo posible la mayor parte de las veces una resección del mismo, respetando el resto del ovario. La ecografía constituye el modelo diagnóstico de elección y la combinación de ecografía con radiografía simple de abdomen proporciona un diagnóstico más exacto en la mayoría de los casos, siendo la resonancia magnética o la TAC el que aporte el diagnóstico diferencial. Las Rx de quiste dermoide se caracteriza por una cápsula bastante radioopaca y el líquido oleoso que contiene es radiolúcido, esta conjugación presenta muchas veces una característica de aspécto radiológico de calcificación en la pared del mismo. Cólico nefrítico, teratoma ovárico y radiografía simple de aparato urinario Romero Pérez P, Martínez Hernández MªC. Servicio de Urología. Policlínico San Carlos Denia (Alicante).Actas Urol Esp. 2007:31(8):936- 937ACTAS UROLÓGICAS ESPAÑOLAS SEPTIEMBRE 2007 31.- Se trata de femenino de 33 años que acude al servicio cursa en éste momento con diagnóstico de preclampsia , el fármaco de elección que se administra en esta patología es: a) Nifedipina. b) Inhibidores de la enzima convertidora de angiotensina. c) Clonidinas. d) Alfametildopa. • Prevenir complicaciones a corto plazo de las mujeres con PA elevada que comprometa el bienestar fetal • Cuando la PAS es mayor o igual a 150 mmHg y la PAD mayor o igual a 100 mmHg.  El propósito es alcanzar cifras de TA alrededor de 140/90.  La medicación antihipertensiva se reserva para los casos en que la PAD ≥ 100 mmHg.  Se recomienda continuar el tratamiento antihipertensivo previo al embarazo, exceptuando el uso de IECA.  La alfametildopa y la hidralazina vía oral son los fármacos de elección dado su uso extensivo con seguridad y eficacia y sin efectos colaterales para el feto (excepto hidralazina en lupus). • ALFA METILDOPA 500-2000 MG/DÍA • HIDRALAZINA 50-200 MG/DÍA • LABETALOL 100-400 MG/DIA • ATENOLOL 50-200 MG/DÍA • NIFEDIPINA 10-30 MG/DÍA
  • 38. 1. Aagard K, Belfort M. Eclampsia: Morbility, mortality, and management. Clin Obstet Gynecolol. 2005; 48: 12-23. 2. Oyarzún E. Síndrome hipertensivo del embarazo en Oyarzún E. Ed. Embarazo de alto riesgo. Ediciones Universidad Católica de Chile. Santiago. 1997: 157- 175. 3. Roberts J, Redman C. Pre-eclamsia: More than pregnancy induced hypertens 32.- Se trata de femenino de 56 años de edad refiere aumento de volumen a nivel abdominal, con predominio de hemiabdomen inferior con la siguiente sintomatología: plenitud, estreñimiento, se acompaña de USG pélvico que demuestra imagen quística en ovario derecho de 15 por 15 cms. El diagnóstico más probable es: a) Teratoma quístico. b) Disgerminoma. c) Endometrioma. d) Cistadenoma seroso. Los Tumores de Ovario son una patología frecuente dentro del contexto de la patología femenina. Por esta causa consultan un grupo elevado de mujeres, tanto las consultas de ginecología como las de Cirugía propiamente dicha. Las edades oscilan desde las tempranas hasta las ya avanzadas, siendo el riesgo de degeneración maligna muy variable y relacionado con le edad. La experiencia de la clínica revela la alta incidencia de tumores de ovario en la etapa del climaterio, comprendida entre los 35 y 65 años de edad 1. El cistoadenoma seroso de ovario (CSO) es un tipo de tumor derivado del epitelio superficial (celómico), formado por áreas quísticas. El cistoadenoma seroso de ovario es el tumor más frecuente de aquellos que provienen del epitelio celómico superficial. Hay tumores pequeños macroscópicamente y tumores masivos que ocupan toda la pelvis e incluso la cavidad abdominal. Estas frecuentes neoplasias quísticas uniloculares están tapizadas por células epiteliales altas, cilíndricas y ciliadas, llenas de un líquido seroso claro y de superficie lisa con abundantes vasos. Las variedades benigna, limítrofe y maligna representan, en conjunto, 30% aproximadamente de todos los tumores del ovario. El riesgo de presentar tumores epiteliales se incrementa con el paso de la edad, ya que pese a que la declinación de la función ovárica marca el envejecimiento gonadal progresivo, el ovario humano nunca pierde su capacidad para generar tumores. Por lo general, cuando es detectado, su tamaño es grande, en donde la imagenología puede ayudarnos a considerar su diagnóstico 1. Capítulo 22 Tumores Benignos de Ovario. En: Novak ER, Jones G., Jokes HW. Tratado de Ginecología. 9 ed. Ciudad de la Habana. Editorial Científico Técnica; 1977.p.432 – 66. 2. MedlinePlus Enciclopedia Médica en Español: Quistes Ováricos. Disponible en: http://vsearch.nlm.nih.gov/vivisimo/cgibin/querymeta?v%3Aproject=medlineplusspanish&spell=s pell&query=Quistes+Ov%C3%A1ricos Acceso: Actualizado 20/6/06. Capítulo XL Tumores Ováricos En: Llusiá Botella J, Núñez Clavero JA. Tratado de Ginecología. Ciudad de la Habana. Editorial Científico Técnica. 1983; T 3.1; p. 751 – 803. 33.- Paciente femenino de 27 años de edad con deseo de un embarazo, antecedentes de G3 A2 - P1 se le realiza una histerosalpingografía, se constata que existe un síndrome de Asherman. Ello significa que se trata de:
  • 39. a) Útero bicorne b) Endometriosis en la trompa c) Sinequias uterinas d) Insuficiencia istmico cervical El síndrome de Asherman es una enfermedad ginecológica rara que se caracteriza por la presencia de sinequias (adherencias) intrauterinas que pueden ocasionar amenorrea (ausencia de períodos menstruales regulares) e infertilidad. En 1894 Heinrich Fritsch describe por primera vez la presencia de sinequias intrauterinas de tipo postraumático, en una paciente que desarrolló una amenorrea secundaria a un curetaje. Posteriormente en 1927 Bass informó de veinte casos de atresia (oclusión de una abertura natural) cervical tras abortos inducidos, pero no fue hasta 1948, cuando Joseph G. Asherman recopiló la información hasta entonces existente y acuñó el nombre con el que se conoce actualmente a la enfermedad. Asherman describió originalmente dos tipos diferentes de amenorrea secundaria, en función de su etiología (estudio de las causas de las enfermedades): la amenorrea traumática atrética, debida a estenosis del orificio cervical interno y la amenorrea debida a adherencias intrauterinas. Posteriormente ambas entidades se agruparon en una única entidad bajo el nombre de síndrome de Asherman. Suele presentarse en mayor proporción tras dilataciones y curetajes uterinos de repetición y sobre todo si se realizan durante el embarazo o si existe infección uterina en el momento en el que se realizan estas intervenciones. Las adherencias intrauterinas pueden producirse debido a cualquier factor que lleve a una destrucción de las paredes del miometrio (capa muscular de la pared del útero). Sin embargo, hay que distinguir entre factores predisponentes, siendo el principal de ellos el embarazo y factores causales, entre los que se encuentran: traumatismos uterinos, intervenciones quirúrgicas que afecten al útero, agentes físicos o químicos e infecciones uterinas por tuberculosis o esquistosomiasis. En cualquier caso, el factor más importante es el trauma uterino en el momento del parto o el puerperio. El cuadro clínico es muy variable y las manifestaciones clínicas varían con el grado de oclusión de la cavidad uterina y la severidad de las adherencias, pudiendo presentarse: esterilidad cuando la oclusión de la cavidad uterina incluye porciones proximales (más cerca de un centro, tronco o línea media) de las trompas de Falopio o cuando las adherencias impiden la nidación del huevo; las pacientes presentan con frecuencia amenorrea, oligomenorrea (disminución de la frecuencia de las menstruaciones), dismenorrea (menstruación dolorosa) y abortos repetidos. 1. Hysteroscopic treatment of severe Asherman's syndrome and subsequent fertility. Capella- Allouc S; Hum Reprod, 1999 May.
  • 40. 34.- Se trata de femenino de 31 años de edad, la cual inicia con hiperemesis gravídica de difícil control, así como sangrado trasvaginal. Se realiza el diagnóstico de mola hidatiforme. La primera opción de tratamiento indicado en esta patología es: a) Histerectomía total abdominal. b) Legrado por aspiración. c) Metotrexate y seguimientos radiográficos. d) Legrado uterino instrumental La enfermedad trofoblástica gestacional agrupa a diferentes entidades interrelacionadas: mola completa, generalmente diploide con origen cromosómico paterno, mola parcial generalmente triploide, tumor trofoblástico del lecho placentario y coriocarcinoma, con tendencias variables a la invasión local y a las metástasis, cuyo denominador común es la hipersecreción de hCG. El coriocarcinoma es diploide y proviene de ambos progenitores, excluyendo probablemente su origen directo en la mola completa. El tumor trofoblástico del lecho placentario está constituido por trofoblasto mononuclear intermedio no conteniendo vellosidades coriónicas e inmunohistoquimicamente caracterizado por expresar muchas de sus células hPL y unas pocas hCG Tratamiento Hay que tratar las complicaciones como la hiperémesis, anemia, hipertensión, y alteraciones electrolíticas, coagulopatías, alteraciones cardio-respiratorias y preeclampsia, procediendo a evacuar la mola lo antes posible, con lo que se producirán menos malignizaciones. La evacuación del contenido uterino se realiza mediante dilatación, y legrado por aspiración. Además se pauta profilaxis antibiótica y oxitócicos.3
  • 41. La histerectomía, con la mola en su interior, está indicada en pacientes de edad superior a 40 años o en mujeres con más de tres hijos, ya que en ambos grupos se ha demostrado una mayor incidencia de malignización. . Tras la cirugía, se mide la concentración de gonadotropina coriónica humana para determinar si la extirpación ha sido completa. Si es así, el valor de esta hormona vuelve a la normalidad, en unas 8 semanas, y se mantiene en esos valores. Si una mujer a la que se le ha extirpado una mola queda embarazada, es difícil interpretar un valor alto de gonadotropina coriónica humana, porque podría estar causado tanto por el embarazo como por una parte de la mola que no se ha extirpado. En consecuencia, a las mujeres a las que se les ha extirpado una mola se les recomienda no quedar embarazadas durante un año. Las molas hidatiformes benignas no necesitan quimioterapia, pero las malignas sí. Los fármacos que se usan para este tratamiento son el metotrexato, la dactinomicina o una combinación de ambos. Silverman L,Romero Zambrano F, Saldaño S. Enfermedad molar. Diagnóstico, tratamiento y seguimiento, 1987. 4- Puertas A, López Fernandez J et al. Enfermedad trofoblástica. Casuística del Hospital Virgen de las Nieves de Granada. Clín Invest Gin Obs 1993; 20: 98-103. 5- Enfermedad trofoblástica gestacional. Propuesta Normativa Perinatológica y Ginecológica de Alto Riesgo. Ministerio de Asuntos Sociales. Tucumán. 1996-1997; 19:171-183. 6- Jones. Enfermedad Trofoblástica Gestacional: qué hemos aprendido en la última década. Am J Gynecol Obstet 1990;162: 1286-1292.
  • 42. 7- Resúmen del simposio Enfermedad Troblástica Gestacional del Segundo Congreso Nacional de AGORA, 1990. 8- Gonzalez Merlo et al. Protocolos de diagnóstico y tratamiento en Obstetricia y Ginecología, Barcelona: editorial Salvat, 7:35-45. 35.- Se trata de femenino de 33 años que ingresa a la sala de urgencias, inicia con convulsiones por presentar preclampsia, decide iniciar tratamiento, el fármaco elección en ésta patología es: a) Diacepam. b) Fenitoína. c) Donadores de óxido nítrico. d) Sulfato de magnesio. Manejo de la Preeclampsia 1. Manejo ambulatorio: HTA sin proteinuria significativa, se recomienda el reposo en cama. Monitoreo de TA, peso, presencia de proteínas en orina. Ecografías periódicas para ver el feto y evaluar posibles retardo de crecimiento. 2. Manejo hospitalario: para mujeres con HTA inducida por el embarazo y 2+ o más o proteinuria significativa y en quienes falló el manejo ambulatorio. 3. Laboratorio y evaluación del peso: debe realizarse diariamente. Evaluación de la dinámica fetal. Monitoreo de síntomas como cefalea, alteraciones visuales y dolor epigástrico. 4. El parto es el tratamiento de elección: el cual debe realizarse cuando el feto está maduro pero puede realizarse en forma temprana si la salud de la madre está en peligro o si hay evidencia de distress fetal. El parto está indicado cuando la paciente cumple con los criterios de preeclampsia severa. Betametasona 12.5 mg IM dos veces por día puede estimular la maduración de los pulmones fetales. 5. Terapia antihipertensiva: está indicada sólo si la TA es persistentemente > 160/110 , es importante disminuir la TA hasta una diastólica de 90 a 100 mmHg porque la presión normal podría resultar en hipoperfusión de la placenta. Los diuréticos nunca están indicados, estas pacientes ya son hipovolémicas. Los IECA no deben ser usados durante el embarazo. Las medicaciones de largo plazo, incluyen alfa metildopa, atenolol y labetalol. 6. Terapia anticonvulsivante: A- Profilaxis de las convulsiones: está indicada en todas las pacientes pre-eclámpticas durante el trabajo de parto y el parto y por un mínimo de 24 hs luego del mismo. Algunos mantienen la terapia con magnesio hasta que comienza la diuresis. El Sulfato de Magnesio es la droga de elección. La dosis profiláctica es de 4 a 6 g de sulfato de magnesio IV y continúa con 2 g c/ hora. B- Tratamiento de las convulsiones: Sulfato de Magnesio 1 g/min IV hasta controlar las convulsiones hasta un máximo de 4 a 6 g. El nivel terapéutico es de 4 meq/l. Toxicidad del magnesio: ausencia de reflejo patelar, debilidad muscular, parálisis respiratoria y depresión cardíaca, 10 ml al 10 % de gluconato de calcio puede ser administrada IV. La terapia con sulfato de magnesio continúa por lo menos 24 horas en el post parto, la terapia puede detenerse si la excreción urinaria es > 200 ml/h por cuatro horas consecutivas. C- Prevención: 81 mg de aspirina diarios pueden ser administrados luego del primer trimestre en mujeres con hipertensión crónica o historia previa de preeclampsia, sin embargo la eficacia de esta indicación ha sido cuestionada. Myers JE, Baker PN. Hupertensive diseases and eclampsia. Curr Opin Obstet Gynecol 2002; 14: 119-125 2. Tierney, McPhee, Papadakis. Diagnóstico clínico y tratamiento 2003. 38ª ed, México, Manual Moderno, 2003: 770-773 3. Wilson MI, Goodwin TM, Pan VI, Ingles SA. Molecular epidemiology of preeclampsia. Obstet and Gynecol Survey 2003; 58(1):39-66
  • 43. 4. Burrow GM. Complicaciones médicas durante el embarazo. 4ª ed, México, McGraw-Hill panamericana: 1996: 1-25 5. Guyton AC, Hall JE. Embarazo y lactancia en: Tratado de fisiología médica, 10ª ed, México, McGraw-Hill Interamericana 2001: 1135-45 6. Vaticon D. Fisiología de la fecundación, embarazo, parto y lactancia, en: Tresguerres JAF. Fisiología Humana. México, Interamericana McGraw-Hill, 1992: 1086-1109 7. Pridjian G, Puschett JB. Preeclampisa. Part 1: Clinical and Pathophysiologic Considerations. Obstet and Gynecol Survey 2002; 57 (9): 598-618 8. Pridjian G, Puschett JB. Preeclampisa. Part I1: Experimental and Genetic Considerations. Obstet and Gynecol Survey 2002; 57 (9): 619-40 9. IMSS. Embarazo de alto riesgo. Guía diagnóstica terapéutica. Rev Med IMSS 1998; 36(1):45-60 36.- Se trata de femenino de 47 años que presenta sinusorrragia de 3 meses, sin alteraciones menstruales previas, por lo demás asintomática, el diagnóstico más probable a la especuloscopía es: a) Adenomiosis b) Malformaciones Müllerianas c) Pólipo endometrial d) Pólipo endocervical Pólipo es toda formación sésil o pediculada que se fija a la cavidad uterina. Las formaciones polipoideas dentro del aparato genital femenino se dividen en endometriales y cervicales. Dentro de los pólipos cevicales se diferencian los ectocervicales y los endocervicales; estos últimos son considerados como los verdaderos por estar constituidos por epitelio cilíndricco. No se conocen con certeza los factores asociados a su histogénesis (teoría más admitida: hiperplasia focal de la mucosa endocervical por estímulo estrogénico) (1). Los pólipos cervicales se consideran la forma más frecuente de tumor benigno de cérvix. Constituyen de un 3 a un 10% de las consultas ginecológicas; presentan máxima incidencia entre la 4° y 5° década de la vida. Algunos estudios demuestran que hasta en el 56% de casos de mujeres postmenopáusicas se encuentran asociados pólipos cervicales y endometriales; es mucho menor esta frecuencia antes de la menopausia (2). La clínica de presentación más común es la hemorragia intermenstrual, espontánea o durante el coito (3). También pueden ser asintomáticos (hallazgo casual en estudio ecográfico) o, en ocasiones, alcanzar grandes tamaños: llegan a aflorar por el orificio cervical externo; se denominan "pólipos gigantes" cuando superan 2 cm (4). Para el diagnóstico habitualmente se utiliza la ecografía. Los casos sintomáticos se consideran de indicación quirúrgica por su riesgo de degeneración a adenocarcinoma cervical. Los pólipos cervicales constituyen una patología frecuente en la consulta ginecológica, sobre todo en mujeres mayores de 20 años que han tenido hijos y son poco comunes antes de la menarquia. En la mayoría de los casos, solamente se presenta un pólipo, aunque en ocasiones se pueden encuentrar 2 ó 3. La época más frecuente es al final de su vida reproductiva y después de la menopausia. En las primeras, las lesiones suelen ser más grandes, difíciles de tratar y con frecuencia recidivan. La causa de los pólipos cervicales aún no se ha comprendido completamente, pero con frecuencia son el resultado de una infección crónica, una respuesta local anormal a los niveles de estrógeno o a una congestión local de los vasos sanguíneoscervicales. Clínicamente se manifiesta con sangrado genital fuera de la menstruación, sobre todo durante la relación sexual. Son benignos, no se malignizan pero deben ser extirpados para controlar la sinusorragia.
  • 44. 37.- A 23-year-old nulligravid female has not menstruated in the past 4 months. Previously, her menstrual cycles were regular. She is otherwise well and denies recent onset of stress, change in exercise routine, headaches, visual field alterations, or galactorrhea.. She has a body mass index of 24, blood pressure of 120/78 mm Hg, and does not appear hirsute. No adnexal masses can be palpated. Laboratory investigations reveal a negative beta human chorionic gonadotropin (β-hCG), normal thyroid-stimulating hormone (TSH), and prolactin levels. What is the next best step in the management of this patient? a) Measurement of luteinizing hormone. b) CT scan of the sella turca. c) Prescribe oral estrogen for 21 days followed by 7 days of medroxyprgesterona and reevaluate. d) Preescribe 7 days of medroxyprgesterona and reevaluate. La Prueba de respuesta a progestágenos se basa en la observación de que el tratamiento con progestágeno (acetato de medroxiprogesterona 10 mg por 5 a 6 días) solo induce la menstruación en las mujeres con concentraciones normales de estrógenos circulantes. Una prueba positiva (hemorragia después de concluir el tratamiento con progestágenos) señala cifras normales de producción de estrógenos y una prueba negativa (sin hemorragia por privación), hipogonadismo franco. Danforth, Tratado de Obstetricia y Ginecologia, 9ª Edición, Ed. Mc Graw Hill Interamericana, Pág 668 38.- Se trata de femenino de 27 años, Gesta 1, Para 1. Con dos citologías “lesión de alto grado”, prueba de Schiller positiva y biopsia de cérvix que demuestra carcinoma “In Situ”. La conducta es: a) Conización. b) Histerectomía total abdominal. c) Histerectomía y salpingooforectomía bilateral. d) Electrocauterización del cérvix. La conización cervical es el tratamiento de elección en pacientes con cáncer cervicouterino microinvasor y más si existedeseo de fertilidad. Asimismo, la histerectomía extrafasciales un método adecuado en lesiones de 0.5 a 3 mm de invasión.Además se propone que, para pacientes con lesiones de 3.1 a 5 mm de invasión, a partir de la membrana inicial sinfactores de mal pronóstico como invasión vascular y linfática, sean tratadas con histerectomía extrafascial, ya que en aquellas a las que se realizó linfadenectomía pélvica, con este tipo de lesión, no se encontró metástasis a ganglios linfáticos. Resultados del tratamiento en cáncer cervicouterino microinvasor en el Instituto Nacional de Cancerología de México (1980-1999)
  • 45. 1.- Mestwerdt G. Fruhdiagnose des Kollumkarzinoms. Zentralb Gynaekol, 1947 ;69 :326. 2. - Morrow CP, Curtin JP. Surgery for cervical neoplasia. In Gynecologic Cancer Surgery. New York, Churchill Livingstone, 1996, p 472.3. 3. - Burghardt E, Holzer E. Diagnosis and treatment of microinvasive carcinoma of the cervix uteri. J Obstet and Gynecol 1977; 49:641-653. 39.- Se trata de femenino de 35 años la cual presenta un nódulo mamario palpable de aparición brusca. La ecografía revela un nódulo anecogénico, de limites muy precisos, morfología regular y refuerzo posterior, único de 3.5 cms. de diámetro. El diagnóstico más probable es: a) Cáncer. b) Displasia fibrosa. c) Fibroadenoma. d) Quiste Quistes. Los quistes mamarios son fáciles de detectar con la ecosonografía. Pueden ser lesiones únicas o múltiples que se observan como imágenes redondeadas, anecogénicas, de paredes delgadas, contornos bien definidos, con importante reforzamiento acústico posterior y sombras laterales delgadas. Pueden presentar septos intraquísticos y, en ocasiones, se pueden observar ecos internos que sugieren detritus celulares o proceso inflamatorio. Se debe descartar la presencia de lesiones intraquísticas o la coexistencia de otras alteraciones benignas o malignas. En caso de ser sintomáticos, el tratamiento adecuado es la punción y aspiración de la lesión con aguja guiada por palpación o ecosonografía de acuerdo con ell tamaño, profundidad y características del contenido. El uso del ultrasonido garantiza el vaciamiento completo.
  • 46. REFERENCIAS: Barth V, Prechtel K. Mama normal. En: Barth V, Prechtel K, editores. Atlas de patología de la glándula mamaria. 2da ed. Madrid: Editorial Médica Panamericana, 1991. Bush H, McCredie A. Carcinoma of the breast during pregnancy and lactation. In: Allen HH, Nisker JA. Cancer in pregnancy. New York: Futura Publishing Co. Inc., 1986. Byrd BF, Bayer DS, Robertson JC, Stephenson JE Jr. Treatment of breast tumor associated with pregnancy and lactation. Ann Surg. 1962; 155:940-7. 40.- Femenino de 36 años, es atendida en consulta externa con reporte de papanicolaou que reporta un NIC I, la especuloscopía se observa cérvix con ectropión periorificiario. El agente etiológico más probable causante de esta infección es: a) neisseria gonorreae. b) clamidya trachomatis. c) virus del papiloma humano. d) treponema pallidum.